Re: [Wien] AFM type II

2020-01-29 Thread pieper

Dear Gerhard,

nice ... thanks a lot for the references!

Best regards,

Martin


---
Dr. Martin Pieper
Karl-Franzens University
Institute of Physics
Universitätsplatz 5
A-8010 Graz
Austria
Tel.: +43-(0)316-380-8564


Am 2020-01-29 13:16, schrieb Fecher, Gerhard:

Dear Martin,
this concerns your remark:
"With two magnetic species, say, Mn and Cu, you would wind up with
different size of the moment on Mn and Cu. I know of no case where 
exact

compensation into an AFM structure occures by accident in such a
situation."

You may have the situation of a completely compensated ferrimagnet 
exampels are:

CrMnSb (or VFeSb) in the cubic C1b structure
H. van Leuken and R. A. de Groot, Phys. Rev. Lett. 74, 1171 (1995)
or more complicated
Mn1.5FeV0.5Al
Rolf Stinshoff et al; Phys. Rev. B 95, 060410(R) (2017)

However, it is by purpose rather than by accident.

This was already found by Neel in his work on antiferromagnets
(probably it is mentioned in the Nobel lecture)



Ciao
Gerhard

DEEP THOUGHT in D. Adams; Hitchhikers Guide to the Galaxy:
"I think the problem, to be quite honest with you,
is that you have never actually known what the question is."


Dr. Gerhard H. Fecher
Institut of Inorganic and Analytical Chemistry
Johannes Gutenberg - University
55099 Mainz
and
Max Planck Institute for Chemical Physics of Solids
01187 Dresden

Von: Wien [wien-boun...@zeus.theochem.tuwien.ac.at] im Auftrag von
pieper [pie...@ifp.tuwien.ac.at]
Gesendet: Mittwoch, 29. Januar 2020 12:49
An: A Mailing list for WIEN2k users
Betreff: Re: [Wien] AFM type II

No one can give you an honest answer without knowing the structure you
put these elements in. Zr, S, Se are almost certainely non-magnetic, 
but
there are quite a few structures with magnetic moments on Cr, Cu, and 
of

course on Mn.

To make terminology more complicated, remember that AFM means fully
compensated magnetic moments. The net magnetization of a unit cell of
some AFM structure is zero. This does not happen by coincidence, it is
because by symmetry all moments have the same size, and there are as
many of them pointing in one direction as there are pointing in exactly
the opposite direction.

So, IF your compound REALLY is AFM by experiment (NO net 
magnetization),

you almost certainely have only one magnetic species in there (probably
Mn). With two magnetic species, say, Mn and Cu, you would wind up with
different size of the moment on Mn and Cu. I know of no case where 
exact

compensation into an AFM structure occures by accident in such a
situation. You always get something with net moment - and these are
called ferrimagnetic structures. And since we are at it: there are
canted and helical strucutures where the moments are not collinear (not
within the scope of Wien2k), there are spin density waves, ...

Scanning this thread my advice would be to study a (good) book on solid
state physics, with special attention payed to its chapter discussing
magnetic order. If it doesn't have such a chapter its not a good book -
at least not for you. Do NOT use wikipedia or this mailing list and its
archive as a substitute for such a reading. It will not work.


---
Dr. Martin Pieper
Karl-Franzens University
Institute of Physics
Universitätsplatz 5
A-8010 Graz
Austria
Tel.: +43-(0)316-380-8564


Am 2020-01-27 17:45, schrieb djamel slamnia:

WHEN I STUDY A COMPOUND CONTAINS THIS ELEMENETS CU  MN   CR  ZR S SE

BETWEEN THEM WITCH ONE TO PUT IT SPIN UP OR DOWN AND NON-MAGNETIC ???

 Le lundi 27 janvier 2020 à 14:22:39 UTC+1, Gavin Abo
 a écrit :

 As previously mentioned [1], a short literature survey showed that
AFM type II and III are terms used for _fcc_ and _bcc_ lattices.
Since spacegroup 156 is not one of those, it might be inappropriate to
use those terms for spacegroup 156 having a _primitive_ lattice [2] of
the hexagonal crystal family.  If you do a more extensive literature
survey yourself and find a paper (article, book, etc.) that defines
the AFM magnetic orders for spacegroup 156, then reference and use it
for what the AFM order is.  If there is not any notations and terms
for AFM magnetic orders for spacegroup 156, you might have to make
your own figure or write in your own words what the definition is
should any AFM magnetic orders exist for it.
Keep in mind that as mentioned before in the mailing list archive, the
initial configuration can be set in case.inst with "instgen_lapw -ask"
[3].

Though, you need to check the final magnetic order that comes out of
the scf [4], because the configuration set in case.inst with
instgen_lapw is just the initial one that could change [5,6].

It is also possible to try to force a magnetic order using dmatup/dn
matrices but the final magnetic order is still what comes out of the
scf and could be different [6-11].

Therefore, it likely not beneficial to name the AFM order before
starting a calculation such that you would li

Re: [Wien] AFM type II

2020-01-29 Thread pieper
No one can give you an honest answer without knowing the structure you 
put these elements in. Zr, S, Se are almost certainely non-magnetic, but 
there are quite a few structures with magnetic moments on Cr, Cu, and of 
course on Mn.


To make terminology more complicated, remember that AFM means fully 
compensated magnetic moments. The net magnetization of a unit cell of 
some AFM structure is zero. This does not happen by coincidence, it is 
because by symmetry all moments have the same size, and there are as 
many of them pointing in one direction as there are pointing in exactly 
the opposite direction.


So, IF your compound REALLY is AFM by experiment (NO net magnetization), 
you almost certainely have only one magnetic species in there (probably 
Mn). With two magnetic species, say, Mn and Cu, you would wind up with 
different size of the moment on Mn and Cu. I know of no case where exact 
compensation into an AFM structure occures by accident in such a 
situation. You always get something with net moment - and these are 
called ferrimagnetic structures. And since we are at it: there are 
canted and helical strucutures where the moments are not collinear (not 
within the scope of Wien2k), there are spin density waves, ...


Scanning this thread my advice would be to study a (good) book on solid 
state physics, with special attention payed to its chapter discussing 
magnetic order. If it doesn't have such a chapter its not a good book - 
at least not for you. Do NOT use wikipedia or this mailing list and its 
archive as a substitute for such a reading. It will not work.



---
Dr. Martin Pieper
Karl-Franzens University
Institute of Physics
Universitätsplatz 5
A-8010 Graz
Austria
Tel.: +43-(0)316-380-8564


Am 2020-01-27 17:45, schrieb djamel slamnia:

WHEN I STUDY A COMPOUND CONTAINS THIS ELEMENETS CU  MN   CR  ZR S SE

BETWEEN THEM WITCH ONE TO PUT IT SPIN UP OR DOWN AND NON-MAGNETIC ???

 Le lundi 27 janvier 2020 à 14:22:39 UTC+1, Gavin Abo
 a écrit :

 As previously mentioned [1], a short literature survey showed that
AFM type II and III are terms used for _fcc_ and _bcc_ lattices.
Since spacegroup 156 is not one of those, it might be inappropriate to
use those terms for spacegroup 156 having a _primitive_ lattice [2] of
the hexagonal crystal family.  If you do a more extensive literature
survey yourself and find a paper (article, book, etc.) that defines
the AFM magnetic orders for spacegroup 156, then reference and use it
for what the AFM order is.  If there is not any notations and terms
for AFM magnetic orders for spacegroup 156, you might have to make
your own figure or write in your own words what the definition is
should any AFM magnetic orders exist for it.
Keep in mind that as mentioned before in the mailing list archive, the
initial configuration can be set in case.inst with "instgen_lapw -ask"
[3].

Though, you need to check the final magnetic order that comes out of
the scf [4], because the configuration set in case.inst with
instgen_lapw is just the initial one that could change [5,6].

It is also possible to try to force a magnetic order using dmatup/dn
matrices but the final magnetic order is still what comes out of the
scf and could be different [6-11].

Therefore, it likely not beneficial to name the AFM order before
starting a calculation such that you would likely want to identify the
name of the magnetic order after having finished the converged
calculation.
 [1]
https://www.mail-archive.com/wien@zeus.theochem.tuwien.ac.at/msg19515.html
[2] https://en.wikipedia.org/wiki/Crystal_structure#Lattice_systems
[3]
https://www.mail-archive.com/wien@zeus.theochem.tuwien.ac.at/msg10044.html

[4]
https://www.mail-archive.com/wien@zeus.theochem.tuwien.ac.at/msg17516.html
[5]
https://www.mail-archive.com/wien@zeus.theochem.tuwien.ac.at/msg03243.html
[6]
https://www.mail-archive.com/wien@zeus.theochem.tuwien.ac.at/msg06739.html
[7]
https://www.mail-archive.com/wien@zeus.theochem.tuwien.ac.at/msg14259.html
[8]
https://www.mail-archive.com/wien@zeus.theochem.tuwien.ac.at/msg05054.html
[9]
https://www.mail-archive.com/wien@zeus.theochem.tuwien.ac.at/msg13124.html
[10]
https://www.mail-archive.com/wien@zeus.theochem.tuwien.ac.at/msg16281.html
[11]
https://www.mail-archive.com/wien@zeus.theochem.tuwien.ac.at/msg16286.html

On 1/26/2020 1:01 PM, djamel slamnia wrote:





THANKS AGAIN SIR

I NEED TO KNOW WHAT IS THE AFM ORDERS FOR P3M1 (156) ??? TYPE II OR
III

THANKS IN ADVANCE





___
Wien mailing list
Wien@zeus.theochem.tuwien.ac.at
http://zeus.theochem.tuwien.ac.at/mailman/listinfo/wien
SEARCH the MAILING-LIST at:
http://www.mail-archive.com/wien@zeus.theochem.tuwien.ac.at/index.html
___
Wien mailing list
Wien@zeus.theochem.tuwien.ac.at
http://zeus.theochem.tuwien.ac.at/mailman/listinfo/wien
SEARCH the MAILING-LIST at:
http://www.mail-archive.com/wien@zeus.theochem.tuwien.ac.at/

Re: [Wien] AFM type II

2020-01-27 Thread pieper


I guess you are talking about the conventional classification of AFM 
ordering used in neutron diffraction. Consider AFM as a standing wave of 
spin orientations. Different Types of AFM are then distinguished by the 
wave vector of the standing wave in the crystal lattice.


AFM-I is the AFM order with the shortest possible wavelength, that is 
moments on nearest neighbor planes are antiparallel. In an fcc structure 
the nearest neighbors of an atom at (0,0,0) are on the two planes 
'above' and 'below' along the space diagonal of the cube at positions 
(1/2,1/2,0) and so on. The wave vector of the standing wave describing 
spin orientation is perpendicular to these planes of parallel spins. The 
length of the wave vector q in reciprocal space is such that going the 
distance d to the next plane with parallel moments (half the space 
diagonal) in that direction result in an identical situation in the wave 
function cos(qx), that is q*d=2*pi.


For AFM-II moments on planes with next nearest neighbors are 
antiparallel. And so on. The longer the wavelength of the standing wave 
(or the shorter q in reciprocal space) the more unit cells in the 
crystal lattice you will need to represent the AFM structure (depending 
on the distance between lattice planes in your structure without AFM 
order).


Good luck with figuring out the directions and lengths of wave vectors 
in your structure yourself,


Martin Pieper


---
Dr. Martin Pieper
Karl-Franzens University
Institute of Physics
Universitätsplatz 5
A-8010 Graz
Austria
Tel.: +43-(0)316-380-8564


Am 2020-01-26 21:01, schrieb djamel slamnia:

THANKS AGAIN SIR

I NEED TO KNOW WHAT IS THE AFM ORDERS FOR P3M1 (156) ??? TYPE II OR
III

THANKS IN ADVANCE

 Le dimanche 26 janvier 2020 à 20:51:41 UTC+1, Gavin Abo
 a écrit :


THE DEFINITION FOR THE COMPOUND   A=B = 3.74 A   ALPHA = BETA = 90
GAMMA = 120
for AFM type I : i creat  superstructure  x super cell target
lattice H  :  x =1,   y = 1 ,  Z =2  then x sgroup, program define
automatically the space group the same of my original space group
156 without warrning


As you have described above (for Z=2), your attempt at creating a
supercell has failed as "x sgroup" collapsed the supercell structure
back to the non-supercell structure.

As mentioned on the FAQ page for supercell construction, you need to
displace an atom, change an atom, or use a special label:

http://susi.theochem.tuwien.ac.at/reg_user/faq/supercells.html

In order to keep the supercell without "x sgroup" reducing it back to
the original structure, refer to previous posts in the mailing list
archive about breaking the symmetry.  A few of the many posts about
that as examples are at the three links below:
 
https://www.mail-archive.com/wien@zeus.theochem.tuwien.ac.at/msg18380.html

https://www.mail-archive.com/wien@zeus.theochem.tuwien.ac.at/msg01866.html
https://www.mail-archive.com/wien@zeus.theochem.tuwien.ac.at/msg15517.html



but when try to do it for type II : x super cell  P
:  x =1,   y = 1 ,  Z =1 x sgroup could not define the space group


In section "3.12 Setting up a new case" on page 29 in the WIEN2k 19.1
usersguide [
http://susi.theochem.tuwien.ac.at/reg_user/textbooks/usersguide.pdf ],
there is the statement:

"Alternatively with the new StructGen you can specify the spacegroup
and only the inequivalent positions. The equivalent ones will be
generated automatically."

This means spacegroups in WIEN2k are defined according to the
inequivalent positions and not by the equivalent positions.

For the case above (Z=1), it is likely that "x supercell" found some
equivalent positions in the original structure and automatically added
special labels to them changing them into inequivalent positions.
Thus, a supercell structure was successfully created.  If you want the
supercell structure to reduce back to the original structure, you
would likely just need to remove all or some of the special labels in
StructGen before running "x sgroup".

For understanding the inequivalent and equivalent positions with
WIEN2k spacegroups, the example in the post at the following link
might helpful:

http://zeus.theochem.tuwien.ac.at/pipermail/wien/2013-January/018171.html

___
Wien mailing list
Wien@zeus.theochem.tuwien.ac.at
http://zeus.theochem.tuwien.ac.at/mailman/listinfo/wien
SEARCH the MAILING-LIST at:
http://www.mail-archive.com/wien@zeus.theochem.tuwien.ac.at/index.html
___
Wien mailing list
Wien@zeus.theochem.tuwien.ac.at
http://zeus.theochem.tuwien.ac.at/mailman/listinfo/wien
SEARCH the MAILING-LIST at:
http://www.mail-archive.com/wien@zeus.theochem.tuwien.ac.at/index.html

___
Wien mailing list
Wien@zeus.theochem.tuwien.ac.at
http://zeus.theochem.tuwien.ac.at/mailman/listinfo/wien
SEARCH the MAILING-LIST at:  
http://www.mail-archive.com/wien@zeus.theochem.tuwien.ac.at/index.html


Re: [Wien] density matrix is not positive

2020-01-11 Thread pieper
If I understand correctly, then the SCF cycle does not wade through some 
river (to stay in the pedagogic example). It walks along the shore, 
sometimes with one foot in the water (the density generated by mixer). 
The other foot (the density generated from the wave functions) always 
comes down on dry land. This way the SCF can step across cracks in 
reality that are narrower than the step width.


This also explains why one apparently can be rather careless about 
semi-positiveness of dmats when manipulating them to stabilize certain 
solutions with the -orb option, correct?


I can appreciate that this has advantages if there are a lot of rivers. 
I don't understand why this is always better than going for unitary 
transformations only, because it does add a lot of definitely wrong 
directions to explore. But I am happy to trust the experts.


So thanks a lot for the explanations,

Martin


---
Dr. Martin Pieper
Karl-Franzens University
Institute of Physics
Universitätsplatz 5
A-8010 Graz
Austria
Tel.: +43-(0)316-380-8564


Am 2020-01-08 16:10, schrieb Laurence Marks:

Yet more pedagogy...

Over the last few years I have been trying to add to the mixer
algorithms so it can sense when their might be piranha around. One of
these is large fluctuations of the potential; if these occur it
becomes more cautious. Another currently being tested for a future
release is taking a step backwards if something was not right. Slowly
the mixer is becoming smarter...I hope.

_
Professor Laurence Marks
"Research is to see what everybody else has seen, and to think what
nobody else has thought", Albert Szent-Gyorgi
www.numis.northwestern.edu [1]

On Wed, Jan 8, 2020, 08:40 Laurence Marks 
wrote:


For pedagogical reasons, let me explain why forcing dmat to positive
definite may not be good.

The mixer is solving a fixed point problem, which with +U is for
both the dmat and density. Currently it can explore all values,
including ones which are not physically reasonable. If we limit it
to positive definite then there are some forbidden regions. The
problem may be slower if the mixer can't explore them.

As an illustration, suppose your home is on the other side of a
shallow stream. If you demand that your feet don't get wet, you may
have to walk a long way to a bridge to get home. If you are OK with
getting your feet wet, you just ford the river. Fine, so long as
there are no pirhana fish (ghost bands) in the stream.

_
Professor Laurence Marks
"Research is to see what everybody else has seen, and to think what
nobody else has thought", Albert Szent-Gyorgi
www.numis.northwestern.edu [1]

On Wed, Jan 8, 2020, 07:01 Laurence Marks 
wrote:

Being more specific, the unmixed density matrix (dmat) produced by
one scf iteration is necessarily positive definite. The mixer
produces a new dmat that is a linear combination of prior dmats
used, i.e.

dmat_(n+1) = sum_i c_i * dmat_i

If some of the coefficients c_i are negative the new density matrix
may not be positive definite. A trivial example used two 1x1
matrices M=1 and M=2.

As Peter said, at convergence (:MV) the dmat must be positive
definite since the input is equal to the output.

N.B., one could constrain the dmat to be positive definite. However,
mathematically this may converge slower, not faster.

_
Professor Laurence Marks
"Research is to see what everybody else has seen, and to think what
nobody else has thought", Albert Szent-Gyorgi
www.numis.northwestern.edu [1]

On Wed, Jan 8, 2020, 06:40 Peter Blaha
 wrote:
The "new" (unmixed) dmat is always ok, only through mixing of the
individual elements this may happen.

When reaching scf, the "mixed" dmat = "unmixed" dmat and thus this
should no longer occur.

On 1/8/20 1:19 PM, pieper wrote:

Sorry for interrupting, but I am intrigued. I was not aware that

the

density matrix can leave the physically meaningfull realm of
semi-positive operators.

May I spoil this thread with the question why the mixer produces

such

non-physical trial states? And if it does, what forces drive rho

back to

proper density operators during convergence?

---
Dr. Martin Pieper
Karl-Franzens University
Institute of Physics
Universitätsplatz 5
A-8010 Graz
Austria
Tel.: +43-(0)316-380-8564


Am 2020-01-04 11:50, schrieb Peter Blaha:

I repeated your calculations. Yes, intermediately I also got

these

density matrix warnings, but NOT with final convergence.

Most likely, this warning should be removed from mixer (Laurence

!),

as it does not give any meaningful hint or it should be printed

only

if one is nearly converged (:DIS is lower than ...)

I have a few other comments:

You seem to start a Ferro-magnetic calculation. As was mentioned
before, maybe an AFM structure is more stable. So check other

possible

magnetic configurations.

You seem to start with init_lapw, and thus with the default
(ferromagnetic) case.inst

For magnetic oxides it is highly recommended to s

Re: [Wien] density matrix is not positive

2020-01-08 Thread pieper
Sorry for interrupting, but I am intrigued. I was not aware that the 
density matrix can leave the physically meaningfull realm of 
semi-positive operators.


May I spoil this thread with the question why the mixer produces such 
non-physical trial states? And if it does, what forces drive rho back to 
proper density operators during convergence?


---
Dr. Martin Pieper
Karl-Franzens University
Institute of Physics
Universitätsplatz 5
A-8010 Graz
Austria
Tel.: +43-(0)316-380-8564


Am 2020-01-04 11:50, schrieb Peter Blaha:

I repeated your calculations. Yes, intermediately I also got these
density matrix warnings, but NOT with final convergence.

Most likely, this warning should be removed from mixer (Laurence !),
as it does not give any meaningful hint or it should be printed only
if one is nearly converged (:DIS is lower than ...)

I have a few other comments:

You seem to start a Ferro-magnetic calculation. As was mentioned
before, maybe an AFM structure is more stable. So check other possible
magnetic configurations.

You seem to start with init_lapw, and thus with the default
(ferromagnetic) case.inst

For magnetic oxides it is highly recommended to start with:

instgen -ask
This lets you define a possible AFM arrangement and in particular you
should set the O-atoms as non-magnetic. This save a lot of scf cycles.

-rkmax 7.5 is quite some "overkill". This rkmax value is for the small
O-atoms and RKmax=6 (beginning) and 7 (final convergence) is
sufficient.

On the other hand, a HDLO for Nd-4f states is highly recommended.

I never start the scf-cycle with "-orb" right after intitialization,
but always do (at least a crude) GGA calculation before switching on
-orb. Otherwise you may even more likely end up in some local minimum
but not the ground state.

I got a half-metallic solution with a spin-up 4f peak down at -3.5 eV
and another one at EF. You expected that the 4f states are all removed
from EF, but your compound seems to be "special" (is this compound
experimentally known ??).

The stoichiometry of Nd4 Ni3 O8  suggests Ni2+ ions, leaving a (Nd4)10+
charge state, i.e. a Nd2.5+ state, which is very unusual. But I got
34f electrons, which hints to a Nd3+ and in fact one of your Oxygens
has p-states above EF.

With this half-filled 4f-DOS peak at EF also spin-orbit could be very 
important.


And as always in such LDA+U calculations, play with the initial
case.dmat file. In my case I have 2 4f orbitals "fully" occupied (and
thus shifted down), but 2 others are partly occupied (close to 1/2)
and thus not much shifted and pinned to EF.

As mentioned before, additional symmetry splitting with AFM for the 2
equivalent Nd atoms may lead to further splitting.


Am 03.01.2020 um 10:30 schrieb Fan:
Thank you for your reply, Prof. Blaha. In fact, when I change 
parameters in inorb, I always do 'rm case.dmat*' and 'rm case.vorb*' , 
so I think it should not be the problem. But I do value your 
suggestions, so I started a new session, only used the struct file of 
Nd4Ni3O8 which is attached here, and then

init_lapw -b -rkmax 7.5 -numk 2000 -sp -vxc 5 -lvns 6
create inorb and indm by copying them from templates and modify them 
accordingly

runsp_lapw -p -orb -i 200 -ec 0.0001 -cc 0.001 -NI
during the scf cycle, QTL-B warning appeared, but the value was very 
small (2~3, still could be a trouble?), the density matrix warnings 
appeared at about the 20th cycle and persisted.


With regards.

Fan



___
Wien mailing list
Wien@zeus.theochem.tuwien.ac.at
http://zeus.theochem.tuwien.ac.at/mailman/listinfo/wien
SEARCH the MAILING-LIST at:  
http://www.mail-archive.com/wien@zeus.theochem.tuwien.ac.at/index.html



___
Wien mailing list
Wien@zeus.theochem.tuwien.ac.at
http://zeus.theochem.tuwien.ac.at/mailman/listinfo/wien
SEARCH the MAILING-LIST at:  
http://www.mail-archive.com/wien@zeus.theochem.tuwien.ac.at/index.html


Re: [Wien] x symmetso error

2019-12-17 Thread pieper


Hi,

in your struct file there is no indication that you did initso. It has 
been said again and again and it is in the UG: SOC may change  the 
symmetry operations applicable for a given crystal structure, depending 
on the direction of the quantization axis (magnetization). initso will 
detect this and present you with a new struct file. Accept this file! 
Without actually trying I suspect that it will do so (lower symmetry) in 
your structure if you point it along 100, while all symmetry operations 
will be preserved if you point it along 001.


And be carefull to use the same structure file (with lowest symmetry) 
for ALL calculations where you want compare total energies (e.g. 
magnetic anisotropy)


Best regards


---
Dr. Martin Pieper
Karl-Franzens University
Institute of Physics
Universitätsplatz 5
A-8010 Graz
Austria
Tel.: +43-(0)316-380-8564


Am 2019-12-16 10:48, schrieb Riyajul Islam:

Dear Wien2k users,
I'm using wien2k 19.1 version. I'm trying to include SOC in my
calculation after GGA+U was done. I did GGA+U+SOC calculations along
001 magnetization direction without any errors. But while I chose 100
direction, I get an error during x symmetso

forrtl: severe (174): SIGSEGV, segmentation fault occurred
Image  PCRoutineLine
Source
symmetso   004514E3  Unknown   Unknown
Unknown
libpthread-2.23.s  2AD9F2D14390  Unknown   Unknown
Unknown
libiomp5.so2AD9F355C871  Unknown   Unknown
Unknown
libiomp5.so2AD9F355C5E4  Unknown   Unknown
Unknown
libiomp5.so2AD9F355E4C5  Unknown   Unknown
Unknown
libiomp5.so2AD9F355F89F  Unknown   Unknown
Unknown
libiomp5.so2AD9F355411B  Unknown   Unknown
Unknown
libiomp5.so2AD9F3554007  Unknown   Unknown
Unknown
libiomp5.so2AD9F3553D52  Unknown   Unknown
Unknown
libiomp5.so2AD9F355454D  Unknown   Unknown
Unknown
libiomp5.so2AD9F34DDD61  Unknown   Unknown
Unknown
ld-2.23.so [1] 2AD9F2AEBE15  Unknown   Unknown
 Unknown
libc-2.23.so [2]   2AD9F3850FF8  Unknown   Unknown
 Unknown
libc-2.23.so [2]   2AD9F3851045  Unknown   Unknown
 Unknown
symmetso   0046D1E4  Unknown   Unknown
Unknown
symmetso   0040C191  MAIN__359
symmetso.f
symmetso   004044F2  Unknown   Unknown
Unknown
libc-2.23.so [2]   2AD9F3837830  __libc_start_main Unknown
 Unknown
symmetso   004043E9  Unknown   Unknown
Unknown
0.6u 0.0s 0:00.71 100.0% 0+0k 0+42368io 0pf+0w
error: command   /home/edison/Wien2k19.1/symmetso upsymmetso.def
failed

Here I'm also attaching the structure file. Kindly help me solve this
problem.

Kind regards

--

Riyajul Islam
Ph.D Scholar
National Institute of Technology Nagaland

Chumukedima, Dimapur
Nagaland 797103, India



Links:
--
[1] http://ld-2.23.so
[2] http://libc-2.23.so
___
Wien mailing list
Wien@zeus.theochem.tuwien.ac.at
http://zeus.theochem.tuwien.ac.at/mailman/listinfo/wien
SEARCH the MAILING-LIST at:
http://www.mail-archive.com/wien@zeus.theochem.tuwien.ac.at/index.html

___
Wien mailing list
Wien@zeus.theochem.tuwien.ac.at
http://zeus.theochem.tuwien.ac.at/mailman/listinfo/wien
SEARCH the MAILING-LIST at:  
http://www.mail-archive.com/wien@zeus.theochem.tuwien.ac.at/index.html


Re: [Wien] In which approach the magnetism is treated in the Wien2k Code ?

2019-12-13 Thread pieper

Hi,

please read the user guide (and, probably, a good book on solid state 
physics). Wien2k is an all electron code. There is a switch to take into 
account the spin degree of freedom for the electrons (spin polarized 
calculations). In such a calculation the spin of each electron is taken 
into account, regardless of wether its itinerant or localized.


Best regards


---
Dr. Martin Pieper
Karl-Franzens University
Institute of Physics
Universitätsplatz 5
A-8010 Graz
Austria
Tel.: +43-(0)316-380-8564


Am 2019-12-10 21:46, schrieb Abderrahmane Reggad:

Helli wien users

I want to know in which approach the magnetism is treated in wien2k
code?
We know that the Fe metal in bcc structure is ferromagnetic and is
considered as having a dual nature itinerant and localized.
How is teated this magnetic state in wien2k code?

Best regards

--

Dr. Abderrahmane Reggad
Engineering Physics LaboratoryFaculty of Material Sciences, Ibn
Khaldoun University, Tiaret, 14000, AlgeriaTel: +213(0)561861963 -
Algeria
___
Wien mailing list
Wien@zeus.theochem.tuwien.ac.at
http://zeus.theochem.tuwien.ac.at/mailman/listinfo/wien
SEARCH the MAILING-LIST at:
http://www.mail-archive.com/wien@zeus.theochem.tuwien.ac.at/index.html

___
Wien mailing list
Wien@zeus.theochem.tuwien.ac.at
http://zeus.theochem.tuwien.ac.at/mailman/listinfo/wien
SEARCH the MAILING-LIST at:  
http://www.mail-archive.com/wien@zeus.theochem.tuwien.ac.at/index.html


Re: [Wien] Spin-polarization and spin-orbit coupling

2019-09-25 Thread pieper

Reading your questions I get the impression that you miss a crucial
point of what and how Density Functional Theory (DFT) does. Therefore
I would like to extend on what Peter Blaha said. Maybe it helps to
clarify some of his answers.

Pauli's Hamiltonian represents the energy of an electron in some
electromagnetic field (your external magnetic field).

DFT aims to find the ground state of a system of many interacting
Fermions (electrons) represented by any Hamiltonian for the
interaction you care to throw at it. The mean interaction energy of a
given electron spin with all others in the system is the same as its
energy in some fictitious internal field.

DFT works via a subtle reinterpretation of the role of the charge
density, which has far reaching consequences. In 1964 Hohenberg and
Kohn proved a fascinating property of the charge density: This single,
real valued function of (without spin) one spacial variable uniquely
determines the many particle wave function of interacting electrons in
an external Coulomb potential (due to, in this case, nuclear
charges). Calculation of the spacial charge density from the wave
function by an expectation value  is not a one way
process. If two solutions of Schroedingers equation for the
interacting many particle system in a given external Coulomb potential
give the same charge density, then the two wave functions with all
their N space variables for N electrons are the same. One can, in
principle, go back: the charge density determines the wave
function - and with it everything, including the energy.

Furthermore, the ground state charge density can be determined by a
variational principle and its Euler-Lagrange equations (the Kohn-Sham
equations). They can be solved very effectively in a self consistent,
iterative process: start from some charge densitiy (the closer to the
solution the better to avoid local minima). Then calculate the
potentials - solve the Kohn-Sham equations - calculate the charge
density - compare it to previous ones. If the difference is larger
than some threshold, mix up a variation of it and calculate the
potentials ...

The most simple case is if your favorite interaction Hamiltonian has
Coulomb potentials from local charges and no spin
contribution. Spin-up and -down charge densities are identical (Local
Densitsity Approximation, LDA, for a certain type of electric 
potential).


If there are exchange interaction energies present (products of the
spin operators of pairs of interacting electrons) the densities of the
two spin directions become different (Local Spin Density
Approximation, LSDA, the electric potential stays the same). Such a
Heisenberg exchange interaction does, however, not depend on the
oriention of interacting spins in the crystal lattice. Only the
strength of the equivalent internal field has a meaning, not its
direction.

In contrast, a spin-orbin coupling in the Hamiltonian does depend on
the orientation of the spin moment in the lattice. With the
interaction energy depending on spin orientation, so does the
equivalent internal field. The system is magnetically anisotropic. You
have to specifiy the orientation of the moment to determine the energy
of the graound state.

There is, of course, a huge number of books an reviews on DFT. The UG
cites enough stuff to keep you busy for a long time. Personally, I
like a review of A. Becke: THE JOURNAL OF CHEMICAL PHYSICS 140, 18A301
(2014)


---
Dr. Martin Pieper
Karl-Franzens University
Institute of Physics
Universitätsplatz 5
A-8010 Graz
Austria
Tel.: +43-(0)316-380-8564


Am 2019-09-24 08:23, schrieb Luigi Maduro - TNW:

Dear WIEN2k users,

I have three questions concerning the inclusion of spin in a material
in WIEN2k.

The three questions concern the two terms where a spin-dependent term
appears in the Pauli Hamiltonian for magnetic systems, which are:

Question 1)
In the Pauli Hamiltonian a term appears which is a dot product of the
spin-matrices of the system and an effective magnetic field.

The effective magnetic field is a summation of an external magnetic
field and an exchange-correlation term. The exchange-correlation term
B_xc, is expressed as a derivative of the density w.r.t. the
magnetization (in the LDA framework) and that B_xc is parallel to the
magnetization density vector. If I understand correctly then the
material of interest is magnetic when B_xc is nonzero.

When doing a spin-polarized calculation, what happens then to the
external magnetic field term? Is the external magnetic field term set
to zero?

Question 2)
The other term in the Pauli Hamiltonian is the spin-orbit coupling
(SOC) term, which is proportional to (1/r x dV/dr ) (dV/dr = the
derivative of the potential w.r.t. the radial coordinate).

When doing a calculation including SOC the script init_so asks for the
magnetization direction (in hkl).

In a non-spin polarized calculation with SOC the magnetization
direction has no meaning, is this correct?

Question 3)

If the system of interest

Re: [Wien] Magnetic moments converging in a different direction to the one they are defined

2019-04-16 Thread pieper
Fe3O4 being an old but unsatisfied love of mine a few additional 
comments:


Determining exchange constants by spin reversal only makes sense if the 
changes in electronic structure are small (see e.g. P. Novak et. al 
PHYSICAL REVIEW B 71, 184433, 2005).


This (usually) works best in insulators, it is a delicate problem in 
metals, and much more so in Fe3O4 with its Vervey transition. Here a 
very intricate coupling between electronic and structural degrees of 
freedom is at work. I seem to recall that the low temperature phase is a 
comlicated mess (see e.g. Novak et al, PRB 61, 1256, 2000 and references 
therein). So complicated that, as far as I remember, in the early 
2000nds S.Cottenier, R. Laskowski, J. Rusz, M. Rots and P. Novak gave a 
talk on a Wien2k Workshop calculating exchange interactions in magnetite 
using the non-collinear magnetism version  NCM-Wien2k. Unfortunately I 
don't have time to search for literature on that one, but you probably 
don't want to get into NCM anyway.


However, I don't think you can avoid DFT+U or +EECE - at least not for 
Fe3O4. I am sure you can find a lot of literature on DFT+U and +EECE of 
Fe3O4, among others by Novak, Madsen, ...  This may introduce an 
additional parameter in your comparisons of your structures. If you are 
using an older version of Wien2k, upgrade! Wien26_16 had a bug with 
DFT+U (see 
https://www.mail-archive.com/wien@zeus.theochem.tuwien.ac.at/msg15590.html).


Best regards,

Martin Pieper


---
Dr. Martin Pieper
Karl-Franzens University
Institute of Physics
Universitätsplatz 5
A-8010 Graz
Austria
Tel.: +43-(0)316-380-8564


Am 2019-04-16 10:29, schrieb Penny, Charles:

Dear all,

 I am running spin-polarised calculations on a range of iron-spinel
structures (namely, magnetite (Fe3O4), maghemite (gamma-Fe2O3) and
greigite (Fe3S4)) with the objective of calculating magnetic exchange
energies in these minerals. This requires calculating total energies
of lot of different spin configurations. This process has worked well
for magnetite and maghemite, but I have encountered a problem with
greigite.

 When I run a calculation on a spin configuration of greigite that
isn’t the ferrimagnetic ground state (e.g. a ferromagnetic
configuration) the calculation converges to the ferrimagnetic
solution, with the sublattice moments pointing in opposing directions.


 In the examples below, I have used a low-symmetry unit cell with
eight unique iron atoms which allows me to calculate the required
number of spin configurations for estimating J_ij. Atoms 1-4
correspond to A site iron atoms in the spinel structure, atoms 5-8
correspond to B site iron atoms and atoms 9-16 are sulphur atoms. In a
ferrimagnetic system the A and B sites have opposing moments and
sulphur atoms are non-magneitc.

 When I define a ferrimagnetic spin configuration, the calculation
proceeds as expected, with the final moments looking like;

 rkmax_8_k_500.scf::MMINT:  MAGNETIC MOMENT IN INTERSTITIAL  =
-0.05116

 rkmax_8_k_500.scf::MMI001: MAGNETIC MOMENT IN SPHERE   1=
2.47349

 rkmax_8_k_500.scf::MMI002: MAGNETIC MOMENT IN SPHERE   2=
2.47348

 rkmax_8_k_500.scf::MMI003: MAGNETIC MOMENT IN SPHERE   3=
2.47348

 rkmax_8_k_500.scf::MMI004: MAGNETIC MOMENT IN SPHERE   4=
2.47348

 rkmax_8_k_500.scf::MMI005: MAGNETIC MOMENT IN SPHERE   5=
-3.01699

 rkmax_8_k_500.scf::MMI006: MAGNETIC MOMENT IN SPHERE   6=
-3.01699

 rkmax_8_k_500.scf::MMI007: MAGNETIC MOMENT IN SPHERE   7=
-3.01699

 rkmax_8_k_500.scf::MMI008: MAGNETIC MOMENT IN SPHERE   8=
-3.01699

 rkmax_8_k_500.scf::MMI009: MAGNETIC MOMENT IN SPHERE   9=
-0.03675

 rkmax_8_k_500.scf::MMI010: MAGNETIC MOMENT IN SPHERE  10=
-0.03675

 rkmax_8_k_500.scf::MMI011: MAGNETIC MOMENT IN SPHERE  11=
-0.03675

 rkmax_8_k_500.scf::MMI012: MAGNETIC MOMENT IN SPHERE  12=
-0.03675

 rkmax_8_k_500.scf::MMI013: MAGNETIC MOMENT IN SPHERE  13=
-0.03675

 rkmax_8_k_500.scf::MMI014: MAGNETIC MOMENT IN SPHERE  14=
-0.03675

 rkmax_8_k_500.scf::MMI015: MAGNETIC MOMENT IN SPHERE  15=
-0.03675

 rkmax_8_k_500.scf::MMI016: MAGNETIC MOMENT IN SPHERE  16=
-0.03675

 rkmax_8_k_500.scf::MMTOT:  SPIN MAGNETIC MOMENT IN CELL =
-14.88108

 Final energy;

 rkmax_8_k_500.scf::ENE  : ** TOTAL ENERGY IN Ry =
-43322.30312592

 However, when I define a ferromagnetic spin configuration the system
converges to a ferrimagnetic solution with final moments;

 k_500_rkmax_8.scf::MMINT:  MAGNETIC MOMENT IN INTERSTITIAL  =
0.05118

 k_500_rkmax_8.scf::MMI001: MAGNETIC MOMENT IN SPHERE   1=
-2.47348

 k_500_rkmax_8.scf::MMI002: MAGNETIC MOMENT IN SPHERE   2=
-2.47347

 k_500_rkmax_8.scf::MMI003: MAGNETIC MOMENT IN SPHERE   3=
-2.47346

 k_500_rkmax_8.scf::MMI004: MAGNETIC MOMENT IN SPHERE   4=
-2.47346

 k_500_rkmax_8.scf::MMI005: MAGNETIC MOMENT IN SPHERE   5=
3.01697

 k_500_rkmax_8.scf::MMI006: MAGNETIC MOMENT IN SPHERE   6=
3.01697

 k_500_rkmax_8.scf::MMI007: MAGNETIC

Re: [Wien] Augmented Plane Wave

2019-03-22 Thread pieper

Dear Pablo,

I suspect your problem occurs because you left out the word which 
"Augmented" refers too: It is an "Augmented Plane Wave Method",
that is, the Method is augmented (including additional basis functions), 
not the plane waves (in amplitude or intensity).


Best regards,

Martin Pieper

---
Dr. Martin Pieper
Karl-Franzens University
Institute of Physics
Universitätsplatz 5
A-8010 Graz
Austria
Tel.: +43-(0)316-380-8564


Am 2019-03-22 02:49, schrieb delamora:

Dear Wien users,
 I have a question about the name of

 "Augmented Plane Wave"
 I had the idea that when the wave enters the Muffin Tin sphere the
amplitude of the wave increased.
 Trying to see this I found that when a wave crosses a step function,
 https://quantummechanics.ucsd.edu/ph130a/130_notes/node149.html

 When the incoming wave

 exp(ikx)

 reaches an upwards step function there is a reflected wave
 R exp(-ikx)

 and a transmitted wave
 T exp(ik'x)

 what this article shows is;
 1 + R = T

 That is, the amplitudes of the incoming wave and the reflected wave
add to the amplitude of the transmitted wave

 If I take this into a square well then I would understand that the
waves inside the well have the total amplitude equal to the incoming
and transmitted wave. That is, when the wave enters the Muffin Tin the
amplitude of wave is not AUGMENTED. So why is this method called
"Augmented Plane Wave"?

 Saludos

 Pablo
___
Wien mailing list
Wien@zeus.theochem.tuwien.ac.at
http://zeus.theochem.tuwien.ac.at/mailman/listinfo/wien
SEARCH the MAILING-LIST at:
http://www.mail-archive.com/wien@zeus.theochem.tuwien.ac.at/index.html

___
Wien mailing list
Wien@zeus.theochem.tuwien.ac.at
http://zeus.theochem.tuwien.ac.at/mailman/listinfo/wien
SEARCH the MAILING-LIST at:  
http://www.mail-archive.com/wien@zeus.theochem.tuwien.ac.at/index.html


Re: [Wien] Problem with DOS

2019-01-16 Thread pieper
Assuming that by 'very fine band' you mean a very narrow band my first 
guess would be that it is missing in your DOS because it sits between 
two energies on the energy axis of the plot. Focus the energy range 
where your DOS is calculated on the interval where the band actually is.


Good luck,

Martin Pieper


---
Dr. Martin Pieper
Karl-Franzens University
Institute of Physics
Universitätsplatz 5
A-8010 Graz
Austria
Tel.: +43-(0)316-380-8564


Am 14.01.2019 23:49, schrieb Wien2k User:

Dear wien2k users:

The band structure has given a very fine band but when I plot the
corresponding density of state I can not find the peak corresponding
to this band (or the value of   this peak  is less than the format of
output file of dos (F?.8))

how can I get the peak value even if it is less than E-8
___
Wien mailing list
Wien@zeus.theochem.tuwien.ac.at
http://zeus.theochem.tuwien.ac.at/mailman/listinfo/wien
SEARCH the MAILING-LIST at:
http://www.mail-archive.com/wien@zeus.theochem.tuwien.ac.at/index.html

___
Wien mailing list
Wien@zeus.theochem.tuwien.ac.at
http://zeus.theochem.tuwien.ac.at/mailman/listinfo/wien
SEARCH the MAILING-LIST at:  
http://www.mail-archive.com/wien@zeus.theochem.tuwien.ac.at/index.html


Re: [Wien] somethint about the symmetry in the struct file

2019-01-16 Thread pieper

In addition to Gavin's comments:

I believe you want to do a permutation of the list of symmetry 
operations placed by symmetry at the end of case.struct. You want to do 
this in file case.struct_st, NOT in case.struct?


To my best knowledge you can change whatever you want in file 
case.struct_st. It is a book-keeping output of program symmetry. Its 
single use is at the end of symmetry to (optionally) replace an existing 
file case.struct. After that is done you almost certainely may tidy it 
up as you whish, or place comments in there, or even delete it. As Gavin 
said, try it.


Just DO NOT replace case.struct with the edited file - or do that only 
if you know exactly what you are doing (which you apparently do not 
know).


What do you mean by 'the first matrix is not unitary'? The one operation 
that is always present in the list is the identity, or unity. You most 
probably mean that one, and you want to have it at the top of the list?


May I ask WHY you want change the sequence of symmetry operations?

Good luck,

Martin Pieper


---
Dr. Martin Pieper
Karl-Franzens University
Institute of Physics
Universitätsplatz 5
A-8010 Graz
Austria
Tel.: +43-(0)316-380-8564


Am 16.01.2019 04:11, schrieb 姜若诗:

Hello,I want to ask something about the symmetry matrix in the
case.struct.

If I change the order of the symmetry matrix in the case.struct.  For
example, the first matrix is not unitary,I change the eighth matrix
and the first matrix in the case.struct_st in ordet to make sure the
first one is unitary,then I run it.Will it have some influence to my
result?

 Wish your reply.


Sincerely


Jasmine
___
Wien mailing list
Wien@zeus.theochem.tuwien.ac.at
http://zeus.theochem.tuwien.ac.at/mailman/listinfo/wien
SEARCH the MAILING-LIST at:
http://www.mail-archive.com/wien@zeus.theochem.tuwien.ac.at/index.html

___
Wien mailing list
Wien@zeus.theochem.tuwien.ac.at
http://zeus.theochem.tuwien.ac.at/mailman/listinfo/wien
SEARCH the MAILING-LIST at:  
http://www.mail-archive.com/wien@zeus.theochem.tuwien.ac.at/index.html


Re: [Wien] EFG: theory Vs experiment for a case

2018-12-19 Thread pieper
As an addendum to previous comments by the experts I strongly recommend 
to always ask (and answer) yourself the question:
what symmetries does the experimental result imply, and what symmetries 
are present in my model?


You state that samples prepared in a precedure 'where synthetic 
parameters can be controlled' (whatever that means) 'shows STO with 
probe showing very pure HFI'. I take it that TDPACS finds Hf in a unique 
environment. All of the spectra is accounted for by a single value 
abs(V_zz)=1.69e21 v/m^2, eta=0.52? No (sizeable) broadening, no other 
lines from different EFG's?


In that case:

1) Whatever model you set up in case.struct, your Hf-site MUST NOT have 
a threefold (or higher) rotation symmetry in its point symmetry group 
(check outputs of structure initialzation programs).


2) Your structure (with your defect of the day) should have a 
significantly lower total energy than other possibilities - otherwise a 
superposition of the corresponding spectra is expected.


Up to now your calculations without neighboring oxygen defects seem to 
be useless because you did not take into account the first point. With 
respect to the second point I am very dubious about your calculations 
assuming oxygen defects in some neighborhood. Some Hf-vacancy pair may 
have a calculated EFG resembling the experimental one. But in an 
unrelaxed structure this is meaningless. And even if you insist to take 
something like that serious, you will have to come up with a really good 
reason why that special defect structure is the only one present in your 
sample!


Looking at your result that the artificial V_zz from your unrelaxed 
3x2x2 supercell already has the same size as the experimental one I 
strongly suspect that you will get a pretty good simulation of the 
experimental result by some very small rearrangement of atoms in the 
nearest, at most the next neighbor shell - WITHOUT oxygen vacany. Put Hf 
on a Ti-site and really DO a relaxation. Start it from a structure with 
appropriate (low) point symmetry at Hf. (shift some neighboring atom(s) 
a tiny bit out of symmetry in some plausible direction).


And remember, EFG is short range - from my experience even a 2x2x2 
supercell may give you a good idea what happens.



---
Dr. Martin Pieper
Karl-Franzens University
Institute of Physics
Universitätsplatz 5
A-8010 Graz
Austria
Tel.: +43-(0)316-380-8564


Am 19.12.2018 09:48, schrieb Ashwani Kumar:

System under study is SrTiO3 doped with
Hf-181 tdpac probe for TDPAC spectroscopy by thermal equillibriation
method (1250 C for >12 hours). the doping is substitutional as Hf(IV)
and Ti(IV) has nearby ionic radius and same oxidation state. Bulk STO
shows no efg, no assymetry parameter as STO is cubic lattice and Hf/Ti
present at octahedral symmetry but in another synthetic procedure
where synthesis parameters can be controlled shows STO with probe
showing very pure HFI. From literature, STO is known to have oxygen
vacancies depending on synthesis route.
   Calculation: STO structure is optimized and
minimized. Created supercell 3x2x2 as it allows me to use 5 processor
with 8 gb ram (with 3x3x3, calc. restricted to only 2 processors due
to RAM limitation. So long scf run time. i will try now) . Problem of
breaking symmetry did not strike me at the time of calculation.
Replaced one Ti by Hf. Then supercell calculation was done for oxygen
vacancies in first cordination and fourth nearest neighbour
cordination. The efg matched for the later case (oxygen vacancy at
fourth nearest neighbour) but not assymetry parameter. Minimization of
forces on supercell was not done ( i thought basic lattice unit was
already optimized).

 What i understood from previous replies of this thread is
1.Must do 3x3x3 supercell calculation, 2. Electron electron
correlation can also be included for improvement of calculation, 3.
Need to check calculation with reduced RMT of Sr and Ti.

To what % agreemnet wrt experimental efg data, efg values from
theoretical calculation can be accepted.
Thanks Dr. Cottenier for the suggestion, i had already subscribed to
HF course A (ID: iak). I will subscribe to Course B after finishing
course A.

Thanks,

A. Kumar

On Wed, Dec 19, 2018 at 12:08 AM Ashwani Kumar 
wrote:


hi,
thanks for reply. the assymetry parameter, (Vxx-Vyy)/Vzz,  is
zero (wien2k calculation) whereas i got 0.52 from TDPAC (Time
dependent perturbed angular correlation) spectroscopy for a SrTiO3
(STO) defect structure. EFG component is -1.63 x10^21 V/m2 (wien2k,
lapw2 -efg) and i obtained 1.69 x 10^21 V/m2 (calculated from TDPAC
results). STO has cubic lattice so no efg and no assymetry
parameter (for both wien2k and TDPAC) but defect STO structure
showed very pure hyperfine interactions with assymetry parameter :
0.52. So i am not having confidence over my wien2k calculation
because :
1. i am getting assymetry parameter =0
2. Negative EFG which i understood from previous answers that
negative s

Re: [Wien] Qestion about DOS results

2018-12-12 Thread pieper
 ...


Good luck,

Martin Pieper


---
Dr. Martin Pieper
Karl-Franzens University
Institute of Physics
Universitätsplatz 5
A-8010 Graz
Austria
Tel.: +43-(0)316-380-8564


Am 11.12.2018 23:06, schrieb shaymlal dayananda:

Dear all

Sorry, my reply to the original mail chain is waiting for the
moderator approval! So I am sending this as a new email.



I have actually considered hubard-U (4.5 eV is included) and spin
orbital coupling also added. My structure is U3O8,

case.indmc
 -12. Emin cutoff energy
  2   number of atoms for which density matrix is
calculated
  1  1  3 index of 1st atom, number of L, L1
  2  1  3 index of 1st atom, number of L, L1
  0  0krad, kls

case.inorb

  1  2  0 nmod, natorb, ipr
 PRATT  1.0BROYD/PRATT, mixing
  1 1 3  iatom nlorb, lorb
  2 1 3  iatom nlorb, lorb
  1  nsic 0..AMF, 1..SIC, 2..HFM
   0.3307 0.00U J (Ry)   Note: we recommend to use U_eff = U-J
and J=0
   0.3307 0.00U J

I am having a followup question for your comments.
1. Can I conclude FM for my system?

  because: atom-1(uranium1) is non-magnetic, atom-2 uranium
(multiplicity is 2 for this atom) has a magnetic moment  of 0.71935.
These two  uranium has parallel magnetism.

2. This system is U3O8, (not U2O5). It has only 11 atoms in the unit
cell as 1 (U), 2(U), 2(U), 3(O), 4(O), 4(O), 5 (O), 6(O), 6(O), 7(O),
7(O).
With this, is this possible to accept the obtained DOS?
(And do we necessarily should get different DOS for FM/AFM cases for
their spin UP/DN cases?

Thank you

Shayam
___
Wien mailing list
Wien@zeus.theochem.tuwien.ac.at
http://zeus.theochem.tuwien.ac.at/mailman/listinfo/wien
SEARCH the MAILING-LIST at:
http://www.mail-archive.com/wien@zeus.theochem.tuwien.ac.at/index.html

___
Wien mailing list
Wien@zeus.theochem.tuwien.ac.at
http://zeus.theochem.tuwien.ac.at/mailman/listinfo/wien
SEARCH the MAILING-LIST at:  
http://www.mail-archive.com/wien@zeus.theochem.tuwien.ac.at/index.html


Re: [Wien] Regarding Local environments of individual atom in a structure

2018-09-25 Thread pieper

High Sandeep,

I think this has not been answered, so I will give it a try:

Yes, case.outputnn would be one place to look for information about 
local environments. By way of an example I copied a few lines from 
case.outputnn of a Co hcp structure. The lines starting with % are 
comments I inserted


...
 ATOM:  1  EQUIV.  1  Co AT   0.3   0.7   0.25000
% Specifies the environment of which atom is listed in the following 
lines.

% Position in crystal coordinates.
 RMT(  1)=2.34000 AND RMT(  1)=2.34000
% RMT of that atom (there are 2 Co per unit cell) and of a nearest 
neighbor

 SUMS TO 4.68000  LT.  NN-DIST= 4.71810
% nearest neighbor distance must be greater than sum of RMT's
 ATOM:  1  CoAT -0.  0. -0.2500 IS  4.71810 A.U.   
2.49671 ANG

% Position in crystal coordinates of the first nearest neighbor,
& and distance in a.u. and Angstrom
 ATOM:  1  CoAT -0.  0.  0.7500 IS  4.71810 A.U.   
2.49671 ANG

% The same for the second nearest neighbor
 ATOM:  1  CoAT  0.6667  1. -0.2500 IS  4.71810 A.U.   
2.49671 ANG
 ATOM:  1  CoAT  0.6667  1.  0.7500 IS  4.71810 A.U.   
2.49671 ANG
 ATOM:  1  CoAT  0.6667  0. -0.2500 IS  4.71810 A.U.   
2.49671 ANG
 ATOM:  1  CoAT  0.6667  0.  0.7500 IS  4.71810 A.U.   
2.49671 ANG
% There are 6 nearest neighbors in the hcp structure, all at the same 
distance.
 ATOM:  1  CoAT  1.  0.6667  0.2500 IS  4.73773 A.U.   
2.50710 ANG

% Position in crystal coordinates and distance
% of the first next nearest neighbor atom
...

There are also 6 next nearest neighbors in this case, which are listed 
in the following lines. If you need further shells set the associated 
call parameter for nn to higher values (see user guide).


If you need (binding) angles you can calculate them from the position 
data (remember to transform to cartesian coordinates), or use your 
favorite crystal structure viewer (xcrysden works fine).


Take information about local symmetry from case.outputs (or 
case.outputsgroup). In case.outputs you will find for each atom in 
case.struct a list of symmetry elements for that position. For atom 1 in 
th above case of hcp-Co this reads:


...
 ATOM:   1
Co operation #  1 1
Co operation #  4 2 || 120
Co operation #  5 2 || 210
Co operation #  6 2 || 1-10
Co operation # 14 m (s-h) n  z
Co operation # 18 m (s-v)  n  100
Co operation # 19 m (s-v)  n  010
Co operation # 20 m (s-v)  n  110
Co operation # 21 3 || z
Co operation # 22 3 || -z
Co operation # 23 S3 (-6) || z
Co operation # 24 S3 (-6) || -z
% There is identity, three 2-fold axes, four mirror planes,
%two 3-fold axes (along +/-z), and two 6-fold screw axes
  pointgroup is -6m2 (neg. iatnr!!)
% The point group with these symmetry elements
  axes should be: -6 || z, m n y
  z-rotation vector:  0.  0.  1.
  y-rotation vector:  0.  1.  0.2
% Orientation of the local axes, highest symmetry along z
%(here 6-fold rotation), next along y (mirror plane normal y)
  WARNING: LOCAL ROTATION MATRIX CHANGED
LOCAL ROT MATRIX:   NEWOLD
   1.000 0.000 0.000  0.000 0.000 
0.000
   0.000 1.000 0.000  0.000 0.000 
0.000
   0.000 0.000 1.000  0.000 0.000 
0.000

% Local rotation matrices belonging to this orientation (see user guide)
...

I hope this helps. Best regards,

Martin Pieper



---
Dr. Martin Pieper
Karl-Franzens University
Institute of Physics
Universitätsplatz 5
A-8010 Graz
Austria
Tel.: +43-(0)316-380-8564


Am 18.09.2018 12:34, schrieb sandeep Kumar:

Dear Prof. Peter Blaha and WIEN2k Users,

I am trying to find the local environments of an individual atom in a
structure. In VASP code, it is written clearly in the OUTCAR file that
how many atoms and which atom are nearest neighbour around an
individual atom in a structure. We can also analysis local
environments from the structure itself using VESTA.

Can we do in the WIEN2k code using case.outputnn file ? Could you
please explain it to me?

Thanks

Sandeep

--

Dr. Sandeep Kumar (Postdoctoral Research Fellow)
Institute for Nanotechnology & Advanced Materials,
Department of Chemistry,
Bar-Ilan University, Ramat-Gan 52900, Israel
___
Wien mailing list
Wien@zeus.theochem.tuwien.ac.at
http://zeus.theochem.tuwien.ac.at/mailman/listinfo/wien
SEARCH the MAILING-LIST at:
http://www.mail-archive.com/wien@zeus.theochem.tuwien.ac.at/index.html

___
Wien mailing list
Wien@zeus.theochem.tuwien.ac.at
http://zeus.theochem.tuwien.ac.at/mailman/listinfo/wien
SEARCH the MAILING-LIST at:  
http://www.mail-archive.com/wien@zeus.theochem.tuwien.ac.at/index.html


Re: [Wien] charged and neutral cell

2017-12-01 Thread pieper
As I said before: when you remove an atom from case.struct, you remove 
the nuclear charge AND all Z electrons. This is neutral. If your perfect 
structure was neutral, then the new structure is neutral as well.


Don't try to tell Wien2k that Ba should be missing 2 electrons and O 
should have them. It makes no sense: with Wien2k you have a very 
sophisticated tool which has been designed by extremely clever people to 
do with enormous precision exactly what you apparently want to do by 
hand: distribute electrons in a solid. Throw the program a handfull of 
neutral atoms, tell it their positions - thats what the struct file is 
for - and let it do it's magic.


As a guidance, I really recommend the UG, and use the w2web interface. 
In short (fill in the gaps from the UG):


- create a case directory for the perfect structure (perhaps using the 
web interface w2web). Generate a valid case.struct file (via w2web's 
structeditor, from a cif file ... whatever). Walk through 
initialization. Run the scf. Inspect the file case.scf. Calculate DOS, 
do a band structure spagetti plot ... I gues you have done this already.


- create a new case directory and copy the struct file of the perfect 
structure to this directory. Use supercell to generate your 2x2x2 unit 
cell (perhaps P, not centered). Rename it to match the directory name. 
Use w2webs the structeditor to kill one Ba atom, if that is what you 
want to do first. (A 2x2x2 unit cell will be a pedagogical exercise, it 
is too small to accomodate the structural relaxation such a massive 
defect will need). Make sure that the RMT's in the struct file are at 
least 5% from touching since you definitely want to give Wien2k a chance 
to relax the defect structure (see UG on structural relaxation). Walk 
step by step through initialization: nn first. It will be unhappy with 
the multiplicities and creat a new struct. Accept the this new struct 
file. Next sgroup, which will most probably again make its own changes. 
Accept them. With a little bit of luck this struct file is then ok for 
the symmetry program. If the symmetry program crashes it may be due to 
different rounding of exact positions in sgroup and symmetry - there was 
a recent thread about in the mailing list. If symmetry is happy you are 
probably in the green with the defect structure - inspect the structure 
with your favorite crystal structure viewing tool, perhaps xcrysden. 
Proceed with initialization. If you want run a simple scf without 
structural relaxation as a reference. For anything beyond pedagogical 
exercises I think scf runs with structural relaxation are mandatory.


- rinse and repeat with whatever defect structure you are interested in, 
each in fresh case directory.



---
Dr. Martin Pieper
Karl-Franzens University
Institute of Physics
Universitätsplatz 5
A-8010 Graz
Austria
Tel.: +43-(0)316-380-8564


Am 30.11.2017 20:27, schrieb chin Sabsu:

Dear Pieper Sir,

Thank you for detailed reply.

Okay, I understood few things now.
But if we remove one atom from the super cell then how we can say that
the defective cell is charge neutral?

I assume that I will be convinced from your next reply  that "how a
defective supercell is considered charge neutral (A^+2 from 2x2x2 cell
of ABO3)".

Now Please guide me how I can create  fully charged A or B or  oxygen
vacancies defects in ABO3 system?

Thank you in advance.

Sincerely
Chin

 On Thursday, 30 November 2017 6:26 PM, pieper
<pie...@ifp.tuwien.ac.at> wrote:

No, if you remove atoms in the case.struct file, you remove atoms,
that
is neutral entities. Just don't play around with the entry Z for the
number of electrons (which you should not do if you are not sure what
you are doing). It defaults to the nuclear charge - use the w2w2b
interface and leave the entry empty in structgen, it will be filled in

automatically. You need not do anything special during initialization
to
maintain charge neutrality.

As for the second question, readjust your concept of valence to align
with how it is used within Wien2k. In a nutshell, distinction between
core and valence electrons in Wien2k is made by their binding energy
to
a specific nuclear charge. If the binding energy is large enough, the
total charge density of the electron is to high accuracy within the
sphere of radius RMT around that nuclear charge - this is a core
electron. If this is not the case, its a valence electron. It is much
more difficult to be a core electron in Wien2k than it is in your
classical thinking of Ba2+ and O2-. Leave that distinction to the
program lstart during initialization. (There is a lot more, look into
the introductory section of the UG and continue reading from there).

If you want to calculate defect structures, start from the perfect
structure. Perhaps begin with examples that come with the
Wien2k-package. After a successful scf run inspect the file case.scf.
Search (grep) for
:NEC
You will find two numbers, the total number of charges, nucle

Re: [Wien] charged and neutral cell

2017-11-30 Thread pieper
No, if you remove atoms in the case.struct file, you remove atoms, that 
is neutral entities. Just don't play around with the entry Z for the 
number of electrons (which you should not do if you are not sure what 
you are doing). It defaults to the nuclear charge - use the w2w2b 
interface and leave the entry empty in structgen, it will be filled in 
automatically. You need not do anything special during initialization to 
maintain charge neutrality.


As for the second question, readjust your concept of valence to align 
with how it is used within Wien2k. In a nutshell, distinction between 
core and valence electrons in Wien2k is made by their binding energy to 
a specific nuclear charge. If the binding energy is large enough, the 
total charge density of the electron is to high accuracy within the 
sphere of radius RMT around that nuclear charge - this is a core 
electron. If this is not the case, its a valence electron. It is much 
more difficult to be a core electron in Wien2k than it is in your 
classical thinking of Ba2+ and O2-. Leave that distinction to the 
program lstart during initialization. (There is a lot more, look into 
the introductory section of the UG and continue reading from there).


If you want to calculate defect structures, start from the perfect 
structure. Perhaps begin with examples that come with the 
Wien2k-package. After a successful scf run inspect the file case.scf. 
Search (grep) for

:NEC
You will find two numbers, the total number of charges, nuclear and 
electrons. They should be equal within the accuracy lstart uses to 
decide wether or not charge density leaks out of the RMT-sphere.


You will also find
:NOE
Which is the total number of not-core electrons, or

:CTOnnn
which is the total charge (in one spin channnel) in sphere nnn, and many 
other projections of the charge density on specific volumes, energy 
windows, spin or orbital states. Use the UG section on the scf file as 
an introduction. In adition you can use analysis programs like AIM (see 
UG, section on AIM) to calculate details.


Then create a new case directory and restart there by modifying your 
file case.struct - perhaps using programs like structgen and supercell.


Good luck



---
Dr. Martin Pieper
Karl-Franzens University
Institute of Physics
Universitätsplatz 5
A-8010 Graz
Austria
Tel.: +43-(0)316-380-8564


Am 30.11.2017 06:12, schrieb chin Sabsu:

Dear Peter Sir and other experts,

Could you please explain how to deal charged vacancy and neutral
vacancy?

If I remove an atom (say x^+2) from the unit cell then the Unit cell
will be charged unit cell with charge -2e.
If I remove an atom (say x^-2) from the unit cell then the Unit cell
will be charged unit cell with charge +2e.

Are the above statements are correct?

If above statements are correct then please suggest me how to make the
system neutral after creating vacancy.

Below are the lines from case.in2 and case.inm where, as far as I
know, I need to  do some changes.

Assuming that I have xx NE in case.in2 and I remove an atom of vacancy
+2 then how I should modify these two files so that the final
defective cell remains charge neutral?

case.in2
TOT (TOT,FOR,QTL,EFG,FERMI)
   -13.2xx.0   0.50 0.05  1   EMIN, NE, ESEPERMIN, ESEPER0,
iqtlsave

case.inm
MSR1   0.0   YES  (BROYD/PRATT, BG charge (-1 for core hole), norm)

I have one another question:

How the number NE (here I kept it xx) in case.in2 changes case by
case?

In case of BaZrO3 it, NE, is 40 but I could not find any way to
calculate this number manually because Ba is having +2, Zr also +2 and
O is -2. Total number of atoms are 5(Ba*1, Zr*1 and O*3) in the unit
cell and below is electronic configuration of atoms.

Ba: _1s22s2_2p6_3s2_3p63d10_4s2_4p64d10_5S2_5P6_6S2_
Zr: _1s22s2_2p6_3s2_3p63d10_4S2_4P6_4D25S2_
_O:__1s2 2S2 2P4_

_Bold are treated as F (valence)  states during initialization._
Looking forward to hearing from any of the expert.

Sincerely

Chin
___
Wien mailing list
Wien@zeus.theochem.tuwien.ac.at
http://zeus.theochem.tuwien.ac.at/mailman/listinfo/wien
SEARCH the MAILING-LIST at:
http://www.mail-archive.com/wien@zeus.theochem.tuwien.ac.at/index.html

___
Wien mailing list
Wien@zeus.theochem.tuwien.ac.at
http://zeus.theochem.tuwien.ac.at/mailman/listinfo/wien
SEARCH the MAILING-LIST at:  
http://www.mail-archive.com/wien@zeus.theochem.tuwien.ac.at/index.html


Re: [Wien] Ueff

2017-11-28 Thread pieper


Until one of the real experts (I am not among them) finds time to 
answer: My guess is that both is correct, Ueff=U-J as well as 
Ueff=U-J/2. That is, the meaningful thing to do is set J=0 (see UG 
section on ORB), and not estimate it from DFT.


The common ground between ORB or DFT and the magnetism of atoms as in 
Hunds rules is a multiband Hubbard model.


For the latter, the multiple correlation energy parameters of such a 
model are summarized into one intra-orbital U, one inter-orbital V, and 
a Hund's rule parameter J reducing the inter-orbital correlation energy 
for same spin direction. [1].


For the former, the parameter Ueff represents on-site contributions to 
the correlation energy which are underestimated by L(S)DA. Ueff is, 
therefore, an approximation that has to estimate the contribution 
already accounted for by L(S)DA  - which leads to options 1,2,3 for 
different self-interaction corrections.


I doubt that there is a general and safe way to assign specific parts of 
the correlation energy to U, V, and J. At least, the authors of the ORB 
code recommend to avoid the ambiguities [2], and to set J=0.


[1] see for example, the talk 'Localized Electrons with Wien2k' by Elias 
Assmann Wien, workshop 2013, ldau.pdf among the documentations on the 
Wien2k site

[2] G. K. H. Madsen and P. Novák, Europhys. Lett. 69, 777 (2005)

---
Dr. Martin Pieper
Karl-Franzens University
Institute of Physics
Universitätsplatz 5
A-8010 Graz
Austria
Tel.: +43-(0)316-380-8564


Am 28.11.2017 08:42, schrieb karima Physique:

Dear prof. P. Blaha and wien2k users :

I am still waiting for your answers and I thank you in advance.

my question was :

Does Ueff=U-J or Ueff=U- 1/2 J? how to estimate J using constrained
DFT.?

Thank you in advance

2017-11-27 12:05 GMT+01:00 karima Physique
<physique.kar...@gmail.com>:


Dear Wien2k users:

Does Ueff=U-J or Ueff=U- 1/2 J? how to estimate J using constrained
DFT.?

Thank you in advance

___
Wien mailing list
Wien@zeus.theochem.tuwien.ac.at
http://zeus.theochem.tuwien.ac.at/mailman/listinfo/wien
SEARCH the MAILING-LIST at:
http://www.mail-archive.com/wien@zeus.theochem.tuwien.ac.at/index.html

___
Wien mailing list
Wien@zeus.theochem.tuwien.ac.at
http://zeus.theochem.tuwien.ac.at/mailman/listinfo/wien
SEARCH the MAILING-LIST at:  
http://www.mail-archive.com/wien@zeus.theochem.tuwien.ac.at/index.html


Re: [Wien] Question Regarding NMR Calculations and Nuclear quadrupole coupling Constant

2017-11-23 Thread pieper

To clarify this:

Yes, there is a line shift by nuclear quadrupole interaction, and, 
depending on the size of the EFG, it can be significant for the 
determinatation of chemical shift or even Knight shift. It appears in 
perturbation theory beyond first order, which describes the familiar 
quadrupole splitting. For this you might want to look at


M. H. Cohen and F. Reif,
Quadrupole Effects in Nuclear Magnetic Resonance Studies of Solids,
Solid State Physics, Vol 5
Academic Press, 1957
editors F. Seitz and D. Turnbull

Of course, nowadays you can also do a numerical diagonalization of the 
Hamiltonian for magnetic field and quadrupole interaction.


Best regards,


---
Dr. Martin Pieper
Karl-Franzens University
Institute of Physics
Universitätsplatz 5
A-8010 Graz
Austria
Tel.: +43-(0)316-380-8564


Am 22.11.2017 13:33, schrieb Peter Blaha:

This is a question to NMR-experimentalists. They usually know how they
obtain the CS and quadrupol splitting from their experimental data.

I don't think the quadrupole moment influences the value of the CS.

On 11/22/2017 11:24 AM, sandeep Kumar wrote:

Dear Professor Peter Blaha and Dr. Robert Laskowski,

It is known thatquadrupolar nuclei such as 17O the resonance frequency
is a combination of the chemical shift and the isotropic quadrupole
coupling (goes like Cq^2/w0, Cq is the coupling and w0 is the Larmor
frequency) and for a perfectly symmetric environment it should be 
zero.




I have calculated NMR chemical shift and quadrupole coupling 
constantof

an insulator systems.  My question is how we can calculate the
contributions of Cq in chemical shifts and how much
the quadrupole moment influences the chemical shift?



Please correct me if I am wrong.



Thanks



Sandeep Kumar


--
Dr. Sandeep Kumar, Post-doc
Department of Chemistry,
The Lise Meitner-Minerva Center for Computational Quantum Chemistry &
The Institute for Nanotechnology and Advanced Materials,
Bar-Ilan University, Ramat-Gan 52900, Israel



___
Wien mailing list
Wien@zeus.theochem.tuwien.ac.at
http://zeus.theochem.tuwien.ac.at/mailman/listinfo/wien
SEARCH the MAILING-LIST at:  
http://www.mail-archive.com/wien@zeus.theochem.tuwien.ac.at/index.html



___
Wien mailing list
Wien@zeus.theochem.tuwien.ac.at
http://zeus.theochem.tuwien.ac.at/mailman/listinfo/wien
SEARCH the MAILING-LIST at:  
http://www.mail-archive.com/wien@zeus.theochem.tuwien.ac.at/index.html


Re: [Wien] HDLO and LVNS

2017-11-13 Thread pieper

High Pablo,

I think there is a reasonable amount of information in the UG, including 
a citation for further reading (which I never followed):


UG Version 17.1, ch. 7.5, first par.:

When adding such LOs, make sure their E-parameters are far away from 
each other (typically at least 1 Ry, often 2.4 Ry), otherwise ghostbands 
may occur. In addition (or sometimes, alternatively) a second-derivative 
(HDLO) LO for improved E-linearization of valence bands (in particular 
for d and f-states and large RMTs)[Karsai et al., 2017] can be defined.


So, HD indeed stands for 'higher derivative' in general, and in Wien2k 
2nd derivatives are implemented.


Furthermore, see the 'Interpretive comments' for case.in1 in the UG, 
there especially the comments on NAPWL.


Maybe you also want to note that there is a warning in the OPTICS 
section that HDLO's are not supported in that package.


Best regards,

Martin Pieper

---
Dr. Martin Pieper
Karl-Franzens University
Institute of Physics
Universitätsplatz 5
A-8010 Graz
Austria
Tel.: +43-(0)316-380-8564


Am 12.11.2017 23:46, schrieb delamora:

Thank you, but I think that the usersguide should have a minimal
explanation, for example, the two lines that you wrote.

-

DE: Wien <wien-boun...@zeus.theochem.tuwien.ac.at> en nombre de Gavin
Abo <gs...@crimson.ua.edu>
ENVIADO: domingo, 12 de noviembre de 2017 12:57 p. m.
PARA: wien@zeus.theochem.tuwien.ac.at
ASUNTO: Re: [Wien] HDLO and LVNS

I believe HD stands for high derivative.  See where it says high
derivative LO (HDLO) in the Computer Physics Communications Vol. 220
p. 230 (2017) article:

https://doi.org/10.1016/j.cpc.2017.07.008

On 11/12/2017 8:22 AM, delamora wrote:


Dear Peter Blaha,
In the 17.1 version the HDLO and LVNS appear;
atom 1 has a large sphere , consider setting HDLOs and/or
larger LVNS
I searched in the usersguide and I did not find much information;
For LVNS there is no explanation, all I found is in 5.1.3;
-lvns L -> in batch mode: LVNS_max (default: 4)
and for HDLO it is not clear what 'HD' stand for

Cheers

Pablo de la Mora

___
Wien mailing list
Wien@zeus.theochem.tuwien.ac.at
http://zeus.theochem.tuwien.ac.at/mailman/listinfo/wien
SEARCH the MAILING-LIST at:
http://www.mail-archive.com/wien@zeus.theochem.tuwien.ac.at/index.html

___
Wien mailing list
Wien@zeus.theochem.tuwien.ac.at
http://zeus.theochem.tuwien.ac.at/mailman/listinfo/wien
SEARCH the MAILING-LIST at:  
http://www.mail-archive.com/wien@zeus.theochem.tuwien.ac.at/index.html


Re: [Wien] Number of charge concentration in the system

2017-11-03 Thread pieper

No, that is the total number of valence electrons (see User Guide).

For results you should first look at case.scf (though volume and total 
electron number is hardly a result of Wien2k ...). For example use the 
grep command in a terminal set in your case directory:



grep VOLUME *.scf

:VOL  : UNIT CELL VOLUME = 149.45692


grep NUCLEAR *.scf

:NEC01: NUCLEAR AND ELECTRONIC CHARGE 54.053.99861

(if I do these grep in a dir with a Wien2k case for hcp-Co, a hexagonal 
structure with 2 Co per unit cell)



grep ELECTRONS *.scf

:NOE  : NUMBER OF ELECTRONS  =  30.000

(the number of valence electrons, like file case.in2)

Or view case.scf in your favorite editor and search for 'UNIT CELL', 
'NUCLEAR', 'VALENCE', or labels likes ':VOL', ':NOE', ':NEC', ... See 
the UG for a list and an explanation what (most of) the labels mean.


---
Dr. Martin Pieper
Karl-Franzens University
Institute of Physics
Universitätsplatz 5
A-8010 Graz
Austria
Tel.: +43-(0)316-380-8564


Am 02.11.2017 02:28, schrieb halim said:

Dear Professor Pieper,

Thank you very much for your answer and clarification, and detailed
answer.

Do you mean the total numbers of electrons is NE printed in file.in2?

Thank you for your answer.

Halim Said

 Le Mercredi 1 novembre 2017 22h26, pieper <pie...@ifp.tuwien.ac.at> a
écrit :

You want to know the overall electron concentration? Why use Wien2k
for
that one?

If your unit cell is spanned by crystallografic basis vectors a,b,c
then
the volume is V=a*(b x c).

If your unit cell contains n_i atoms of elements nuclear charge Z_i
the
total number of electrons is N=sum_i(n_i*Z_i).

The concentration is c=N/V.

And yes, volume V as well as total number of electrons N are printed
in
Wien2k's result file ...

---
Dr. Martin Pieper
Karl-Franzens University
Institute of Physics
Universitätsplatz 5
A-8010 Graz
Austria
Tel.: +43-(0)316-380-8564

Am 01.11.2017 05:19, schrieb halim said:

Dear Wien2k  users community,

I would like to ask you if we can get the results of  total number

of

concentration of the system from wien2k code

Looking forward your answer.

Halim Said
___
Wien mailing list
Wien@zeus.theochem.tuwien.ac.at
http://zeus.theochem.tuwien.ac.at/mailman/listinfo/wien
SEARCH the MAILING-LIST at:


http://www.mail-archive.com/wien@zeus.theochem.tuwien.ac.at/index.html
___
Wien mailing list
Wien@zeus.theochem.tuwien.ac.at
http://zeus.theochem.tuwien.ac.at/mailman/listinfo/wien
SEARCH the MAILING-LIST at:
http://www.mail-archive.com/wien@zeus.theochem.tuwien.ac.at/index.html
___
Wien mailing list
Wien@zeus.theochem.tuwien.ac.at
http://zeus.theochem.tuwien.ac.at/mailman/listinfo/wien
SEARCH the MAILING-LIST at:
http://www.mail-archive.com/wien@zeus.theochem.tuwien.ac.at/index.html

___
Wien mailing list
Wien@zeus.theochem.tuwien.ac.at
http://zeus.theochem.tuwien.ac.at/mailman/listinfo/wien
SEARCH the MAILING-LIST at:  
http://www.mail-archive.com/wien@zeus.theochem.tuwien.ac.at/index.html


Re: [Wien] Number of charge concentration in the system

2017-11-01 Thread pieper
You want to know the overall electron concentration? Why use Wien2k for 
that one?


If your unit cell is spanned by crystallografic basis vectors a,b,c then 
the volume is V=a*(b x c).


If your unit cell contains n_i atoms of elements nuclear charge Z_i the 
total number of electrons is N=sum_i(n_i*Z_i).


The concentration is c=N/V.

And yes, volume V as well as total number of electrons N are printed in 
Wien2k's result file ...



---
Dr. Martin Pieper
Karl-Franzens University
Institute of Physics
Universitätsplatz 5
A-8010 Graz
Austria
Tel.: +43-(0)316-380-8564


Am 01.11.2017 05:19, schrieb halim said:

Dear Wien2k  users community,

I would like to ask you if we can get the results of  total number of
concentration of the system from wien2k code

Looking forward your answer.

Halim Said
___
Wien mailing list
Wien@zeus.theochem.tuwien.ac.at
http://zeus.theochem.tuwien.ac.at/mailman/listinfo/wien
SEARCH the MAILING-LIST at:
http://www.mail-archive.com/wien@zeus.theochem.tuwien.ac.at/index.html

___
Wien mailing list
Wien@zeus.theochem.tuwien.ac.at
http://zeus.theochem.tuwien.ac.at/mailman/listinfo/wien
SEARCH the MAILING-LIST at:  
http://www.mail-archive.com/wien@zeus.theochem.tuwien.ac.at/index.html


Re: [Wien] Querry in a resultant structure

2017-10-27 Thread pieper
In my humble opinion YOU have to decide wether your structure looks like 
what you want. I only can/will expand somewhat on my previous advice to 
look at case.outputnn and to compare structures in xcrysden:


Since xcrysden is difficult about arrows, do this in colors: You have 4 
purple marbles in your unit cell. There are 3 ways to replace two of 
them by identical green marbles, depicted as AFM I - III in your figure. 
So, to avoid the fuss of talking xcrysden into plotting arrows, take the 
supercell generated .struct file and produce 3 different .struct files 
by replacing two Ni with, say, Co. Look at these with xcrysden and 
decide which is what. Rename all Ni-colored atoms Ni1, all Co-colored 
atoms Ni2 (and perhaps make all S identical) and ask sgroup what it 
thinks of the idea.


As for the mysterious downgrading from rhombohedral to monoclinic: Look 
at your structures along the (1,1,1) direction. Is there a 3-fold 
symmetry?


I hope this helps,


---
Dr. Martin Pieper
Karl-Franzens University
Institute of Physics
Universitätsplatz 5
A-8010 Graz
Austria
Tel.: +43-(0)316-380-8564


Am 25.10.2017 22:39, schrieb Abderrahmane Reggad:

Hello again

Here is the 3 different AFM configurations

https://ibb.co/mqySFm

Here are the 4 independent Ni atoms

https://ibb.co/nt2nFm

Now the big problem lies in if is it possible tp get a monoclinic
structure (space group #11) with orthorhombic lattice parameters as we
know that the rhombohedral structure in wien2k is represented bt a
hexagonal lattice parameters.

Best regards
___
Wien mailing list
Wien@zeus.theochem.tuwien.ac.at
http://zeus.theochem.tuwien.ac.at/mailman/listinfo/wien
SEARCH the MAILING-LIST at:
http://www.mail-archive.com/wien@zeus.theochem.tuwien.ac.at/index.html

___
Wien mailing list
Wien@zeus.theochem.tuwien.ac.at
http://zeus.theochem.tuwien.ac.at/mailman/listinfo/wien
SEARCH the MAILING-LIST at:  
http://www.mail-archive.com/wien@zeus.theochem.tuwien.ac.at/index.html


Re: [Wien] Querry in a resultant structure

2017-10-25 Thread pieper
It is a valid procedure to generate a unit cell that supports placing 
antiparallel spins on the two Ni sublattices containing Ni1 and Ni2 
atoms.


I cannot tell you wether the new structure is 'true' or not.

Concerning the crystal structure you simply should check youerself with 
xcrysden (or some similar program): There should be no visible change in 
the positions when viewing the two structures from the same angle. You 
also should compare the distances to nearest and next nearest neighbors 
for the old and new structure - they are listed in the respective 
case.outputnn and should be identical.


Concerning the magnetic structure I don't know what exactly you mean 
with 'type III' af. If you are uneasy with the concept of an 
antiferromagnetic wave vector you might try to visualize what the 
difference to type I and type II is. Then look into outputnn again and 
decide if you really want these neighbors being antiparallel, or if it's 
the neighbours at some other distance.


Best regards,

Martin Pieper


---
Dr. Martin Pieper
Karl-Franzens University
Institute of Physics
Universitätsplatz 5
A-8010 Graz
Austria
Tel.: +43-(0)316-380-8564


Am 24.10.2017 14:04, schrieb Abderrahmane Reggad:

Thank you pieper for your answer

I am studying the magnetic order of type III with Ni1,Ni2 up and Ni3,
Ni4 down according the figure included.

This is the procedure that I adopted:

-supercell 1x1x1 P to make the 4 atoms Ni indepedent a

-  I make the Ni1 and Ni2 to be the atom Ni1 and the atoms Ni3 and Ni4
to be the atom Ni2.
- sgroup to search a new space group

- I got the new strycture with monoclinic space group but with
orthorhombic lattice parameters.

I want to know if this new structure is true or not.

The non magnetic structure is as follows:

NiS-MnP

P   LATTICE,NONEQUIV.ATOMS:  2 62_Pnma

MODE OF CALC=RELA unit=ang

 10.056514  6.692230 10.722761 90.00 90.00 90.00
ATOM  -1: X=0.0050 Y=0.2500 Z=0.2000

  MULT= 4  ISPLIT= 8

  -1: X=0.9950 Y=0.7500 Z=0.8000

  -1: X=0.5050 Y=0.2500 Z=0.3000

  -1: X=0.4950 Y=0.7500 Z=0.7000

Ni NPT=  781  R0=0.5000 RMT= 2.08Z: 28.0

LOCAL ROT MATRIX:0.000 1.000 0.000

 0.000 0.000 1.000

 1.000 0.000 0.000

ATOM  -2: X=0.2000 Y=0.2500 Z=0.5700

  MULT= 4  ISPLIT= 8

  -2: X=0.8000 Y=0.7500 Z=0.4300

  -2: X=0.7000 Y=0.2500 Z=0.9300

  -2: X=0.3000 Y=0.7500 Z=0.0700

S  NPT=  781  R0=0.0001 RMT= 1.70Z: 16.0

LOCAL ROT MATRIX:0.000 1.000 0.000

 0.000 0.000 1.000

 1.000 0.000 0.000

   8  NUMBER OF SYMMETRY OPERATIONS

Best regards
___
Wien mailing list
Wien@zeus.theochem.tuwien.ac.at
http://zeus.theochem.tuwien.ac.at/mailman/listinfo/wien
SEARCH the MAILING-LIST at:
http://www.mail-archive.com/wien@zeus.theochem.tuwien.ac.at/index.html

___
Wien mailing list
Wien@zeus.theochem.tuwien.ac.at
http://zeus.theochem.tuwien.ac.at/mailman/listinfo/wien
SEARCH the MAILING-LIST at:  
http://www.mail-archive.com/wien@zeus.theochem.tuwien.ac.at/index.html


Re: [Wien] Querry in a resultant structure

2017-10-23 Thread pieper

I don't know what your question is, but hope that a comment might help.

Apparently you did not change size and shape of the unit cell (using a 
1*1*1 supercell), so the lattice constants and angles are the same. 
However, you told sgroup that there is not one type of Ni in the unit 
cell but two. This change in the decoration of the bricks your crystal 
is built of broke some symmetries in their arrangement - now you are 
down to monoclinic.


I don't know exactly what af wave vector you want to establish in your 
structure (+-+-, or ++--++--, or ... counting along which direction?). 
Is your problem that your Ni1 and Ni2 are not the two you actually 
wanted to have antiparallel moments?


If 'afIII' fits into this unit cell (Ni1 and Ni2 do have antiparallel 
moments), and if the atoms are where you want them, run symmetry, set 
spin directions in case.inst, run lstart ...


Type III sounds comlicated enough to not fit into this unit cell - 
meaning that you cannot complete one period of the +/- count in your 
chosen direction within the unit cell. In that case you have to change 
shape and/or size of the unit cell. Maybe the options provided by 
supercell (double the cell along basis directions ...) are sufficient. 
There is no guarantee for this to be the case. You may have to work out 
the fitting new shape and the positions of the atoms in the new unit 
cell using additional tools.


I hope this helps,

Martin Pieper


---
Dr. Martin Pieper
Karl-Franzens University
Institute of Physics
Universitätsplatz 5
A-8010 Graz
Austria
Tel.: +43-(0)316-380-8564


Am 22.10.2017 20:57, schrieb Abderrahmane Reggad:

Dear wien users

I am studing the magnetic structure of type III for a orthorhombic
structure.

After doing the supercell and labelling the independent atoms for
spin-up and spin-dn atoms and using the sgroup command I got the
following structure:

The space group of a monoclinic structure and the lattice parameters
are those of orthorhombic structure.

NiS-MnP-afmIII

P   LATTICE,NONEQUIV.ATOMS:  4 11_P21/m

MODE OF CALC=RELA unit=bohr

 10.998349 10.005529  6.864228 90.00 90.00 90.00

ATOM  -1: X=0.25021227 Y=0.1424 Z=0.2500
  MULT= 2  ISPLIT= 8
  -1: X=0.74978773 Y=0.8576 Z=0.7500
Ni1NPT=  781  R0=0.5000 RMT= 2.21Z: 28.0

LOCAL ROT MATRIX:1.000 0.000 0.000
 0.000 1.000 0.000
 0.000 0.000 1.000
ATOM  -2: X=0.75024661 Y=0.50012576 Z=0.7500
  MULT= 2  ISPLIT= 8
  -2: X=0.24975339 Y=0.49987424 Z=0.2500
Ni2NPT=  781  R0=0.5000 RMT= 2.21Z: 28.0

LOCAL ROT MATRIX:1.000 0.000 0.000
 0.000 1.000 0.000
 0.000 0.000 1.000
ATOM  -3: X=0.08986359 Y=0.75012482 Z=0.7500
  MULT= 2  ISPLIT= 8
  -3: X=0.91013641 Y=0.24987518 Z=0.2500
S 1NPT=  781  R0=0.0001 RMT= 1.81Z: 16.0

LOCAL ROT MATRIX:1.000 0.000 0.000
 0.000 1.000 0.000
 0.000 0.000 1.000
ATOM  -4: X=0.41013993 Y=0.25012471 Z=0.7500
  MULT= 2  ISPLIT= 8
  -4: X=0.58986007 Y=0.74987529 Z=0.2500
S 2NPT=  781  R0=0.0001 RMT= 1.81Z: 16.0

LOCAL ROT MATRIX:1.000 0.000 0.000
 0.000 1.000 0.000
 0.000 0.000 1.000
   4  NUMBER OF SYMMETRY OPERATIONS

Best regards
___
Wien mailing list
Wien@zeus.theochem.tuwien.ac.at
http://zeus.theochem.tuwien.ac.at/mailman/listinfo/wien
SEARCH the MAILING-LIST at:
http://www.mail-archive.com/wien@zeus.theochem.tuwien.ac.at/index.html

___
Wien mailing list
Wien@zeus.theochem.tuwien.ac.at
http://zeus.theochem.tuwien.ac.at/mailman/listinfo/wien
SEARCH the MAILING-LIST at:  
http://www.mail-archive.com/wien@zeus.theochem.tuwien.ac.at/index.html


Re: [Wien] negative position in rstruc. Please report .

2017-10-20 Thread pieper


In my experience this can happen because of rounding issues with sgroup 
(my guess is in the output format). The structure file generated by 
sgroup (which you seem to have correctly accepted) may have atoms in 
slightly off positions, very close to the symmetric positions they 
should occupy. This may confuse the next program in initialization, 
namely symmetry.


Last time I stumbled into this error message I either
-let supercell generate a shifted unit cell with the origin at a 
different atom. However, there were cases where I was unable to find a 
shift producing a viable unit cell.
- or located rounding errors in the positions generated by sgroup. After 
the sgroup step, open the struct file in w2web's structeditor and look 
for symmetry related coordinates like, for example, 0.499. Set 
them to 0.5 (at a glance, in your struct put 0. instead 0f 
0.3334 etc.).


I hope one of the two works in your case as well



---
Dr. Martin Pieper
Karl-Franzens University
Institute of Physics
Universitätsplatz 5
A-8010 Graz
Austria
Tel.: +43-(0)316-380-8564


Am 20.10.2017 10:25, schrieb mohamed mahdi:

i m trying to performe a calculation for ErCo4.5Si0.5 , after making a
supercell and accepting the structer from wien2k : when running int
again I got this error
the .struct

ERROR: negative position in rstruc. Please report .
___
Wien mailing list
Wien@zeus.theochem.tuwien.ac.at
http://zeus.theochem.tuwien.ac.at/mailman/listinfo/wien
SEARCH the MAILING-LIST at:
http://www.mail-archive.com/wien@zeus.theochem.tuwien.ac.at/index.html

___
Wien mailing list
Wien@zeus.theochem.tuwien.ac.at
http://zeus.theochem.tuwien.ac.at/mailman/listinfo/wien
SEARCH the MAILING-LIST at:  
http://www.mail-archive.com/wien@zeus.theochem.tuwien.ac.at/index.html


Re: [Wien] Spin Magnetic susceptibility

2017-10-03 Thread pieper

My two cent on this:

Spins of itinerant electrons or, more specifically, electrons in the 
immediate vicinity of the Fermi surface usually will dominate the 
magnetic field response at T=0 (Pauli susceptibilty) - if there is a 
Fermi surface, i.e. if you have a metal, and if you do not consider 
magnetic order.


However, I would not jump to the conclusion that this is the 'only' spin 
contribution. Basically chi=M/H. The spin part of the magnetization M is 
the field (H) induced occupation difference between spin-up and -down 
channel. It occures because of the Zeeman interaction between electron 
spin and field. Wether or not the electrons are localized doesn't 
matter. An example might be the large spin susceptibility of 
antiferromagnetic insulators.


Best regards,

---
Dr. Martin Pieper
Karl-Franzens University
Institute of Physics
Universitätsplatz 5
A-8010 Graz
Austria
Tel.: +43-(0)316-380-8564


Am 02.10.2017 18:44, schrieb karima Physique:

Thank you very much and as you said, we are still waiting for a
confirmation from Prof. P. Blaha and Wien2k users.

2017-09-30 21:53 GMT+02:00 Yundi Quan <yq...@ucdavis.edu>:


Hi,

As I understand it, the susceptibility you obtain by shifting DOS is
the Pauli susceptibility, i.e. itinerant electrons near the Fermi
level. But I might be wrong.

On Sat, Sep 30, 2017 at 8:40 AM, karima Physique
<physique.kar...@gmail.com> wrote:


Dear Prof. P. Blaha and Wien2k user;

Is the spin part of the magnetic susceptibility due to the
contribution of delocalized electrons only?

___
Wien mailing list
Wien@zeus.theochem.tuwien.ac.at
http://zeus.theochem.tuwien.ac.at/mailman/listinfo/wien [1]
SEARCH the MAILING-LIST at:




http://www.mail-archive.com/wien@zeus.theochem.tuwien.ac.at/index.html

[2]


___
Wien mailing list
Wien@zeus.theochem.tuwien.ac.at
http://zeus.theochem.tuwien.ac.at/mailman/listinfo/wien [1]
SEARCH the MAILING-LIST at:


http://www.mail-archive.com/wien@zeus.theochem.tuwien.ac.at/index.html

[2]




Links:
--
[1] http://zeus.theochem.tuwien.ac.at/mailman/listinfo/wien
[2] 
http://www.mail-archive.com/wien@zeus.theochem.tuwien.ac.at/index.html

___
Wien mailing list
Wien@zeus.theochem.tuwien.ac.at
http://zeus.theochem.tuwien.ac.at/mailman/listinfo/wien
SEARCH the MAILING-LIST at:
http://www.mail-archive.com/wien@zeus.theochem.tuwien.ac.at/index.html

___
Wien mailing list
Wien@zeus.theochem.tuwien.ac.at
http://zeus.theochem.tuwien.ac.at/mailman/listinfo/wien
SEARCH the MAILING-LIST at:  
http://www.mail-archive.com/wien@zeus.theochem.tuwien.ac.at/index.html


Re: [Wien] The expression of spin susceptibility

2017-09-25 Thread pieper

Hi Karima,

I am a strong believer in the idea that one learns best what one works 
out by oneself, so I prefer to just point you in a direction: Take


chi=M/B,

where B is in T (beware of a shortcut here between B and mu_0*H, mu_0 
being the vacuum permeability), and M is some density of magnetic 
moments (per volume, weight, particle number, you-name-it).


The units on the left hand side of that equation are stated as 
[m^3/(unit cell content)]. Set aside that Peter Blaha admitted there 
might be a typo there, the use of 10^6 cm^3 there indicates that this is 
meant to be a cgs molar susceptibility.


On the right hand side the units are [mu_B/(bohr^3*T)]. With bohr^3 a 
volume this is clearly a volume susceptibility.


Did you try to convert mu_B/bohr^3 to cgs or SI units? You will find 
conversion factors for mu_B (magnetic moment) and bohr (length) e.g. on 
the NIST website. From there you should be able to convert to cgs and be 
able to tell us what exactly the typo is ...


Best regards,


---
Dr. Martin Pieper
Karl-Franzens University
Institute of Physics
Universitätsplatz 5
A-8010 Graz
Austria
Tel.: +43-(0)316-380-8564


Am 23.09.2017 23:55, schrieb karima Physique:

Dear Prof. P. Blaha and wien2k users;

what the meaning of the number 6.258116 in the expression of spin
susceptibility?
___
Wien mailing list
Wien@zeus.theochem.tuwien.ac.at
http://zeus.theochem.tuwien.ac.at/mailman/listinfo/wien
SEARCH the MAILING-LIST at:
http://www.mail-archive.com/wien@zeus.theochem.tuwien.ac.at/index.html

___
Wien mailing list
Wien@zeus.theochem.tuwien.ac.at
http://zeus.theochem.tuwien.ac.at/mailman/listinfo/wien
SEARCH the MAILING-LIST at:  
http://www.mail-archive.com/wien@zeus.theochem.tuwien.ac.at/index.html


[Wien] Fermi-contact hyperfine fields

2017-09-21 Thread pieper

Dear Wien2k users,

I have a question about the Fermi-contact fields printed at the HFFnnn 
labels in the scf file.


Back in 2010 Peter Blaha advised in this mailing list that the 
Fermi-contact interaction can be underestimated at least for 3d-metals 
like Fe by 10 - 20%
[www.mail-archive.com/wien@zeus.theochem.tuwien.ac.at/msg02201.html] and 
Pavel Novak indicated that in his opinion it might be difficult to 
correct this misbehaviour by some suitable local or semi-local Vxc 
[www.mail-archive.com/wien@zeus.theochem.tuwien.ac.at/msg03011.html].


Since this was pre-NMR-package and at the beginning of hybrids I wonder 
wether the situation has improved in the meantime? The NMR-package as I 
understand it calculates shielding current distributions so I don't 
expect it improves on Fermi-contact fields - is that correct? And is 
there a way to get improved values from hybrids that I am unaware of?


Best regards,

Martin Pieper


--
Dr. Martin Pieper
Karl-Franzens University
Institute of Physics
Universitätsplatz 5
A-8010 Graz
Austria
Tel.: +43-(0)316-380-8564

___
Wien mailing list
Wien@zeus.theochem.tuwien.ac.at
http://zeus.theochem.tuwien.ac.at/mailman/listinfo/wien
SEARCH the MAILING-LIST at:  
http://www.mail-archive.com/wien@zeus.theochem.tuwien.ac.at/index.html


Re: [Wien] WIEN2k_17.1

2017-09-21 Thread pieper


Hi Pavel,

if this is not the trivial missing space, this looks to me as if libxc's 
./config has difficulties with f90 files for your ifort. What's your 
ifort version?


Maybe try to run

./config

without any parameters and look what it says about the compilers it 
finds. Thats what the INSTALL file of libxc seems to say (and what 
worked with me, but I am using gfortran).


Best regards,

Martin Pieper


---
Dr. Martin Pieper
Karl-Franzens University
Institute of Physics
Universitätsplatz 5
A-8010 Graz
Austria
Tel.: +43-(0)316-380-8564


Am 21.09.2017 11:41, schrieb nov...@fzu.cz:

Dear WIEN users,

 I successfully compiled WIEN2k_17.1 with the exception of lapw0. As
required in User's Guide (11.1.2) I downloaded libxc-3.00. However, 
after

./configure FC=ifort--prefix=$LIBXCDIR error appeared:

checking for Fortran flag to compile .f90 files... unknown
configure: error: Fortran could not compile .f90 files

I then changed in configure f90 to f, but a new error appeared:

configure: error: in `/storage/praha1/home/novakpa/libxc-3.0.0':
configure: error: f program to find the size of a Fortran integer 
failed


Can you help please.

Pavel
___
Wien mailing list
Wien@zeus.theochem.tuwien.ac.at
http://zeus.theochem.tuwien.ac.at/mailman/listinfo/wien
SEARCH the MAILING-LIST at:
http://www.mail-archive.com/wien@zeus.theochem.tuwien.ac.at/index.html

___
Wien mailing list
Wien@zeus.theochem.tuwien.ac.at
http://zeus.theochem.tuwien.ac.at/mailman/listinfo/wien
SEARCH the MAILING-LIST at:  
http://www.mail-archive.com/wien@zeus.theochem.tuwien.ac.at/index.html


Re: [Wien] 2 problems with w2web

2017-09-17 Thread pieper
Certainely not the recommended way to do things, but the following 
solved my problem with save_lapw in the w2web Utils menue:


In

file:///opt/WIEN2k/SRC_w2web/htdocs/util/savelapw.pl

change line

 save the results of an eels calculation 
only (innes, elnes, broadspec, qtl files, but no figures)


to

 save the results of an eels calculation 
only (innes, elnes, broadspec, qtl files, but no figures)


that is, change the name of the checkbox from 'p' to 'u',

and change accordingly line

$cmdline .= " -eels" if ($p);

to

$cmdline .= " -eels" if ($u);

No recompile necessary.

As I said before, I am no good around perl, so don't ask me why this 
works, i.e. where $p becomes true. I don't think the browser is to 
blame, since I had the misbehaviour with Firefox, Opera, and Konqueror 
(KDE's own browser).


Best regards,

Martin Pieper



---
Dr. Martin Pieper
Karl-Franzens University
Institute of Physics
Universitätsplatz 5
A-8010 Graz
Austria
Tel.: +43-(0)316-380-8564


Am 16.09.2017 13:28, schrieb Peter Blaha:

No, in my test the default save_lapw from "Utils" saves everything (do
not click on any option)

Yes, :HFFxxx is not printed during MSR1a (minimization) mode, but only
once the script has changed back to MSR1.
This is a bit difficult to fix 

1) The default call of the save_lapw script from the 'Utils' menue 
lists (and saves) only
"qtl*, elnes, innes, inq, inb, broadspec, klist and kgen files saved 
under ./Co-hcp-test".


This probably is the second last of the alternative options one can 
check in w2web's 'save_lapw' menue. I  was unable to call the default 
behaviour since there is no box to check for this.


I guess this is w2web working on a wrong default, the save_lapw 
command from terminal works as advertised.


I noticed this only when I updated my Linux (now OpenSuSe Leap 42.2) 
and simultaneously Wien2k (now Wien2k 17.1). So the problem might be 
connected with my new perl version (perl 5, version 18, subversion 2 
(v5.18.2)).




2) The plot feature of w2web's scf analysis script crashes when 
structural relaxation is active and HFFnnn's are selected for the 
plot.


Probable reason (I ran into this when writing a python script for 
analysis)): It appears that in scf cycles with structural relaxation 
active the HFFnnn labels are printed (perhaps HFF's are only 
calculated?) to case.scf only during the finalizing iterations. This 
makes the number of iterations larger than the number of HFFnnn 
values. Maybe the module filling in the plot arrays is confused by 
this.


Best regards,

Martin Pieper




___
Wien mailing list
Wien@zeus.theochem.tuwien.ac.at
http://zeus.theochem.tuwien.ac.at/mailman/listinfo/wien
SEARCH the MAILING-LIST at:  
http://www.mail-archive.com/wien@zeus.theochem.tuwien.ac.at/index.html


[Wien] 2 problems with w2web

2017-09-13 Thread pieper
I encountered two minor problems with w2web of Wien2k 17.1 (I am no good 
around perl so my analysis is probably poor or simply wrong):


1) The default call of the save_lapw script from the 'Utils' menue lists 
(and saves) only
"qtl*, elnes, innes, inq, inb, broadspec, klist and kgen files saved 
under ./Co-hcp-test".


This probably is the second last of the alternative options one can 
check in w2web's 'save_lapw' menue. I  was unable to call the default 
behaviour since there is no box to check for this.


I guess this is w2web working on a wrong default, the save_lapw command 
from terminal works as advertised.


I noticed this only when I updated my Linux (now OpenSuSe Leap 42.2) and 
simultaneously Wien2k (now Wien2k 17.1). So the problem might be 
connected with my new perl version (perl 5, version 18, subversion 2 
(v5.18.2)).




2) The plot feature of w2web's scf analysis script crashes when 
structural relaxation is active and HFFnnn's are selected for the plot.


Probable reason (I ran into this when writing a python script for 
analysis)): It appears that in scf cycles with structural relaxation 
active the HFFnnn labels are printed (perhaps HFF's are only 
calculated?) to case.scf only during the finalizing iterations. This 
makes the number of iterations larger than the number of HFFnnn values. 
Maybe the module filling in the plot arrays is confused by this.


Best regards,

Martin Pieper



--
Dr. Martin Pieper
Karl-Franzens University
Institute of Physics
Universitätsplatz 5
A-8010 Graz
Austria
Tel.: +43-(0)316-380-8564

___
Wien mailing list
Wien@zeus.theochem.tuwien.ac.at
http://zeus.theochem.tuwien.ac.at/mailman/listinfo/wien
SEARCH the MAILING-LIST at:  
http://www.mail-archive.com/wien@zeus.theochem.tuwien.ac.at/index.html


Re: [Wien] About the magnetic moment of vanadium in vanadium sulphide

2017-09-12 Thread pieper

Hi, A. Reggad

There are no questions in your last posts so the case seems to be 
closed.


However, I am curious, and there seems to be something to learn in 
addition to the nice comments made by G. Fecher:


First, is the band gap (of NiS) really a good observable to fix the HF 
mixing (for VS)? The user guide (ch. 4.5.7, first par.) says:


The onsite-exact- exchange/hybrid methods apply HF only
inside the atomic spheres and only to one particular
orbital. Thus you can use it only for localized electrons
(see [Tran et al., 2006] for details). Onsite- exact-exchange
will NOT improve gaps in sp-semiconductors.

These sulfides are not sp-semiconductors, but how reliable will this be 
here? Are those d-electrons localized?


Second, do you, A. Reggad, consider your 0.0002 mu_B to be zero? What 
about the AF order the Japanese experimentalists reported in the 
publication you yourself forwarded us? Any comments on the magnetic 
moment Prof. Fecher told us about?


Did you follow his advice?

Did you even read it?


---
Dr. Martin Pieper
Karl-Franzens University
Institute of Physics
Universitätsplatz 5
A-8010 Graz
Austria
Tel.: +43-(0)316-380-8564


Am 11.09.2017 15:55, schrieb Abderrahmane Reggad:

Hi Laurence

The amount of 0.05 of exact exchange is the value that gives a gap
value equal to the experimental one for nickel sulphide and close
values for magnetic moments for some 3d transition metal sulphides
like CrS, FeS, CoS and NiS.

You can check my article about the nickel sulphide

http://www.sciencedirect.com/science/article/pii/S0921452617303915

Best regards
___
Wien mailing list
Wien@zeus.theochem.tuwien.ac.at
http://zeus.theochem.tuwien.ac.at/mailman/listinfo/wien
SEARCH the MAILING-LIST at:
http://www.mail-archive.com/wien@zeus.theochem.tuwien.ac.at/index.html

___
Wien mailing list
Wien@zeus.theochem.tuwien.ac.at
http://zeus.theochem.tuwien.ac.at/mailman/listinfo/wien
SEARCH the MAILING-LIST at:  
http://www.mail-archive.com/wien@zeus.theochem.tuwien.ac.at/index.html


Re: [Wien] (no subject)

2017-09-10 Thread pieper
That's why I told you to look critically at both, the experiments and 
the calculations: in experiments a high Neel temperature may hide an AF 
state.


However, you do realize that changing the convergence criteria alone 
almost certainely will NOT give you the large moment moment Gerhard 
Fecher found instead of your practically zero 0.05 mu_B? You will have 
to follow his suggestions about the initial state!



---
Dr. Martin Pieper
Karl-Franzens University
Institute of Physics
Universitätsplatz 5
A-8010 Graz
Austria
Tel.: +43-(0)316-380-8564


Am 09.09.2017 02:08, schrieb Abderrahmane Reggad:

Hi

Before doing the calculation again with the new energy and charge
criterion I let you this old article  http://journals.jps.jp/doi/abs/10.1143/JPSJ.14.196

Best regards
___
Wien mailing list
Wien@zeus.theochem.tuwien.ac.at
http://zeus.theochem.tuwien.ac.at/mailman/listinfo/wien
SEARCH the MAILING-LIST at:
http://www.mail-archive.com/wien@zeus.theochem.tuwien.ac.at/index.html

___
Wien mailing list
Wien@zeus.theochem.tuwien.ac.at
http://zeus.theochem.tuwien.ac.at/mailman/listinfo/wien
SEARCH the MAILING-LIST at:  
http://www.mail-archive.com/wien@zeus.theochem.tuwien.ac.at/index.html


Re: [Wien] About the magnetic moment of vanadium in vanadium sulphide

2017-09-07 Thread pieper

I agree, there is some confusion. There probably always is.

The confusion here is mainly about definition para-, dia-, ferro-, 
antiferro-, heli-, or non-  magnetic. This comes up every now and then.


As I said before, I like the following distinction.

a) Define Non-Magnetic ONLY in context of calculations, and there as a 
model where in the converged ground state all local moments are smaller 
than the numerical noise.


The rest were defined much earlier from an experimental point of view. 
They all are meant to distinguish response to external magnetic field - 
a field that DOES NOT EXIST in your off the shelf DFT!


b) Various kinds of dia- and para-magentism specify wether the 
magnetization induced by an applied field is opposite (diamagnetic) or 
parallel (paramagnetic) to the field.


c) Magnetically ordered states (FM, AFM of various types, ferri-, heli-, 
spin density wave-, ...) are PM in this sense. They are distinguished by 
the spacial arrangement of non-zero static magnetic moments (all 
parallel, exactly compensated antiparallel, not compensated 
antiparallel, not collinear, varying in size continuously, ...).




So in response to your points:

1.: Yes, but a PM response just might be due to some unnoticed AF 
ordered state. Because of Ockham's razor one would have to convince me 
of that, but who know's?


2.: Yes, you can get a non-magnetic ground state from a spin-resolved 
calculation. 'Local moments' always refers to the moments within RMT's 
(or whatever the DFT code supports to assign local moments). The 
electron spins are of course always there, but spin up and down can 
compensate - and often will compensate due to spin degenerate ground 
states.


3.: No, I would be very surprised (and suspicious) if the calculations 
yield some exactly compensating moment on S. For exact compensation, 
that is an AF arrangement, you have to have the same element pointing 
moments in opposite directions. Even the crystalografic lattice sites 
occupied by the antiparallel moments should be the same. I know of no 
counter example but would be interested if there is one. In VS Vanadium 
just might split one of its sublattices into such an AF arrangement.


4.: There is a DOS plotted in the work A. Reggad linked in his earlier 
question.


5.: That DOS certainely looks metalic and the authors claim agreement 
with experiment.


Best regards,

Martin Pieper



---
Dr. Martin Pieper
Karl-Franzens University
Institute of Physics
Universitätsplatz 5
A-8010 Graz
Austria
Tel.: +43-(0)316-380-8564


Am 07.09.2017 11:02, schrieb E.A.Moore:

I have been following this thread and I think there is some confusion.

1. On the thread it said that the experiment showed it was Pauli
paramagnetic. This is the type of magnetism displayed by some metals
e.g. sodium which is only apparent if you apply a magnetic field.

2. If you include spin in your calculation (GGA or GGA + U) you can
only get ferromagnetic, antiferromagnetic or ferromagnetic states. (An
earlier thread deals with how to get paramagnetic states). I think you
can only get a nonmagnetic state if you do not include spin? A
material with Pauli paramagnetism will be antiferromagnetic in
straight forward spin-including calculations.

3. I assume the 0.05 muB refers to the magnetic moment on V. If
vanadium sulphide is antiferromagnetic and the magnetic moment on
Vanadium is 0.05 muB, then there must be a balancing magnetic moment
on the sulphur.

4. I suspect this compound might be alloy-like. Is there considerable
mixing of V and S in the valence bands?

5. Assuming your formula is VS, it might be worth noting that VO shows
some metallic physical properties.

Elaine A. Moore
Reader in theoretical chemistry
The Open University



-Original Message-
From: Wien [mailto:wien-boun...@zeus.theochem.tuwien.ac.at] On Behalf
Of Fecher, Gerhard
Sent: 07 September 2017 08:12
To: A Mailing list for WIEN2k users
Subject: Re: [Wien] About the magnetic moment of vanadium in vanadium 
sulphide


0.05 muB does not mean that it is antiferromagnetic ! what was your
charge convergence criterion ?

You did never answer my question whether you started the EECE
calculation from a converged GGA calculation.

Why do you like to have an afm state when the experiment tells it is 
not ?


Ciao
Gerhard

DEEP THOUGHT in D. Adams; Hitchhikers Guide to the Galaxy:
"I think the problem, to be quite honest with you, is that you have
never actually known what the question is."


Dr. Gerhard H. Fecher
Institut of Inorganic and Analytical Chemistry Johannes Gutenberg - 
University

55099 Mainz
and
Max Planck Institute for Chemical Physics of Solids
01187 Dresden

Von: Wien [wien-boun...@zeus.theochem.tuwien.ac.at] im Auftrag von
Abderrahmane Reggad [jazai...@gmail.com]
Gesendet: Donnerstag, 7. September 2017 00:26
An: wien@zeus.theochem.tuwien.ac.at
Betreff: Re: [Wien] About the magnetic moment o

Re: [Wien] About the magnetic moment of vanadium in vanadium sulphide

2017-09-07 Thread pieper
Your problem is 'if it's possible to get' AF VS with 0.05 mu_B/Vanadium 
in DFT?
IF (capitals are on purpose here) you did everything right to properly 
converge your calculations to the necessary numerical precision, then 
you give the answer yourself: Yes, it is possible if one uses GGA+U or 
+EECE.


Wether or not this is an improved model of VS remains open. You still 
did very little to convince Prof. Fecher or me that you actually did 
things right to achieve that precision.  And you did even less to 
convince me (us) that experiments might have missed the small moment.


Even if you are convinced of both, that you did everything right and the 
experimentalists did something wrong, you still should discuss very 
carefully the physics behind your decision to improve the model of a 
metal by introducing a local orbital potential.


You probably don't need to convince us, but keep it in mind for fights 
with the referees of a publication on this.


Good luck with your calculations

Martin Pieper

---
Dr. Martin Pieper
Karl-Franzens University
Institute of Physics
Universitätsplatz 5
A-8010 Graz
Austria
Tel.: +43-(0)316-380-8564


Am 07.09.2017 00:26, schrieb Abderrahmane Reggad:

Hi Martin

The problem is that I want to know if it's possible to get a such
value of 0.05 MB for atomic magnetic moment for the AFM state of
vanadium sulphide in NiAs structure.

Hafner and Hobbs have found all the calculations converged to the non
magnetic state because they have used the GGA method. To get the AFM
state they have to use either the EECE or GGA+U methods.

I hope you touch the problem

Best regards
___
Wien mailing list
Wien@zeus.theochem.tuwien.ac.at
http://zeus.theochem.tuwien.ac.at/mailman/listinfo/wien
SEARCH the MAILING-LIST at:
http://www.mail-archive.com/wien@zeus.theochem.tuwien.ac.at/index.html

___
Wien mailing list
Wien@zeus.theochem.tuwien.ac.at
http://zeus.theochem.tuwien.ac.at/mailman/listinfo/wien
SEARCH the MAILING-LIST at:  
http://www.mail-archive.com/wien@zeus.theochem.tuwien.ac.at/index.html


Re: [Wien] About the magnetic moment of vanadium in vanadium sulphide

2017-09-06 Thread pieper
I took an admittedly brief look at your link, and I still don't 
understand where your problem really is.


I have made calculations for 5 3d transition metal sulphides with the 
method PBE+EECE and I have found that all the results are ok with the 
exception of the vanadium sulphide.


I have made 3 calculations (nm, fm and afmI ) to determine the magnetic 
state for all the compounds and I found them to have afm state.


What exactly is the discrepancy between your results and the state Hobbs 
and Hafner found? You did not answer Prof. Fechers and my question about 
the size of the moment you calculated. Nor my question of wether you 
find an insulator or a metal. Especially in case of the latter I might 
ask about your insistance on Hund's rule ...


The problem is about the atomic magnetic moment of vanadium and for the 
other results which are not reliable because there is one study where 
they used the non magnetic and paramagnetic states with the same 
meaning which is contrary to what said Pr Blaha and they mentionned 
that they have found to be non magnetic without details.


At a glance, those authors seem, in fact, to distinguish between 'non 
magnetic' and 'paramgentic': NM is used with respect to results of their 
calculations for cases where in the ground state all moments are zero 
(or below some numerical threshold). PM is used for states observed in 
experiments where it is defined by the response of the sample to a 
magnetic field (which they - and you - don't calculate). Not a bad idea, 
as far as I am concerned.


And they do give details on VS in section 3.1 - in fact, a lot more 
details than you give us here about your calculations.


Best regards

---
Dr. Martin Pieper
Karl-Franzens University
Institute of Physics
Universitätsplatz 5
A-8010 Graz
Austria
Tel.: +43-(0)316-380-8564


Am 06.09.2017 03:59, schrieb Abderrahmane Reggad:

I have made calculations for 5 3d transition metal sulphides with the
method PBE+EECE and I have found that all the results are ok with the
exception of the vanadium sulphide.

I have made 3 calculations (nm, fm and afmI ) to determine the
magnetic state for all the compounds and I found them to have afm
state.

The problem is about the atomic magnetic moment of vanadium and for
the other results which are not reliable because there is one study
where they used the non magnetic and paramagnetic states with the same
meaning which is contrary to what said Pr Blaha and they mentionned
that they have found to be non magnetic without details.

You will find the study in the following link

https://www.researchgate.net/profile/J_Hafner/publication/216328826_Magnetism_and_magneto-structural_effects_in_transition-metal_sulfides/links/00463525eaa9e1230f00.pdf

Best regards
___
Wien mailing list
Wien@zeus.theochem.tuwien.ac.at
http://zeus.theochem.tuwien.ac.at/mailman/listinfo/wien
SEARCH the MAILING-LIST at:
http://www.mail-archive.com/wien@zeus.theochem.tuwien.ac.at/index.html

___
Wien mailing list
Wien@zeus.theochem.tuwien.ac.at
http://zeus.theochem.tuwien.ac.at/mailman/listinfo/wien
SEARCH the MAILING-LIST at:  
http://www.mail-archive.com/wien@zeus.theochem.tuwien.ac.at/index.html


Re: [Wien] About the magnetic moment of vanadium in vanadium sulphide

2017-09-05 Thread pieper

High A. Reggad,

substitute V for Cr in Prof. Fecher's questions, otherwise I am with 
him.


In case you refer by 'almost zero' to the 0.05 mu_B I seem to recall 
from your original question, and now wonder about some fundamental 
discrepancy between the supposed experimental Pauli paramagnetism and 
your supposed antiferromagnetism: Take a VERY close look at both, 
experiment AND calculations.


It is not trivial to experimentally distinguish a PM from an AF with 
such a small moment, especially if you have no idea that there is a Tc, 
let alone where it is. The, say T-dependant SQUID, measurement could 
start already in the AF state if Tc is above RT and never realize the 
state is already AF. Or maybe Tc is below the lowest T the experimental 
set-up could reach?


It is equally not trivial to establish an AF ground state with that 
precision from DFT. Did you consider MMI of V as convergence criterium? 
Did you check convergence to that precision with respect to RKMAX, 
k-mesh, ... ? Did you keep the k-mesh (symmetries) between your 
calculations of the PM and the AF? Did you force the AF structure in 
your calculation? Did you try what happens if you don't impose it? Are 
you shure about your structural data? Did you do structural relaxation? 
Does structural relaxation influence your result of a magnetic ground 
state? What about the influence of the xc-potential? Did you do eece 
with LSDA or with PBE-GGA or with ...?


And what about other physical properties? Maybe most important, is the 
stuff metallic/insulating in experiment/calculations?


In case your question is about this horrible violation of Hund's rules, 
I repeat my former suggestion: Take a close look at the assumptions 
these rules rely on. And never forget that any law and rule of physics 
is valid only within some domain more or less clearly defined by such 
assumptions.




---
Dr. Martin Pieper
Karl-Franzens University
Institute of Physics
Universitätsplatz 5
A-8010 Graz
Austria
Tel.: +43-(0)316-380-8564


Am 05.09.2017 07:45, schrieb Fecher, Gerhard:

About what moment are you talking,
the total magnetic moment or the magnetic moment of the Cr atoms ?

Did you start your EECE calculation from a regular GGA calculation
that had no magnetic moments at the Cr ?

Ciao
Gerhard

DEEP THOUGHT in D. Adams; Hitchhikers Guide to the Galaxy:
"I think the problem, to be quite honest with you,
is that you have never actually known what the question is."


Dr. Gerhard H. Fecher
Institut of Inorganic and Analytical Chemistry
Johannes Gutenberg - University
55099 Mainz
and
Max Planck Institute for Chemical Physics of Solids
01187 Dresden

Von: Wien [wien-boun...@zeus.theochem.tuwien.ac.at] im Auftrag von
Abderrahmane Reggad [jazai...@gmail.com]
Gesendet: Dienstag, 5. September 2017 00:29
An: wien@zeus.theochem.tuwien.ac.at
Betreff: Re: [Wien] About the magnetic moment of vanadium in vanadium 
sulphide


Thanks martin

Experimentally they found that the vanadium sulphide is a pauli
paramagnetic but I have found it to be antiferromagnetic like other
transition metal sulphides but the magnetic moment value equals almost
zero despite the fact that vanadium has 3 inpaired electrons.

Best regards
___
Wien mailing list
Wien@zeus.theochem.tuwien.ac.at
http://zeus.theochem.tuwien.ac.at/mailman/listinfo/wien
SEARCH the MAILING-LIST at:
http://www.mail-archive.com/wien@zeus.theochem.tuwien.ac.at/index.html

___
Wien mailing list
Wien@zeus.theochem.tuwien.ac.at
http://zeus.theochem.tuwien.ac.at/mailman/listinfo/wien
SEARCH the MAILING-LIST at:  
http://www.mail-archive.com/wien@zeus.theochem.tuwien.ac.at/index.html


Re: [Wien] About the magnetic moment of vanadium in vanadium sulphide

2017-09-02 Thread pieper
Be aware that Hund's rules do rest on certain assumptions about the 
relative strength of intra- and inter-atomic couplings. There are, after 
all, a lot of para- or diamagnetic materials around. Try to look that 
up. Is your Vanadium sulphide magnetically ordered from experiment?


In your case the 0.05 mu_B is probably an instance of the problems 
frequently encountered in strongly correlated systems: There are several 
states with very similar total energy, but with very different 
configurations of charges and spins. Search the mailing list for 
magnetic ordered states, or for orbital order.


The scf cycle of DFT will tend to stay near the configuration it started 
from. If you start from a standard non-magnetic configuration set by 
lstart it won't look at magnetically ordered states. Try to start the 
scf cycle from magnetic configurations - you can set them up in lstart - 
and compare the total energies. Be carefull to change only the magnetic 
starting configuration, NOT RMT's, RKMAX, K-mesh, crystal symmetry, 
whatever.


Note that the starting spin configurations you can set in lstart may 
well be not good enough. You may have to modify the density matrix of 
the problematic orbitals by hand and let the scf cycle dig itself in in 
that state by using the option -orbc. This also has been explained in 
posts in this mailing list before - perhaps most detailed by prof. Tran.


Good luck with your calculations

---
Dr. Martin Pieper
Karl-Franzens University
Institute of Physics
Universitätsplatz 5
A-8010 Graz
Austria
Tel.: +43-(0)316-380-8564


Am 02.09.2017 12:24, schrieb Abderrahmane Reggad:

Dear wien2k users

I have calculated the magnetic moment of  vanadium sulphide in NiAs
structure using the EECE méthod with alpha parameter equals to 0.05.

despite the fact that the number of inpariated electrons is equal to 3
I found that the magnetic moment of vanadium is equal to 0.05 MB which
is inconsistent with the prediction of Hund.

Best regards
___
Wien mailing list
Wien@zeus.theochem.tuwien.ac.at
http://zeus.theochem.tuwien.ac.at/mailman/listinfo/wien
SEARCH the MAILING-LIST at:
http://www.mail-archive.com/wien@zeus.theochem.tuwien.ac.at/index.html

___
Wien mailing list
Wien@zeus.theochem.tuwien.ac.at
http://zeus.theochem.tuwien.ac.at/mailman/listinfo/wien
SEARCH the MAILING-LIST at:  
http://www.mail-archive.com/wien@zeus.theochem.tuwien.ac.at/index.html


Re: [Wien] How to know if the M+2 has a high spin from calculation?

2017-08-28 Thread pieper
No, Wien2k probably won't 'determine alone which spin state is 
energetically more stable'.


States with different spin configurations (PM, FM, AF, ...) frequently 
have small differences in total energy but large differences in their 
electronic configuration. Due to this the scf cycle tends to converge in 
local minima associated with the starting configuration. Thats why you 
should set different spin configurations in lstart and see where the scf 
cycle leads you. The solution with lowest total energy is what you seek.


For your low/high spin configuration this might be enough. There are 
more complicated cases where you may have to guide the scf to the 
correct symmetry by populating the density matrices accordingly - this 
has been discussed in the mailing list a lot - see also option -orbc in 
the UG.


Good luck

Martin Pieper


---
Dr. Martin Pieper
Karl-Franzens University
Institute of Physics
Universitätsplatz 5
A-8010 Graz
Austria
Tel.: +43-(0)316-380-8564


Am 27.08.2017 13:06, schrieb Abderrahmane Reggad:

Hello again

I have found in the literature that the spin state configuration is
like the magnetic configuration. So we have to make 2 calculations:
one for the high spin configuration and another one for the low spin
configuration and we look after for the configuration more
energetically stable.

I want to know if the things are so in the wien2k code or it behaves
differently and can determine alone which spin state is the more
energetically stable.

Best regards
___
Wien mailing list
Wien@zeus.theochem.tuwien.ac.at
http://zeus.theochem.tuwien.ac.at/mailman/listinfo/wien
SEARCH the MAILING-LIST at:
http://www.mail-archive.com/wien@zeus.theochem.tuwien.ac.at/index.html

___
Wien mailing list
Wien@zeus.theochem.tuwien.ac.at
http://zeus.theochem.tuwien.ac.at/mailman/listinfo/wien
SEARCH the MAILING-LIST at:  
http://www.mail-archive.com/wien@zeus.theochem.tuwien.ac.at/index.html


Re: [Wien] How to simulate the ionic state of a compound?

2017-08-24 Thread pieper

High, Abderrahmane,


If I introduce the Ni atom and I change the Z number to 27 or 26 it
still nickel because the atom is ont only Z and I change Z the number
of neutrons and protons still inchanged.


NO! Z is the NUCLEAR charge. It sets the number of electrons 
automatically. If you change that you have another element! Try it by 
entering only the chemical symbol (e.g. Ni) in the corresponding field 
of the w2web struct editor. Leave the field Z empty. Press 'save and 
continue editing'. It will have filled in the correct Z automatically. 
One can circumvent this automatism, but it only makes sense if you know 
what you do. As I said before: find out yourself, start with a text book 
and the UG. You might learn enough in the process to know what you are 
doing.



An other question: We suppose that the ionic state of NiO is a result
from the electron density , can we force this ionic state to be
Ni+3S-3 and it's yes how?


Once again: You want to create some charge distribution of your own 
design where atom A lost electrons from and atom B caught them in what 
you decide is their own shell.


This is NOT what DFT or Wien2k does!

DFT gives you a (usually very good) approximation of where the electrons 
actually are in space. This may (in ionic crystals) or may not (with 
covalent or metallic bonds) agree more or less with your ideas.


Note that the agreement depends not only on the charge distribution 
itself! It heavily depends on how you decide by your experimental and/or 
computational methods which electron belongs where. Thats where the 
different definitions of charges Prof. Marks pointed out come from.


And try to get your hands on a good text book on solid state physics. It 
will help you far more than anyone in this mailing list can in his/her 
free time.


Best regards

Martin Pieper

---
Dr. Martin Pieper
Karl-Franzens University
Institute of Physics
Universitätsplatz 5
A-8010 Graz
Austria
Tel.: +43-(0)316-380-8564


Am 24.08.2017 02:23, schrieb Abderrahmane Reggad:

Thank you for all for the rich discussion

If I introduce the Ni atom and I change the Z number to 27 or 26 it
still nickel because the atom is ont only Z and I change Z the number
of neutrons and protons still inchanged.As I mentionned when I have
changed Z I have observed that the total energy changes.

An other question: We suppose that the ionic state of NiO is a result
from the electron density , can we force this ionic state to be
Ni+3S-3 and it's yes how?

Best regards

2017-08-24 0:43 GMT+01:00 Abderrahmane Reggad <jazai...@gmail.com>:


Hello

The reason for my question is that I don't how does a DFT code work
and I want to know if the ionic state is a result or an assumption.

Thank you for contribution to clarify the picture.

Best regards

___
Wien mailing list
Wien@zeus.theochem.tuwien.ac.at
http://zeus.theochem.tuwien.ac.at/mailman/listinfo/wien
SEARCH the MAILING-LIST at:
http://www.mail-archive.com/wien@zeus.theochem.tuwien.ac.at/index.html

___
Wien mailing list
Wien@zeus.theochem.tuwien.ac.at
http://zeus.theochem.tuwien.ac.at/mailman/listinfo/wien
SEARCH the MAILING-LIST at:  
http://www.mail-archive.com/wien@zeus.theochem.tuwien.ac.at/index.html


Re: [Wien] How to simulate the ionic state of a compound?

2017-08-23 Thread pieper
Not knowing what your average Fe knows I still would guess it probably 
doesn't. Maybe thats why nothing else changes when you relabel atoms in 
xcrysden. However, upon adding two protons (and some neutrons) in 
Wien2k, Fe seems to know it became Ni - we learned from the initial 
question that it says so through ETOT. ;-)


Best regards,

Martin Pieper

---
Dr. Martin Pieper
Karl-Franzens University
Institute of Physics
Universitätsplatz 5
A-8010 Graz
Austria
Tel.: +43-(0)316-380-8564


Am 23.08.2017 11:16, schrieb Fecher, Gerhard:

Funny discussion,
I wonder wether iron (Z=26) knows that it is Nickel just if one gives
it a new name ?


Ciao
Gerhard

DEEP THOUGHT in D. Adams; Hitchhikers Guide to the Galaxy:
"I think the problem, to be quite honest with you,
is that you have never actually known what the question is."


Dr. Gerhard H. Fecher
Institut of Inorganic and Analytical Chemistry
Johannes Gutenberg - University
55099 Mainz
and
Max Planck Institute for Chemical Physics of Solids
01187 Dresden

Von: Wien [wien-boun...@zeus.theochem.tuwien.ac.at] im Auftrag von
Víctor Luaña Cabal [vic...@fluor.quimica.uniovi.es]
Gesendet: Mittwoch, 23. August 2017 00:33
An: A Mailing list for WIEN2k users
Cc: Victor Luaña
Betreff: Re: [Wien] How to simulate the ionic state of a compound?

On Tue, Aug 22, 2017 at 11:41:53PM +0200, pieper wrote:

my response to Abderrahmane Reggad appears perhaps a little harsh. It
was not meant that way. I wanted to emphasize that in my view the idea
of telling Wien2k (or any other DFT program) its result (where the
electrons are) and to simulate properties from there is completely
backwards.


Martin,

It was not harsh at all in my perception, but I canot say what
Abderrahmane thought about your words.

I perceived as a normal exchange of ideas in a subject that I love.

I agree that using a questionable populaton technique to obtain a 
number

of electrons associated to an ion from a expensive DFT calculation
is not neccesarily a good idea. Providing a number to say this is a
Ni(+1.98) O(-2.02) crystal should not be the main result from a good
article. However, a good physically based theory of chemical bonding in
molecules, solids and clusters provides far more that that.

Let me be very particular in describing QTAIM (quantum theory of atoms
in molecules, aka Richard F. W. Bader et al work) as a beautiful and
well founded physical teory on chemical bonding.

Bader's QTAIM can be used to partition every property into 
contributions

of a particular atom or functional group.

Then you can find an explanation why many perovskite oxides, for
instance, have a very similar bulk modulus, and the explanation comes
from the fact that the oxide is the major contribution, and that
contribution is common to the incumbent crystals.

This is, however, what I wanted to point out: DFT (or Wien2k) tells 
you
where the electrons are. Thats its central result. It does not make 
any
sense (to me) to use a DFT program to - as A. Reggad put it - 
"simulate

the NiO compound in its ionic state". If NiO would be a ionic compound
then DFT would (hopefully, when set up properly) calculate an electron
density with a lot of weight at O and a lot less at Ni as a RESULT. 
The

simulation of any property one wishes to study can proceed from there.


I agree, of course. That should be the spirit of an ab initio 
calculation.

However, DFT sometimes separates from the ab initio family when the xc
functional is chosen not because of its properties or for exploring the
sensibility of the predictions to the functional but because a spurious
predictive agreement to whatever 'experiment'.

I believe finding explanations to peculiar facts is the real motive
of computational research, at least until the point is achieved at
which theoretical calculations become competitive with experimental
measurements in terms of precision, exactitude and cost.


And if the electron density of NiO does not really resemble the ionic
picture, why use the ionic model to simulate things?


Yes, but having a predictive measurement of the ionicity lets you
examine how ionicity depends on thermodynamic conditions: is the effect
of pressure more important than the effect of temperature or pH ... on
a geological scale?

*Chemical bonding* can be at the core of geophysics, research on
materials, ... and many more fields. However, I'm a peculiar chemist 
and

I love Physical Review since I was working on my phd thesis, long ago.

Best regards from sunny north Spain,
Víctor
--
.  ."Half of the US people use twitter to form its opinion and 
half
   / `' \   also elect the US president. I only hope they are not the 
same

  /(o)(o)\  half". --From a sentence by Gore Vidal
 /`. \/ .'\
/   '`'`   \ "[Technocrats, academics and journalists] are often 
motivated

|  \'`'`/  | by hope while the publi

Re: [Wien] How to simulate the ionic state of a compound?

2017-08-22 Thread pieper

High Victor,

my response to Abderrahmane Reggad appears perhaps a little harsh. It 
was not meant that way. I wanted to emphasize that in my view the idea 
of telling Wien2k (or any other DFT program) its result (where the 
electrons are) and to simulate properties from there is completely 
backwards.


Of course I agree with you that the electron density of a solid is a 
real observable - even in the XXI century. Quantum mechanics was 
invented to calculate such properties and the density is, after all, the 
D in DFT. I also agree that there are any number of solids where a ionic 
description makes perfect sense. I never tried but I am very confident 
that the ionic nature of alkali halides will be evident from the result 
of a DFT calculation.


This is, however, what I wanted to point out: DFT (or Wien2k) tells you 
where the electrons are. Thats its central result. It does not make any 
sense (to me) to use a DFT program to - as A. Reggad put it - "simulate 
the NiO compound in its ionic state". If NiO would be a ionic compound 
then DFT would (hopefully, when set up properly) calculate an electron 
density with a lot of weight at O and a lot less at Ni as a RESULT. The 
simulation of any property one wishes to study can proceed from there.


And if the electron density of NiO does not really resemble the ionic 
picture, why use the ionic model to simulate things?


Best regards,

Martin


---
Dr. Martin Pieper
Karl-Franzens University
Institute of Physics
Universitätsplatz 5
A-8010 Graz
Austria
Tel.: +43-(0)316-380-8564


Am 22.08.2017 20:45, schrieb Víctor Luaña Cabal:

On Tue, Aug 22, 2017 at 07:00:07PM +0200, pieper wrote:

DFT in general and Wien2k especially are there to tell you with
remarkably high precision what the charge distribution in a given
structure actually looks like. Thats FAR better than any hand-waving 
Ni
is 2+, O 2-. If you don't like what DFT tells you and want to draw 
some

fictitious ionic representation of NiO, take xcrysden, plot the
structure, and add labels saying that the (maybe) red circles are Ni2+
and the (maybe) blue circles are O2-.

It should be obvious to you from your lectures and readings on solid
state physics that the total energy depends on Z. If you change the
nuclear charge Z you change the element at that position. One can 
adjust
the number of electrons and their starting distribution in Wien2k, but 
I

plainly won't tell you how. You will learn much more if you find out
yourself - start with the User Guide and a solid state physics text
book.



martin,

Nothing against your description and I agree basically with
everything. However, the electron density (\rho) of a molecule or
solid is a real and mensurable property that exists in 3D real position
space, so the topology of \rho can be examined properly: that's Bader
topological analysis. A mostly physical and matematical description of
molecules and solids. Similarly, there are other scalar properties that
are physically well defined.

By Bader topological analysis we know that some systems are truly
ionic in nature, like alkali halides or simple oxides, but that is not
any surprise and crystallographers have used that since Laue and the
Braggs. Other materials are more interesting, however, and pressure
(thermodynamics, in general) modifies behavior of BP from being mostly
boron phosphide to phosporus boride, including a pressure range in 
which
electrons behave as independent chemical items and we have an 
electride.

[Phys. Rev. B  63 (2001) 125103, Polarity inversion in the electron
density of BP crystal, Paula Mori-Sánchez et al.]

In molecules, for instance, we have the Electrostatic potential as a 
well

defined scalar property. In some ways it brings us from XX century
(the century of quantum mechanics and electron density, quite complex
due to the effect of correlation and basis sets) to XIX century with
Maxwell and Faraday (positive and negative charge distributions are
what we need to know).

Unfortunately, in solids there is a real indetermination with the zero 
of

the EP, that it is basically arbitrary, as much as I know.


Nice subject, if you let me tell, by the way. Best regards,
   Víctor Luaña
--
.  ."Half of the US people use twitter to form its opinion and 
half
   / `' \   also elect the US president. I only hope they are not the 
same

  /(o)(o)\  half". --From a sentence by Gore Vidal
 /`. \/ .'\
/   '`'`   \ "[Technocrats, academics and journalists} are often 
motivated

|  \'`'`/  | by hope while the public at large tends to be more focused
|  |'`'`|  | on fear." -- Russell Mead (The Washington Post, 2017)
 \/`'`'`'\/
==(((==)))===+===
! Dr.Víctor Luaña, in silico chemist & prof. !
! Departamento de Química Física y Analítica !
! Universidad de Oviedo, 33006-Oviedo, Spain !
! e-mail:  <vic...@fluor.quimica.uniovi.es>  !
! ph

Re: [Wien] How to simulate the ionic state of a compound?

2017-08-22 Thread pieper
DFT in general and Wien2k especially are there to tell you with 
remarkably high precision what the charge distribution in a given 
structure actually looks like. Thats FAR better than any hand-waving Ni 
is 2+, O 2-. If you don't like what DFT tells you and want to draw some 
fictitious ionic representation of NiO, take xcrysden, plot the 
structure, and add labels saying that the (maybe) red circles are Ni2+ 
and the (maybe) blue circles are O2-.


It should be obvious to you from your lectures and readings on solid 
state physics that the total energy depends on Z. If you change the 
nuclear charge Z you change the element at that position. One can adjust 
the number of electrons and their starting distribution in Wien2k, but I 
plainly won't tell you how. You will learn much more if you find out 
yourself - start with the User Guide and a solid state physics text 
book.




---
Dr. Martin Pieper
Karl-Franzens University
Institute of Physics
Universitätsplatz 5
A-8010 Graz
Austria
Tel.: +43-(0)316-380-8564


Am 22.08.2017 12:19, schrieb Abderrahmane Reggad:

Dear wien users

I want to simulate the NiO compound in its ionic state. We know that
the Ni and O atoms have in their neutral state the atomic numbers 28
and 8 respectively.

Now, if we want to study the ionic state of the compound Ni+2O-2,
should we modify the atomic numbers to Z=26 for Ni and Z=10 for O, or
the wien2k code do the operation instead of us?

NB: When I change the Z number for Ni atom I found a change in the
energy value.

Best regards
___
Wien mailing list
Wien@zeus.theochem.tuwien.ac.at
http://zeus.theochem.tuwien.ac.at/mailman/listinfo/wien
SEARCH the MAILING-LIST at:
http://www.mail-archive.com/wien@zeus.theochem.tuwien.ac.at/index.html

___
Wien mailing list
Wien@zeus.theochem.tuwien.ac.at
http://zeus.theochem.tuwien.ac.at/mailman/listinfo/wien
SEARCH the MAILING-LIST at:  
http://www.mail-archive.com/wien@zeus.theochem.tuwien.ac.at/index.html


Re: [Wien] Questions about Na2IrO3 (PBE+SOC+U calculation with magnetic order)

2017-08-07 Thread pieper
Of course! The local moments in an AF insulator can be anything. I just 
saw that there is only one type of magnetic atom in the unit cell (Ir) 
and did not think properly. Sorry for that one.


---
Dr. Martin Pieper
Karl-Franzens University
Institute of Physics
Universitätsplatz 5
A-8010 Graz
Austria
Tel.: +43-(0)316-380-8564


Am 07.08.2017 09:18, schrieb Peter Blaha:

The Ir moments are read from case.scf file, but why should it be near
an integer, and what is the unit of the reported moments in case.scf
file? If the unit is bohr magneton, then for a spin 1/2, should it be 
~1.7?


For a FERROmagnetic insulator, the TOTAL spin moment/cell must be
integer. The moments of individual atoms can have any value.

For an ANTIferromagnet, the total spin moment/cell must be zero, and
of course the atomic moments can again have any value.

What you get by :MMT or MMIxxx are spin moments only.

The moments are given in Bohr magnetons (mu_B), but are the spin
moments, not the effective moments. Thus if you have one unpaired
electron the spin moment is one.

PS: For your compound, the orbital moments are probably as important
as the spin moments and you must add them to get the total moment.





The gap can be roughly seen along the k-path I plotted and is checked 
by

DOS plot, which shows it's an insulator.

Yours sincerely,

Hung-Yu

On Sun, Aug 6, 2017 at 11:03 AM, pieper <pie...@ifp.tuwien.ac.at
<mailto:pie...@ifp.tuwien.ac.at>> wrote:

A few thoughts on this:

ad 1)
Presumably Ir is the magnetic ion, so what you describe seems to 
be

ok with me. I did not check your .struct file, however.

ad 2)
See chapters 4.5.6 and 7.4 of the UG (Wien2k 17.1) on ther
orb-program, and the references therein. Personally I found the 
.pdf
file of his talk on LDA+U very usefull that E. Assmann posted on 
the

Wien2k-site.

Note the recommendation in the UG for the SIC-mode of orb for of
strongly correlated systems: set J=0 and use only U_eff=U-J. The
value of U_eff is something you will have to decide, perhaps based
on the approach described by Madsen and Novak cited in the UG. You
also might want to take a look at eece as an alternative (UG 
chapter

4.5.7)

>From many comments here in the mailing list and from the UG 
(again
e.g. chapter 5.5.7, 4.5.8) I take it that PBE is what you should 
do
if you want to calculate spacial charge and spin distributions, 
but

to calculate gaps you may have to switch to numerically much more
costly hybrid methods.

You probably can (mis)use U_eff as a free parameter to adjust the
gap in your PBE calculation to your favorite value. However, the
physical meaning of the value would be dubious (imho), and there 
is

no guarantee that the Ir-moments simultaneously come near your
favorit 'theoretical moments' (whatever the actual value and 
origin

of those is).

ad 3) Don't bother with the starting values of local moments for
atomic configurations, and for an antiferromagnet the interstitial
moment obviously should stay close to zero.

But the moments you give in your table are very far from integer.
Are this Ir-spin moments from  case.scf or did you add orbital
moemnts calculated by lapwdm? The spin moments should be somewhere
near integer for an insulator.

So, how did you determine the gap in the table? Did you plot a 
DOS?

Is this really an insulator, or are there in fact bands crossing
E_F? You might severly misjudge the (direct?) gap depending on 
where

in k-space it is and the points in your k-list.

Good luck

---
Dr. Martin Pieper
Karl-Franzens University
Institute of Physics
Universitätsplatz 5
A-8010 Graz
Austria
Tel.: +43-(0)316-380-8564 <tel:%2B43-%280%29316-380-8564>



Am 04.08.2017 19:22, schrieb Hung Yu Yang:

Dear WIEN2k developers and users,

I am trying to do a calculation on Na2IrO3, which has a band 
gap

~340meV and a zigzag antiferromagnetic order in its ground
state, and
I have some questions as follow:

1. To assign the zigzag antiferromagnetic order, what I did is
that I
first made a cif file that has two inequivalent Ir atoms, and 
let

WIEN2k decide the symmetry for me. I adopted the generated
structural
file (attached in this mail), checked the cif file in some
visulization software and made sure they were structurally
equivalent
except that there were 2 inequivalent Ir atoms instead of 1. 
Is this
the proper way to generate structural files for magnetically 
ordered

systems?

2. After generating the desired structural file, I put up on 
Ir1

atom
and down on the other (Ir2), used several different 
combinations

of U
and J, and I got the following results:

U(eV) J(eV)

Re: [Wien] Questions about Na2IrO3 (PBE+SOC+U calculation with magnetic order)

2017-08-06 Thread pieper

A few thoughts on this:

ad 1)
Presumably Ir is the magnetic ion, so what you describe seems to be ok 
with me. I did not check your .struct file, however.


ad 2)
See chapters 4.5.6 and 7.4 of the UG (Wien2k 17.1) on ther orb-program, 
and the references therein. Personally I found the .pdf file of his talk 
on LDA+U very usefull that E. Assmann posted on the Wien2k-site.


Note the recommendation in the UG for the SIC-mode of orb for of 
strongly correlated systems: set J=0 and use only U_eff=U-J. The value 
of U_eff is something you will have to decide, perhaps based on the 
approach described by Madsen and Novak cited in the UG. You also might 
want to take a look at eece as an alternative (UG chapter 4.5.7)


From many comments here in the mailing list and from the UG (again e.g. 
chapter 5.5.7, 4.5.8) I take it that PBE is what you should do if you 
want to calculate spacial charge and spin distributions, but to 
calculate gaps you may have to switch to numerically much more costly 
hybrid methods.


You probably can (mis)use U_eff as a free parameter to adjust the gap in 
your PBE calculation to your favorite value. However, the physical 
meaning of the value would be dubious (imho), and there is no guarantee 
that the Ir-moments simultaneously come near your favorit 'theoretical 
moments' (whatever the actual value and origin of those is).


ad 3) Don't bother with the starting values of local moments for atomic 
configurations, and for an antiferromagnet the interstitial moment 
obviously should stay close to zero.


But the moments you give in your table are very far from integer. Are 
this Ir-spin moments from  case.scf or did you add orbital moemnts 
calculated by lapwdm? The spin moments should be somewhere near integer 
for an insulator.


So, how did you determine the gap in the table? Did you plot a DOS? Is 
this really an insulator, or are there in fact bands crossing E_F? You 
might severly misjudge the (direct?) gap depending on where in k-space 
it is and the points in your k-list.


Good luck

---
Dr. Martin Pieper
Karl-Franzens University
Institute of Physics
Universitätsplatz 5
A-8010 Graz
Austria
Tel.: +43-(0)316-380-8564


Am 04.08.2017 19:22, schrieb Hung Yu Yang:

Dear WIEN2k developers and users,

I am trying to do a calculation on Na2IrO3, which has a band gap
~340meV and a zigzag antiferromagnetic order in its ground state, and
I have some questions as follow:

1. To assign the zigzag antiferromagnetic order, what I did is that I
first made a cif file that has two inequivalent Ir atoms, and let
WIEN2k decide the symmetry for me. I adopted the generated structural
file (attached in this mail), checked the cif file in some
visulization software and made sure they were structurally equivalent
except that there were 2 inequivalent Ir atoms instead of 1. Is this
the proper way to generate structural files for magnetically ordered
systems?

2. After generating the desired structural file, I put up on Ir1 atom
and down on the other (Ir2), used several different combinations of U
and J, and I got the following results:

U(eV) J(eV) Ueff=U-J (eV)  Moment(Ir1, Ir2)
  Gap(meV)
2.1 0.6 1.5
0.22532,-0.22439~700meV
2.4 02.4
0.30105,-0.30109~750meV
30.6 2.4
0.23225,-0.23235~900meV
31.5 1.5
0.17203,-0.17210~900meV

First, in this test, it seems that the gap is affected by U only, not
Ueff=U-J. Second, the moments of Ir seemed to be closely related to J.
In this situation, what is the proper way to assign U and J? I am not
sure how much it means if I just try to tune U and J until they match
the experimental gap and/or theoretical moments.

3. I also have a question about the unit of moment given here; I
understand that the moments depend on the RMT sizes and I assumed the
unit is in bohr magneton, but the first value shown for Ir atom is
0.76697 and I am not sure in what unit is this value and how it
assigns the initial moment for a certain element (say, Ir in my case).
Also, the interstitial moments are nearly 0 among all the cycles.

For this calculation, kpoints=450, RKmax=8 and I had to use the TEMP
scheme for it to converge. I appreciate any reply from you.

Yours sincerely,

Hung-Yu
___
Wien mailing list
Wien@zeus.theochem.tuwien.ac.at
http://zeus.theochem.tuwien.ac.at/mailman/listinfo/wien
SEARCH the MAILING-LIST at:
http://www.mail-archive.com/wien@zeus.theochem.tuwien.ac.at/index.html

___
Wien mailing list
Wien@zeus.theochem.tuwien.ac.at
http://zeus.theochem.tuwien.ac.at/mailman/listinfo/wien
SEARCH the MAILING-LIST at:  
http://www.mail-archive.com/wien@zeus.theochem.tuwien.ac.at/index.html


Re: [Wien] Spin part of the magnetic susceptibility

2017-07-21 Thread pieper

There have been several comments on this.

I can only guess at what exactly you want to calculate, but to me it 
seems that calculating a (zero-field) spin-resolved DOS would be a good 
start. Then you shift the spin-up and spin-down parts rigidly by mu_B*B 
by hand, and count how many mu_B you get per Tesla field.


If you want/need anything more elaborate you will have to provide more 
information on what kind of measurement in what kind of ferromagnetic 
metal or semiconductor you actually are interested in (and/or read 
Ashcroft & Mermin).



---
Dr. Martin Pieper
Karl-Franzens University
Institute of Physics
Universitätsplatz 5
A-8010 Graz
Austria
Tel.: +43-(0)316-380-8564


Am 21.07.2017 13:33, schrieb Wien2k User:

Dear Wien2k Users;I am still waiting for an answer from Prof P.Blaha
or Wien2k users

2017-07-20 2:03 GMT+02:00 Wien2k User <wien2k.u...@gmail.com>:


dear wien2k users;

How to calculate the spin part of the magnetic susceptibility for a
ferromagnetic metal since the magnetic moment is different from
zero.

___
Wien mailing list
Wien@zeus.theochem.tuwien.ac.at
http://zeus.theochem.tuwien.ac.at/mailman/listinfo/wien
SEARCH the MAILING-LIST at:
http://www.mail-archive.com/wien@zeus.theochem.tuwien.ac.at/index.html

___
Wien mailing list
Wien@zeus.theochem.tuwien.ac.at
http://zeus.theochem.tuwien.ac.at/mailman/listinfo/wien
SEARCH the MAILING-LIST at:  
http://www.mail-archive.com/wien@zeus.theochem.tuwien.ac.at/index.html


Re: [Wien] magnetic susceptibility for a ferromagnetic metal or for a ferromagnetic semiconductor

2017-07-21 Thread pieper
Note that the internal field in a metal (or an insulator) is not 
homogeneous. This is because of the inhomogeneous distribution of 
electrons. You don't have a homogeneous distribution of induced magnetic 
moments, and with that the susceptibility also is inhomogenous. chi is 
chi(r), a function of where in the material you do your measurement.


The property of metals calculated with very impressive accuracy by 
Laskowski and Blaha and their NMR module of Wien2k is this LOCAL 
susceptibility at the position of the nuclei, chi(r=r_atom1), ... 
chi(r=r_atomn). This is what you measure in nuclear magnetic resonance, 
NMR, hence the name of the module.


I guess the 'molar magnetic susceptibility' you want to calculate is 
what you get if you put the material in a (perhaps SQUID-) magnetometer: 
The average of chi(r) over the whole sample volume.


From Peter Blahas comments here I take it that one could calculate this 
average on basis of the results from the NMR module: The spin and 
current distributions are there. My guess is that it will be not as 
trivial as it might look at first sight. For example I seem to recall 
that he mentioned that this is difficult to converge - at least in 
normal metals its a small effect, after all. This is not meant to 
discourage trying, just expect problems.


The NMR module is, however, for 'normal' metals. My warning was about 
high expectations in calculating the macroscopic, homogeneous 
susceptibility of ferromagnets (or other magnetically oŕdered 
materials). Wien2k gives invaluable information about the size of local 
magnetic moments, their exchange energies, moments of itinerant 
electrons in a band structure, ... with the caveats mentioned earlier 
even on the influence of a magnetic field. However, the susceptibility 
is about how the applied field couples the ground state determined by 
DFT to any low lying excited states - and especially in magnetically 
ordered systems these excited states might well be beyond the scope of 
DFT.



---
Dr. Martin Pieper
Karl-Franzens University
Institute of Physics
Universitätsplatz 5
A-8010 Graz
Austria
Tel.: +43-(0)316-380-8564


Am 20.07.2017 18:12, schrieb karima Physique:

Thank you very much for your detailed answer

What I understood from what you wrote is that the DFT does not
accurately estimate the magnetic susceptibility. honestly what is
encouraged me to take an interest in this property is the paper of
Prof. Robert Laskowski and Prof Peter Blaha  (doi:
10.1021/acs.jpcc.5b05947 [3]) where they compared their results for
many metals and found values very similar to the experimental ones.

Now I do not know is what the estimate of molar magnetic
susceptibility is possible with wien2k or its estimate is not always
accurate.

2017-07-20 16:54 GMT+02:00 pieper <pie...@ifp.tuwien.ac.at>:


From what I have understood from userguide and Prof P Blaha's
replies;
For semiconductor and insulator; there is the orbital part of the
magnetic susceptibility only
but for the metals there is also the spin part


 No! This is not correct. Let me expand a little on my view of this
topic in the hope to clarify the problem at least somewhat:

In a nutshell, you want Wien2k to find the state minimizing the total
energy of some solid because you suspect that this is the state whose
properties you observe in some experiment, and someone told you that
DFT is the method of choice to do the search.

Magnetic moments contribute M*B to that energy with M and B being
vectors in a scalar product. In a first approximation two
contributions from electrons to M can be considered separately, namely
the spin moment, and the moment from an orbital momentum (or the
current associated with a non-zero orbital momentum). Both parts
contribute to the energy in the field. That is why Peter Blahas very
first statement was:

A magnetic field influences the spin and orbital degrees of freedom.

It always does. Both contributions to M. No matter wether you have an
insulator or a metal. The influence on specific properties you are
interested in may be too small to bother, but somewhere else it will
show up.

There are different ways to tell Wien2k where to look for the state
with minimum energy in an applied magnetic field (or electric field).
This is where symmetry enters stage: Wien2k searches the state with
lowest energy in the subspace compatible with the symmetries specified
by the input files. The symmetry of the crystal structure is in the
struct file. Concerning magnetic moments you can tell Wien2k to

- consider only states that are symmetric with respect to spin, i.e.
do a non-spin-polarized calculation. Doesn't make sense if you are
interested in the effect of a magnetic field or want to model
ferromagnets, but for the majority of materials and properties it's
perfect to save time and calculate just spin +1/2, then carry the
result over to spin -1/2.

- take into account the two spin channels separately, i.e. do a
spin-polarized calcu

Re: [Wien] magnetic susceptibility for a ferromagnetic metal or for a ferromagnetic semiconductor

2017-07-20 Thread pieper



From what I have understood from userguide and Prof P Blaha's 
replies;
For semiconductor and insulator; there is the orbital part of the 
magnetic susceptibility only

but for the metals there is also the spin part


No! This is not correct. Let me expand a little on my view of this topic 
in the hope to clarify the problem at least somewhat:


In a nutshell, you want Wien2k to find the state minimizing the total 
energy of some solid because you suspect that this is the state whose 
properties you observe in some experiment, and someone told you that DFT 
is the method of choice to do the search.


Magnetic moments contribute M*B to that energy with M and B being 
vectors in a scalar product. In a first approximation two contributions 
from electrons to M can be considered separately, namely the spin 
moment, and the moment from an orbital momentum (or the current 
associated with a non-zero orbital momentum). Both parts contribute to 
the energy in the field. That is why Peter Blahas very first statement 
was:


A magnetic field influences the spin and orbital degrees of freedom.

It always does. Both contributions to M. No matter wether you have an 
insulator or a metal. The influence on specific properties you are 
interested in may be too small to bother, but somewhere else it will 
show up.


There are different ways to tell Wien2k where to look for the state with 
minimum energy in an applied magnetic field (or electric field). This is 
where symmetry enters stage: Wien2k searches the state with lowest 
energy in the subspace compatible with the symmetries specified by the 
input files. The symmetry of the crystal structure is in the struct 
file. Concerning magnetic moments you can tell Wien2k to


- consider only states that are symmetric with respect to spin, i.e. do 
a non-spin-polarized calculation. Doesn't make sense if you are 
interested in the effect of a magnetic field or want to model 
ferromagnets, but for the majority of materials and properties it's 
perfect to save time and calculate just spin +1/2, then carry the result 
over to spin -1/2.


- take into account the two spin channels separately, i.e. do a 
spin-polarized calculation, and introduce B as 'orbital potential' in 
the program orb. Note that a field you put into .inorb acts on the 
electrons in the single atomic shell you specify, and only for the part 
within the muffin-tin radius. Everything else is indirectly adjusted 
according to interactions. I am (of course) with Peter Blaha: One can 
expect very reasonable estimates for field induced moments at the given 
shell, at least if the field induced polarization of electrons not 
affected by .inorb is small. The option also shows a Zeeman splitting of 
spin-up and -down bands due to the interactions, but I expect that 
depending on the case this can rapidly turn into a paedagogical example 
rather than an estimate of the spin susceptibility.


- take into account the fact that the energy contribution M*B quite 
possibly reduced symmetry. M as well as B are vectors, after all, and 
transform as such under symmetry operations. You can switch on 
spin-orbit interaction to tell Wien2k about this. (Be aware that Wien2k 
always assumes that M and B are colinear). As Gerhard Fecher pointed 
out, a reduced symmetry in a magnetic field is NOT taken into account in 
the orb program. With only the orb option active Wien2k may search the 
needle (the energy minimum) in a completely wrong haystack (subspace).


- do a calculation of the linear response of the all-electron wave 
function (representing spins AND orbital moments) to an applied field. 
As I understand it this is what the NMR module does. However, it 
calculates only the local response at the nuclei, not the integrated 
macroscopic susceptibility.


Note that all this does not represent anything like the macroscopic 
susceptibility you will detect with a ferromagnetic (or any other 
magnetically ordered) material in your magnetometer. The susceptibility 
in these cases even at zero Kelvin (where DFT arguably applies) is 
dominated by things the spins can and will do, but Wien2k is completely 
unaware of (magnons, domain walls ...).


Finally, don't take the UG or this mailing list as a substitute for a 
textbook. That is not their intention. Follow the advice of Prof. Fecher 
and read up on the subject of magnetism in solids. Perhaps start with 
something on solid state physics in general, not a specialized treatment 
of magnetism. Personally I like the introduction to Solid State Physics 
by Ashcroft and Mermin.


Good luck

---
Dr. Martin Pieper
Karl-Franzens University
Institute of Physics
Universitätsplatz 5
A-8010 Graz
Austria
Tel.: +43-(0)316-380-8564


Am 19.07.2017 16:02, schrieb karima Physique:

From what I have understood from userguide and Prof P Blaha's replies;
For semiconductor and insulator; there is the orbital part of the
magnetic susceptibility only but for the metals there is also the spin
part and I

Re: [Wien] question

2017-07-14 Thread pieper

That is VERY little information you give!

I suspect you are an inexperienced user trying to work from the command 
line without having worked at least through the examples of the user 
guide. You don't tell which program you started but the message tells 
you that its the wrong program: it has no -orb and -dm switches.


I suggest you use the w2web interface and the examples in the UG to make 
yourself more familiar with the package ...


Good luck


---
Dr. Martin Pieper
Karl-Franzens University
Institute of Physics
Universitätsplatz 5
A-8010 Graz
Austria
Tel.: +43-(0)316-380-8564


Am 14.07.2017 16:10, schrieb Dj Fati:

Could you please suggest an answer to my question
When i do a calculation on the method GGA+U it gives me error
option-orb does not exixt
option- dm does not exist
thanks
___
Wien mailing list
Wien@zeus.theochem.tuwien.ac.at
http://zeus.theochem.tuwien.ac.at/mailman/listinfo/wien
SEARCH the MAILING-LIST at:
http://www.mail-archive.com/wien@zeus.theochem.tuwien.ac.at/index.html

___
Wien mailing list
Wien@zeus.theochem.tuwien.ac.at
http://zeus.theochem.tuwien.ac.at/mailman/listinfo/wien
SEARCH the MAILING-LIST at:  
http://www.mail-archive.com/wien@zeus.theochem.tuwien.ac.at/index.html


Re: [Wien] Questions about imposing external magnetic field on no-magnetic system

2017-07-11 Thread pieper

In case no one has answered this up to now:

ad 1) The procedure itself is ok. You might want switch on SO first and 
converge that without the orbital potential to establish a zero-field 
base line. Remember to put in LARGE fields - your off-the-shelf lab 
field of 10 T will not show up at any energy precision you can achieve. 
Estimate the energy of 1 mu_B in 10 T field in Ry units to see that.


Note that your not-so-recent version of Wien2k is not the best for the 
task. The latest version is 17.1. With 16.1 came the NMR package which 
should be much better suited to calculate the effects of a magnetic 
field.


ad 2) If you apply a magnetic field experimentally in the lab you do it 
at all atoms. I suppose you want to model that situation. imho it makes 
little sense to exempt one or two of your atoms from the field.


Good luck

---
Dr. Martin Pieper
Karl-Franzens University
Institute of Physics
Universitätsplatz 5
A-8010 Graz
Austria
Tel.: +43-(0)316-380-8564


Am 10.07.2017 12:20, schrieb Peng Bingrui:

Dear professor Blaha and WIEN2K users

I'm running WIEN2K of 14 version on Linux system. I'm going to impose
external magnetic field on LaPtBi, a no-magnetic material. The
procedure that I'm going to use is :

1、Do a no-SO calculation : runsp_c_lapw.

2、Do a SO calculation : runsp_c_lapw -so -orb, while including
external magnetic field as orbital potential in the same time.

My questions are:

1、Whether this procedure is OK ? If it is not OK, what is the right
one ?

2、Which atoms and which orbitals should I treat with orbital
potential ? The electron configurations of these 3 atoms are: La (5d1
6s2) ;  Pt  (4f14 5d9 6s1); Bi (4f14 5d10 6s2 6p3).

Thanks very much for your attention.

Sincerely yours,

Bingrui Peng

from the Department of Physics, Nanjing University, China
___
Wien mailing list
Wien@zeus.theochem.tuwien.ac.at
http://zeus.theochem.tuwien.ac.at/mailman/listinfo/wien
SEARCH the MAILING-LIST at:
http://www.mail-archive.com/wien@zeus.theochem.tuwien.ac.at/index.html

___
Wien mailing list
Wien@zeus.theochem.tuwien.ac.at
http://zeus.theochem.tuwien.ac.at/mailman/listinfo/wien
SEARCH the MAILING-LIST at:  
http://www.mail-archive.com/wien@zeus.theochem.tuwien.ac.at/index.html


[Wien] minor problems in 17.1 with w2web and siteconfig_lapw

2017-07-10 Thread pieper



Installation of Wien2k 17.1 went very smoothely, thanks a lot for the 
new siteconfig_lapw! However, I encountered a small problem when I 
wanted to point the linker to the correct LIBXC dir: Make of my LIBXCD 
Version (dowmnloaded June.2017) produced a dir structure 
(LIBXCrootpath)/lib64 and /include, while siteconfig_lapw has a 
hard-wired (LIBXCrootpath)/lib and /include. When I finally figured this 
out it was easy to change the dir name from /lib64 to /lib, but ...


And w2web gives me difficulties with save_lapw: there is no way to the 
default save_lapw (all input files, clms etc). If all option boxes are 
left untouched it uses the last possibility in the list (I forgot which 
one that is). save_lapw from the terminal works just fine.


Best regards,

Martin Pieper

---
Dr. Martin Pieper
Karl-Franzens University
Institute of Physics
Universitätsplatz 5
A-8010 Graz
Austria
Tel.: +43-(0)316-380-8564

___
Wien mailing list
Wien@zeus.theochem.tuwien.ac.at
http://zeus.theochem.tuwien.ac.at/mailman/listinfo/wien
SEARCH the MAILING-LIST at:  
http://www.mail-archive.com/wien@zeus.theochem.tuwien.ac.at/index.html


Re: [Wien] Questions in "initso_lapw" when using "runsp_c_lapw" to run my task

2017-07-10 Thread pieper
A Hubbard U does not make much sense in a non spin polarized calculation 
- and I seem to recall that this is mentioned in the UG.


initso_lapw is, on the other hand, for spin-orbit interaction - there 
the question if you have a spin polarized case makes sense. This is also 
explained in the UG.


So the general advise seems to be stiduy the UG - and if possible some 
of the references therein.


Good luck




---
Dr. Martin Pieper
Karl-Franzens University
Institute of Physics
Universitätsplatz 5
A-8010 Graz
Austria
Tel.: +43-(0)316-380-8564


Am 08.07.2017 17:08, schrieb Peng Bingrui:

Dear wien2k users

I have a question that I'd like to discuss with you.

My purpose is to use LDA+U to treat no-magnetic system. So I use
"runsp_c_lapw -orb" to run my task, as told by the user_guide. And in
"initso_lapw", it asks me "do you have a spinpolarized case ?" and "do
you want to use the new structure for SO calculations ?". I answer
"Yes" to both the two quetions. I'm not very sure whether what I do is
right. I'd like to listen your experience and advices about this.

Thank you very much for your attention.

Bingrui Peng

from the Department of Physics,Nanjing University,China
___
Wien mailing list
Wien@zeus.theochem.tuwien.ac.at
http://zeus.theochem.tuwien.ac.at/mailman/listinfo/wien
SEARCH the MAILING-LIST at:
http://www.mail-archive.com/wien@zeus.theochem.tuwien.ac.at/index.html

___
Wien mailing list
Wien@zeus.theochem.tuwien.ac.at
http://zeus.theochem.tuwien.ac.at/mailman/listinfo/wien
SEARCH the MAILING-LIST at:  
http://www.mail-archive.com/wien@zeus.theochem.tuwien.ac.at/index.html


Re: [Wien] Convergence problem in mbj potential calculation

2017-06-15 Thread pieper
You keep getting back to this match between the (presumably powder-)xrd 
you calculated from your .struct file and the experimental one. This 
does NOT imply that your structural model describes the physical 
situation correctly! With your - in Gerhards words - 'interesting 
approach' you should be able to produce any number of different .struct 
files, and calculating their powder-xrd find a similar 'match with the 
experimental one'.


Did you calculate the xrd with any of the alternative distributions of 
Li, Ni and Ti on their sites in your unit cell? Did you try the same 
with the many, many other distributions you can generate in larger 
supercells? I am prettty sure you will find something that matches your 
xrd data. It will make as much sense as what you have now ...


Be aware of the fact that structural models of Wien2k have symmetry 
properties  different from your physical situation: in your model the 
metal atoms occupy lattice sites regularly, breaking certain symmetries 
of the underlying lattice. The actual distribution in your case is 
(apparently more or less) random, which overall breaks no symmetry, but 
locally breaks every symmetry somewhere.


If the distribution of Li, Ni, and Ti  really is random on one shared 
site I know of only one (expensive) way to proceed: try to reduce the 
influence of short range periodicity in your models by increasing the 
size of the supercell and putting Li, Ni, and Ti at random positions. Do 
structural relaxations, look what changes in your (GGA-) calculations, 
and keep fingers crossed that things you are interested in converge for 
supercells of a size you still can handle.




---
Dr. Martin Pieper
Karl-Franzens University
Institute of Physics
Universitätsplatz 5
A-8010 Graz
Austria
Tel.: +43-(0)316-380-8564


Am 15.06.2017 07:12, schrieb shamik chakrabarti:

Dear Gerhard,

You are right. The system can be written as
(Li0.5Ti0.25Ni0.25)O.

However, my mbj simulation is converged with MSR1 mixing in 221
cycles. And the DOS has come as metallic in this case too. Now, I may
try -eece also. So far, in GGA, GGA+U and in mbj the DOS has come as
metallic. However, if the structure is wrong it will not give the
correct DOS in any case. Although, the xrd is matched with
experimental one, there is an issue of longer Ti-O bond. I have to see
how can I get the correct structure.

Any suggestion in this regard will be helpful for us.

with regards,

with regards,

On Wed, Jun 14, 2017 at 8:00 PM, Fecher, Gerhard <fec...@uni-mainz.de>
wrote:


Interesting approach
as in Li2NiTiO4 the Li, Ni, and Ti atoms are randomly distributed on
the 4a site with oxygen on 4b of the NaCl lattice
therefore one better should write (Li0.5Ti0.25Ni0.25)O

Ciao
Gerhard

DEEP THOUGHT in D. Adams; Hitchhikers Guide to the Galaxy:
"I think the problem, to be quite honest with you,
is that you have never actually known what the question is."


Dr. Gerhard H. Fecher
Institut of Inorganic and Analytical Chemistry
Johannes Gutenberg - University
55099 Mainz
and
Max Planck Institute for Chemical Physics of Solids
01187 Dresden

Von: Wien [wien-boun...@zeus.theochem.tuwien.ac.at] im Auftrag von
shamik chakrabarti [shamik...@gmail.com]
Gesendet: Mittwoch, 14. Juni 2017 15:39
An: A Mailing list for WIEN2k users
Betreff: Re: [Wien] Convergence problem in mbj potential calculation

Dear Gerhard & Laurence,

Thank you very much for your
reply & suggestions. The structure was initially of Li2NiTiO4. The
space group is Fm-3m. We have created a 1x1x1 supercell (with target
lattice primitive) and then distributed Li, Ni & Ti with 2:1:1
ration at 4b site. 4a site contains oxygen. Then we have optimize
the volume and force per atom of the structure sequentially &
results in the present structure. The xrd generated by the present
structure matches exactly with the experimental xrd of Li2NiTiO4.

However, we have shifted the mixing procedure from PORT to MSR1 and
the simulation is converging well now.

with regards,

On Wed, Jun 14, 2017 at 3:17 PM, Laurence Marks
<l-ma...@northwestern.edu<mailto:l-ma...@northwestern.edu>> wrote:
I have said this before, but it is worth repeating: the most common
reason a calculation does not converge is because the
structure/model is unphysical/unchemical. Your Ti-O bonds are too
long, the structure is "obviously" wrong.

Scientific rigor is not optional. What is the R-factor? Are the
temperature factors sane? Is there an independent​ measurement of
the composition, e.g. ICP? Have you measured that it is FM?

---
Professor Laurence Marks
"Research is to see what everybody else has seen, and to think what
nobody else has thought", Albert Szent-Gyorgi
http://www.numis.northwestern.edu [1]
Corrosion in 4D http://MURI4D.numis.northwestern.edu [2]
Partner of the CFW 100% gender equity project,
www.cfw.org/100-perce

Re: [Wien] Import error, no module named numpy

2017-05-22 Thread pieper


The Traceback error message is from python. Its trying to do what 
berrypi tells it to do, so module berrypi is found and probably 
installed ok.


However, berrypi wants python to import numpy - and that is not found. 
Most probably something went wrong when you installed numpy (or 
mathplotlib?).


Try in a terminal to run python and import the module manualy. Enter at 
your user prompt:


user-prompt:~> python
Python 2.7.12 (default, Jun 28 2016, 06:57:42) [GCC] on linux2
Type "help", "copyright", "credits" or "license" for more information.

import numpy # math library



This probably will not work as shown here for my terminal, meaning that 
numpy is either not installed, or in some unexpected place and you will 
have to point python to that place. You will have to look into what you 
did to install numpy (or a  module including it, like mathplotlib).


Martin Pieper


---
Dr. Martin Pieper
Karl-Franzens University
Institute of Physics
Universitätsplatz 5
A-8010 Graz
Austria
Tel.: +43-(0)316-380-8564


Am 20.05.2017 19:43, schrieb Jameela Fatheema:

Dear all,

I am running wien2k version 14 on ubuntu 16.04

The purpose of my calculations is to get born effective charges for
AlN.
The scf has been run successfully for 1000 kpoints. And I am trying to
run the command “berrypi -k 6:6:6” but I am getting the following
error.

ubuntu@ubuntu-7G-Series:~/project/AlNitride/12may$ berrypi -k 6:6:6
Traceback (most recent call last):
 File "/home/ubuntu/WIEN2k_14/SRC_BerryPI/BerryPI/berrypi", line 20,
in 
 import numpy # math library
ImportError: No module named numpy

I have tried to make sure that python , numpy , berrypi and wannier90
have been installed correctly but the error remains.

I would like to know how to solve this problem. Kindly if anyone can
help.

Thank you

Regards

Jameela Fatheema
Department of Physics

University of Peshawar
___
Wien mailing list
Wien@zeus.theochem.tuwien.ac.at
http://zeus.theochem.tuwien.ac.at/mailman/listinfo/wien
SEARCH the MAILING-LIST at:
http://www.mail-archive.com/wien@zeus.theochem.tuwien.ac.at/index.html

___
Wien mailing list
Wien@zeus.theochem.tuwien.ac.at
http://zeus.theochem.tuwien.ac.at/mailman/listinfo/wien
SEARCH the MAILING-LIST at:  
http://www.mail-archive.com/wien@zeus.theochem.tuwien.ac.at/index.html


Re: [Wien] paramagnetic or diamagnetic

2017-05-09 Thread pieper


In general I am as afraid as Gerhard Fecher that this question is at 
least very difficult to answer. By definition one needs to calculate the 
low field response of the electrons, and there is a whole bunch of 
contributions, from the single electron orbital and spin-Pauli 
contributions Peter mentions to quasiparticle contributions that are in 
my understanding simply not present in the ground state calculated by 
DFT. And there are questions of ground states with spin corelations, 
temperature and field dependencies leading to changes from overall 
paramagnetic to diamagnetic response ...


Reading Peters response I wonder about even a single electron 
contribution and the NMR package in the case of metals: Does this 
package calculate only the (paramagnetic) Pauli spin susceptibility? Or 
is their Landau diamagnetism also included? For free electrons it 
amounts to 1/3 of the Pauli spin susceptibility (see your favorite 
textbook on the theory magnetism), so it is by no means safe to simply 
assume its small.


Greetings,

Martin Pieper


Am 08.05.2017 20:19, schrieb Peter Blaha:

In an insulator/semiconductor you have only the orbital part of the
susceptibility. This can be calculated using our NMR package and such
a material will be diamagnetic.

In metals you have in addition a spin suszeptibility, which you can
trivially calculate using spin-polarized calc. and an external field.
Usually this part is paramagnetic. And then you have to see, which
part dominates 

See also our NMR package.

Am 08.05.2017 um 16:28 schrieb Fecher, Gerhard:

I am afraid that this question can not be answered
and I doubt if any answer on this can be generalised to all kinds of 
materials.


As an experimentalist my answer will be: measure the susceptibility 
and it will tell you what your material is.


As you do not apply any magnetic field in your (non-spinpolarized) 
calculation, the induced magnetic moment will be zero
and a) tells you that this is true for both, diamagnetic or 
paramagnetic


What about b) ?
I tried it for Pt and indeed I find that the application of a magnetic 
field induces a magnetic moment (spin polarized calculation !)
that is parallel to the applied field, and linearly dependent on its 
size, as expected for a paramagnet.
However, I did not check whether the electrons in the closed shells 
behave diamagnetic as they should.
I doubt that this will work for all materials as in most cases the 
induced moment will be just to low to decide even if you use brute 
force (very high field, very much k-points etc.)
If a ferro- or other "magnetic" solution is close, then the 
application of the field may break the symmetry in such a way that you 
run into this state instead of staying in the paramagnetic state.

Diamagnetism will probably not bee seen in Semiconductors.
You may try semimetallic graphite which is a "strong" diamagnet to see 
whether it is possible to see any antiparallel allignment of induced 
magnetic moments.


I did not further check, maybe there are some codes available to 
calculate the suscebtibility of para- or diamagnetic materials.



Ciao
Gerhard

DEEP THOUGHT in D. Adams; Hitchhikers Guide to the Galaxy:
"I think the problem, to be quite honest with you,
is that you have never actually known what the question is."


Dr. Gerhard H. Fecher
Institut of Inorganic and Analytical Chemistry
Johannes Gutenberg - University
55099 Mainz
and
Max Planck Institute for Chemical Physics of Solids
01187 Dresden

Von: Wien [wien-boun...@zeus.theochem.tuwien.ac.at] im Auftrag von 
karima Physique [physique.kar...@gmail.com]

Gesendet: Montag, 8. Mai 2017 14:48
An: A Mailing list for WIEN2k users
Betreff: Re: [Wien] paramagnetic or diamagnetic

Thank you very much for your answer
I started a calculation in several magnetic phases (non-magnetic, 
ferromagnetic and antiferromagnetic) and I found that the non-magnetic 
phase is the most stable. so how can I know if the studied  material 
is a paramagnetic or diamagnetic material?

Thank you in advance

2017-05-08 8:06 GMT+02:00 Fecher, Gerhard 
<fec...@uni-mainz.de<mailto:fec...@uni-mainz.de>>:

What distinguishes a paramagnetic from a diamagnetic material ?
a) at zero magnetic field the induced magnetic moment is zero for both
b) at external magnetic field the induced magnetiuc moment is parallel 
/ antiparallel to the applied field.

c) both is true
d) none is true

There was already a discussion about paramagnetism, see
https://www.mail-archive.com/wien@zeus.theochem.tuwien.ac.at/msg15029.html

Ciao
Gerhard

DEEP THOUGHT in D. Adams; Hitchhikers Guide to the Galaxy:
"I think the problem, to be quite honest with you,
is that you have never actually known what the question is."


Dr. Gerhard H. Fecher
Institut of Inorganic and Analytical Chemistry
Johannes Gutenberg - Universi

Re: [Wien] change of lattice parameters while alloying

2017-05-09 Thread pieper
Accept the suggested structure. I always restart with initailization the 
new structure from nn.


If you replace one of the atoms occupying a position with multiplicity 
.gt. 1 (with more than one position line in struct) you break the 
symmetries inducing that multiplicity. You may well wind up with a 
larger primitive unit cell volume. That will change the basis vectors 
a,b,c.


Note that depending on what you want to compare in your series of 
calculations it probably is a good idea to do all calculations in the 
same space group - even if you are left with very low symmetry. Do this 
by giving all Cr in the pristine cell you want to substitute a differnet 
index (Cr1, Cr2 ...).


And remember that you do not calculate an (random) alloy, you calculate 
a periodic structure. Watch out for superstructure effects, perhaps by 
checking larger supercells with the same Br-concentration but different 
configuration.


Martin Pieper


Am 09.05.2017 01:55, schrieb Dr. K. C. Bhamu:

Dear Wien2k Users

I am alloying a quaternery compound: replacing Cr with Br one by one.
What I see is, the pristine compound is cubic with 225 SG. When I
replace Cl by Br one by one then lattice parameters deviates from
cubic to different structure.
like a=b, c or a,b,c. SG also changes.
During initialisation I accepted the new structure suggested by the
w2web. Should I accept the suggested structure?
As per my understanding from previous posts from mailing list we
should accept the suggested structure which is with lower symmetry.

Is the change of lattice parameters is normal behaviour of material or
I am doing any mistake?

Sincerely

Bhamu
___
Wien mailing list
Wien@zeus.theochem.tuwien.ac.at
http://zeus.theochem.tuwien.ac.at/mailman/listinfo/wien
SEARCH the MAILING-LIST at:
http://www.mail-archive.com/wien@zeus.theochem.tuwien.ac.at/index.html

---
Dr. Martin Pieper
Karl-Franzens University
Institute of Physics
Universitätsplatz 5
A-8010 Graz
Austria
Tel.: +43-(0)316-380-8564



___
Wien mailing list
Wien@zeus.theochem.tuwien.ac.at
http://zeus.theochem.tuwien.ac.at/mailman/listinfo/wien
SEARCH the MAILING-LIST at:  
http://www.mail-archive.com/wien@zeus.theochem.tuwien.ac.at/index.html


Re: [Wien] DOS of slab

2017-05-08 Thread pieper
Well, the colleague 'desires to obtain the DOS of the just top 
monolayer'.


He (or she) might dodge your "philosophical" question of what space is 
occupied by the topmost layer since the DOS is, after all, not a volume 
density but an energy density. Even if he solves all your questions and 
specifies what space qualifies as topmost layer he still has to decide 
which states contribute how much. Counting electrons in that volume does 
not solve the problem as stated. For example, what about the unoccupied 
DOS above EFermi?


The no less "philosophical" question then is to specify which states he 
wants to give the honor of being part of team 'topmost layer' - then to 
project on them.


And my best wishes for the most difficult part: convince his audience 
that the choice is convincing for the point to be made. ;-)


Martin Pieper


Am 08.05.2017 08:29, schrieb Fecher, Gerhard:

This might give rise to a "philosophical" question: What is the space
ocupied by the topmost layer ?

To use the DOS of the atoms in the "topmost" layer will neglect the
interstitial between the atoms, that is, the DOS will be not the one
of the complete
"topmost" layer but only the one inside the MT spheres around the
atoms that are defined to be in a selected layer.
(Note, already for bulk materials, the total DOS is not the sum of the
atomic DOSs)
However, to what layer does the interstitial belong ? What if the
surface is buckled ?
And further, is the vacuum part of the "topmost" layer or not ?

I wonder whether the Critic 2 code of the Oviedo Group in Spain can
help by using a Baader analysis,
and to calculate the DOS inside of the space filling basins instead of 
spheres.

This might improve the situation but still leaves some questions open.

Ciao
Gerhard

DEEP THOUGHT in D. Adams; Hitchhikers Guide to the Galaxy:
"I think the problem, to be quite honest with you,
is that you have never actually known what the question is."


Dr. Gerhard H. Fecher
Institut of Inorganic and Analytical Chemistry
Johannes Gutenberg - University
55099 Mainz
and
Max Planck Institute for Chemical Physics of Solids
01187 Dresden
____
Von: Wien [wien-boun...@zeus.theochem.tuwien.ac.at] im Auftrag von
pieper [pie...@ifp.tuwien.ac.at]
Gesendet: Donnerstag, 4. Mai 2017 10:22
An: A Mailing list for WIEN2k users
Betreff: Re: [Wien] DOS of slab

Assuming that you meant this to be a question, and that nobody answered
up to now: If you want a plot of some partial DOS for a bulk crystal,
you select an atom in the unit cell and possibly some orbital. Now you
want to calculate the partial DOS of the topmost layer ... did you try
(to understand) what happens if you select the atom(s) in your topmost
layer?


Am 29.04.2017 08:15, schrieb emami seyyed amir abbas:

Dear users.

I am trying to calculate the electronic and magnetic properties of a
thin film. I created a slab of 5 monolayer and run the scf. I desire
to obtain the DOS of the just  top monolayer but there is no option in
DOS program for this task. In DOS program i can plot DOS just for
specific atom not layer.

best regards.
___
Wien mailing list
Wien@zeus.theochem.tuwien.ac.at
http://zeus.theochem.tuwien.ac.at/mailman/listinfo/wien
SEARCH the MAILING-LIST at:
http://www.mail-archive.com/wien@zeus.theochem.tuwien.ac.at/index.html

---
Dr. Martin Pieper
Karl-Franzens University
Institute of Physics
Universitätsplatz 5
A-8010 Graz
Austria
Tel.: +43-(0)316-380-8564



___
Wien mailing list
Wien@zeus.theochem.tuwien.ac.at
http://zeus.theochem.tuwien.ac.at/mailman/listinfo/wien
SEARCH the MAILING-LIST at:
http://www.mail-archive.com/wien@zeus.theochem.tuwien.ac.at/index.html
___
Wien mailing list
Wien@zeus.theochem.tuwien.ac.at
http://zeus.theochem.tuwien.ac.at/mailman/listinfo/wien
SEARCH the MAILING-LIST at:
http://www.mail-archive.com/wien@zeus.theochem.tuwien.ac.at/index.html

---
Dr. Martin Pieper
Karl-Franzens University
Institute of Physics
Universitätsplatz 5
A-8010 Graz
Austria
Tel.: +43-(0)316-380-8564



___
Wien mailing list
Wien@zeus.theochem.tuwien.ac.at
http://zeus.theochem.tuwien.ac.at/mailman/listinfo/wien
SEARCH the MAILING-LIST at:  
http://www.mail-archive.com/wien@zeus.theochem.tuwien.ac.at/index.html


Re: [Wien] DOS of slab

2017-05-04 Thread pieper
Assuming that you meant this to be a question, and that nobody answered 
up to now: If you want a plot of some partial DOS for a bulk crystal, 
you select an atom in the unit cell and possibly some orbital. Now you 
want to calculate the partial DOS of the topmost layer ... did you try 
(to understand) what happens if you select the atom(s) in your topmost 
layer?



Am 29.04.2017 08:15, schrieb emami seyyed amir abbas:

Dear users.

I am trying to calculate the electronic and magnetic properties of a
thin film. I created a slab of 5 monolayer and run the scf. I desire
to obtain the DOS of the just  top monolayer but there is no option in
DOS program for this task. In DOS program i can plot DOS just for
specific atom not layer.

best regards.
___
Wien mailing list
Wien@zeus.theochem.tuwien.ac.at
http://zeus.theochem.tuwien.ac.at/mailman/listinfo/wien
SEARCH the MAILING-LIST at:
http://www.mail-archive.com/wien@zeus.theochem.tuwien.ac.at/index.html

---
Dr. Martin Pieper
Karl-Franzens University
Institute of Physics
Universitätsplatz 5
A-8010 Graz
Austria
Tel.: +43-(0)316-380-8564



___
Wien mailing list
Wien@zeus.theochem.tuwien.ac.at
http://zeus.theochem.tuwien.ac.at/mailman/listinfo/wien
SEARCH the MAILING-LIST at:  
http://www.mail-archive.com/wien@zeus.theochem.tuwien.ac.at/index.html


Re: [Wien] Band gap calculation of C diamond structure

2017-02-13 Thread pieper


The problem will be in the .struct you started from. You say yourself 
that the structure does not look like diamond in xcrysden. Don't change 
the .struct file in a case directory and restart the scf. The 
calculation relies on consistency of various input files.


The standard advice seems to be:

start a new case in fresh directory,
generate (preferably from w2web) a new struct file,
inspect it with xcrysden (there is no meaning in wasting CPU-time for 
scf-cycles on wrong structures),
proceed with the initialization (check the nearest neighbor distances in 
outputnn to see if you got the units in struct right)

and start the scf only when everything went ok.

... and keep fingers crossed


---
Dr. Martin Pieper
Karl-Franzens University
Institute of Physics
Universitätsplatz 5
A-8010 Graz
Austria
Tel.: +43-(0)316-380-8564


Am 13.02.2017 09:18, schrieb Abhilash Patra:

Dear Wien2k users,
I am running WIEN2k_14.2 on my  Lenovo workstation with fedora 23. I
want to calculate the band gap of C, Si, and Ge with the diamond
structure.
To generate my struct file I used: No. of Atoms-1, Space group-
227_Fd-3m and a=b=c=3.567 and one carbon atom at (0,0,0). Then set RMT
automatically (reduce RMT--0%). Then it set RMT to 1.18. I initiate
calculation with all the default values and run SFC as there is no
warning in the STDOUT.
But when I see the structure in the Xcrysden the figure is not looking
like diamond structure. And the band plot shows no gap

: GAP: -9.   Ry = -.eV  ( metallic )

I have already tried the structure : Atom=1, FCC, a=3.567, one atom at
(0,0,0) and other position at (0.25,0.25,0.25). This gives
diamond-likee structure but in the STDOUT file it gives some warning
like

warning: !!! Struct file is not consistent with space group found.
Number and name of space group: 227 (F d -3 m) [origin choice 2]

and SCF gives error like

head: cannot open ‘WIEN2k.inm’ for reading: No such file
or directory
head: cannot open ‘WIEN2k.inm’ for reading: No such file
or directory
no WIEN2k.clmsum(_old) file found, which is necessary for lapw0 !


  stop error


Please suggest me about the struct file which will give the results
for all like LDA, PBE and mGGAfunctionals

Thanks

--

Abhilash Patra
Research Scholar(Ph.D.)
School of Physical science
NISER,BBSR
___
Wien mailing list
Wien@zeus.theochem.tuwien.ac.at
http://zeus.theochem.tuwien.ac.at/mailman/listinfo/wien
SEARCH the MAILING-LIST at:
http://www.mail-archive.com/wien@zeus.theochem.tuwien.ac.at/index.html

___
Wien mailing list
Wien@zeus.theochem.tuwien.ac.at
http://zeus.theochem.tuwien.ac.at/mailman/listinfo/wien
SEARCH the MAILING-LIST at:  
http://www.mail-archive.com/wien@zeus.theochem.tuwien.ac.at/index.html


Re: [Wien] d-eg orbitals are not effected by applying Ueff

2017-01-28 Thread pieper
From the abstract of the paper you cited in your last mail I gather that 
this an alloy. So I suspect that there is some cite disorder with Co as 
well as Ga occupying the lattice sites randomly?


In that case Co would have a number of different configurations, due to 
the random distribution, that are not represented in your .struct file.


As for the gap: This was a question, not stating some fact. 
Unfortunately I did not follow the literature on GGA+U, so I am not 
aware of any studies with the same problem as yours. But I am also not 
aware of any study saying that a (reasonably sized) U MUST produce a 
gap. Here the real experts might provide more useful comments.


Best regards,

Martin Pieper


---
Dr. Martin Pieper
Karl-Franzens University
Institute of Physics
Universitätsplatz 5
A-8010 Graz
Austria
Tel.: +43-(0)316-380-8564


Am 28.01.2017 01:21, schrieb venkatesh chandragiri:

Dear Prof. Pieper,

the answers are as follows,

I UNDERSTAND THAT THIS IS AN ALLOY? IF YES, WHAT KIND OF SUPERCELL DO
YOU USE TO MODEL THE SYSTEM?

I do not used any supercell configuration. The case.struct file
contains the two atoms as given below. 

ARE THESE T2G AND EG SYMMETRIES ACTUALLY PRESENT IN ALL POSSIBLE
CONFIGURATIONS WITHIN THE ALLOY (PROBABLY NOT)? IS IT POSSIBLE THAT
THE SYMMETRY YOU ARTIFICIALLY IMPOSE IS AT THE BOTTOM OF THIS?

Experimentally, at room temperature, we have refined the crystal
structure of the Co50Ga50 alloy and the same used for the generating
the case.struct. I do not understand the your meaning of " all
possible configurations ".

WHY DO YOU BELIEVE THAT U SHOULD ACT IN THE SAME WAY ON DIFFERENT
SYMMETRIES? COULD IT BE THAT LOCALIZING T2G PAYS OFF MUCH BETTER THAN
EG?

I do not have experience before on the DFT +U studies, but have seen
in literature that one could create artifical energy gap by applying U
in order to explain the insulating behaviour of few oxides. It would
be helpful for me, if you provide me some references where the
localizing t2g pays off much better than eg by applying U. 

DID YOU TRY OTHER CONFIGURATIONS / SUPERCELLS / SYMMETRIES? 

I tried DFT + SO + U , but this also leads to the same feature in DOS
like DFT + U case. No supercells are tried. Could you please suggest
me next step or possible configurations so that I can give a try. 

thanks for your reply and looking forward to your help

venkatesh

=== Co50Ga50.struct ===

Co50Ga50                                        
                              
P   LATTICE,NONEQUIV.ATOMS:  2 221_Pm-3m                  
                    
MODE OF CALC=RELA unit=bohr                            
                       
  5.438632  5.438632  5.438632 90.00 90.00 90.00    
              
ATOM   1: X=0. Y=0. Z=0.
          MULT= 1          ISPLIT= 2
Ga1        NPT=  781  R0=0.5000 RMT=    2.2400   Z: 31.0
                  
LOCAL ROT MATRIX:    1.000 0.000 0.000
                     0.000 1.000 0.000
                     0.000 0.000 1.000
ATOM   2: X=0.5000 Y=0.5000 Z=0.5000
          MULT= 1          ISPLIT= 2
Co1        NPT=  781  R0=0.5000 RMT=    2.3500   Z: 27.0
                  
LOCAL ROT MATRIX:    1.000 0.000 0.000
                     0.000 1.000 0.000
                     0.000 0.000 1.000
  48      NUMBER OF SYMMETRY OPERATIONS
-1 0 0 0.
 0-1 0 0.
 0 0-1 0.
       1
-1 0 0 0.
 0-1 0 0.
 0 0 1 0.
       2
-1 0 0 0.
 0 0-1 0.
 0-1 0 0.
       3
-1 0 0 0.
 0 0 1 0.
 0-1 0 0.
       4
-1 0 0 0.
 0 0-1 0.
 0 1 0 0.
       5
-1 0 0 0.
 0 0 1 0.
 0 1 0 0.
       6
-1 0 0 0.
 0 1 0 0.
 0 0-1 0.
       7
-1 0 0 0.
 0 1 0 0.
 0 0 1 0.
       8
 0-1 0 0.
-1 0 0 0.
 0 0-1 0.
       9
 0-1 0 0.
-1 0 0 0.
 0 0 1 0.
      10
 0 0-1 0.
-1 0 0 0.
 0-1 0 0.
      11
 0 0 1 0.
-1 0 0 0.
 0-1 0 0.
      12
 0 0-1 0.
-1 0 0 0.
 0 1 0 0.
      13
 0 0 1 0.
-1 0 0 0.
 0 1 0 0.
      14
 0 1 0 0.
-1 0 0 0.
 0 0-1 0.
      15
 0 1 0 0.
-1 0 0 0.
 0 0 1 0.
      16
 0-1 0 0.
 0 0-1 0.
-1 0 0 0.
      17
 0-1 0 0.
 0 0 1 0.
-1 0 0 0.
      18
 0 0-1 0.
 0-1 0 0.
-1 0 0 0.
      19
 0 0 1 0.
 0-1 0 0.
-1 0 0 0.
      20
 0 0-1 0.
 0 1 0 0.
-1 0 0 0.
      21
 0 0 1 0.
 0 1 0 0.
-1 0 0 0.
      22
 0 1 0 0.
 0 0-1 0.
-1 0 0 0.
      23
 0 1 0 0.
 0 0 1 0.
-1 0 0 0.
      24
 0-1 0 0.
 0 0-1 0.
 1 0 0 

Re: [Wien] d-eg orbitals are not effected by applying Ueff

2017-01-27 Thread pieper


Not really a comment, just a few questions one might consider (I 
probably would, but I am half an expert at best):


I understand that this is an alloy? If yes, what kind of supercell do 
you use to model the system? Are these t2g and eg symmetries actually 
present in all possible configurations within the alloy (probably not)? 
Is it possible that the symmetry you artificially impose is at the 
bottom of this? Why do you believe that U should act in the same way on 
different symmetries? Could it be that localizing t2g pays off much 
better than eg? Did you try other configurations / supercells / 
symmetries?


Best regards,

Martin Pieper

---
Dr. Martin Pieper
Karl-Franzens University
Institute of Physics
Universitätsplatz 5
A-8010 Graz
Austria
Tel.: +43-(0)316-380-8564


Am 26.01.2017 19:17, schrieb venkatesh chandragiri:

Dear Prof. Lyudmila Dobysheva

Thanks for your reply. There was a old paper on this alloy, which show
similar kind of results, ie. deep pseudo gap with finite states at Ef.
please see the link below

http://iopscience.iop.org/article/10.1088/0305-4608/12/2/009/meta [1]

In this article they discussed the raise in resitivity using the Mott
theory of electron localization when electrons are captured by these
states around Ef, so called localised states.

However, I want to apply U for the Co-d states in order to explain the
raise in resistivity at low temperatures by creating artificial gap.
But although, I applied U= 5 eV on Co-d states, d-t2g states are only
moved well below the Fermi level while d-eg are not effected. This
leads to presence of finite states from d-eg at Ef.

please share any idea you have about this..

Can someone has any another comment on this..?

thanks

venkatesh

Links:
--
[1] http://iopscience.iop.org/article/10.1088/0305-4608/12/2/009/meta

___
Wien mailing list
Wien@zeus.theochem.tuwien.ac.at
http://zeus.theochem.tuwien.ac.at/mailman/listinfo/wien
SEARCH the MAILING-LIST at:
http://www.mail-archive.com/wien@zeus.theochem.tuwien.ac.at/index.html

___
Wien mailing list
Wien@zeus.theochem.tuwien.ac.at
http://zeus.theochem.tuwien.ac.at/mailman/listinfo/wien
SEARCH the MAILING-LIST at:  
http://www.mail-archive.com/wien@zeus.theochem.tuwien.ac.at/index.html


Re: [Wien] How to include the localized d orbitals in the atomic spheres?

2016-11-30 Thread pieper
You should recognize that the local atomic spheres are just a 
theoretical construct to arrive at a (very good) set of basis functions. 
It has nothing to do with wether or not the d-orbitals you describe with 
them are localized or not, or with the size of the gap, the magnetic 
moment ... It defines just the coordinate system one uses to describe 
the state. It has as much to do with what the state is like, as the use 
of cartesian or spherical coordinates has to do with where on earth you 
are.


You also cannot somehow push the electrons out of the intermediate space 
- it would mean some additional potential with simply is not there in 
your material. And using spheres you certainely cannot avoid some 
intermediate space between them.



---
Dr. Martin Pieper
Karl-Franzens University
Institute of Physics
Universitätsplatz 5
A-8010 Graz
Austria
Tel.: +43-(0)316-380-8564


Am 30.11.2016 13:28, schrieb Abderrahmane Reggad:

Thank you Dr Pieper for the clarification.

How much does the including of the localized d orbitals inside the
atomic spheres improve the band gap and the magnetic moment ?

And how can we include all the localized d orbitals inside the atomic
spheres?

Best regards

Mr:
A.Reggad  

Laboratoire de Génie Physique
Université Ibn Khaldoun - Tiaret

Algerie


___
Wien mailing list
Wien@zeus.theochem.tuwien.ac.at
http://zeus.theochem.tuwien.ac.at/mailman/listinfo/wien
SEARCH the MAILING-LIST at:
http://www.mail-archive.com/wien@zeus.theochem.tuwien.ac.at/index.html

___
Wien mailing list
Wien@zeus.theochem.tuwien.ac.at
http://zeus.theochem.tuwien.ac.at/mailman/listinfo/wien
SEARCH the MAILING-LIST at:  
http://www.mail-archive.com/wien@zeus.theochem.tuwien.ac.at/index.html


Re: [Wien] Discrepancy in the simulation of the paramagnetic state

2016-11-29 Thread pieper
My (and probably Xavier's) concern with Regaard's question was something 
else.


I have no problem whatsoever with you finding an approximation for Pt 
using wave functions. After all, your ground state model has zero static 
local moments, as has the Pt you want to model. ;-)


However, the approximation seems at least dubious if the ground state 
model and the low temperature state of the material differ. If the 
material enters some magnetic state and the spin-polarized(!) DFT model 
does not one might look for a problem with the structure data, some 
structural phase transition, ...


So I am with Xavier, and I would at least advise to be careful with the 
idea I understood Regaad did somehow get: Artificially compensate spins 
(e.g. via LDA instead of LSDA) to find an approximation for the 
paramagnetic phase at elevated temperature of a low temperature magnet.


There is at least one difference between the material and the model: the 
model will NOT be paramagnetic (obtain a positive magnetization in an 
applied magnetic field). Wether or not this (or any other differences 
induced by the forced spin compensation) poses a problem will depend on 
what situation one wants to model.



---
Dr. Martin Pieper
Karl-Franzens University
Institute of Physics
Universitätsplatz 5
A-8010 Graz
Austria
Tel.: +43-(0)316-380-8564


Am 28.11.2016 08:33, schrieb Fecher, Gerhard:

I hope you agree that Pt is paramagnetic
I did two calculations for Pt, one was  spin polarized the other not.
The results are identical, no resulting magnetic moment (indeed, I
started with one in the spin polarized case), did I play a trick or
did Wien2k play a trick ?
but may be Wien2k can not be used to calculate the electronic
structure of Pt, because it is paramagnetic (Pt, not Wien2k !).

I hope you agree that Pt is paramagnetic even at Zero temperature.
why do I need to include temperature effects to calculate the ground
state of Pt (at 0 K, where else) ?
... and what should MtC calculations tell me about it ?

Remark 1:
Calculations may be  "spin polarized" (LSDA) or not (LDA) or they may
be even more sophisticated "non-colinear spin polarized" or they may
be for "disordred local moments"
or for "spin spirals", or ???,  just to name some.

Remark 2:
Materials may be diamagnetic, paramagnetic (Langevin, Pauli, van
Vleck), ferromagnetic (localised moments, itinerant), ferrimagnetic
(collinear, non-collinear), etc..

Therefore, I repeat my question:   How do you distinguish diamagnetic,
paramagnetic, ferromagnetic, and ... states ?

The answer is for you, not for me.

I tried to calculate for Pt using Hohenberg Kohn DFT, but I could not
find the functional, all I found was some approximation using wave
functions.
Don't worry I will not ask a question about it ;-)

Ciao
Gerhard

DEEP THOUGHT in D. Adams; Hitchhikers Guide to the Galaxy:
"I think the problem, to be quite honest with you,
is that you have never actually known what the question is."


Dr. Gerhard H. Fecher
Institut of Inorganic and Analytical Chemistry
Johannes Gutenberg - University
55099 Mainz
and
Max Planck Institute for Chemical Physics of Solids
01187 Dresden

Von: Wien [wien-boun...@zeus.theochem.tuwien.ac.at] im Auftrag von
Xavier Rocquefelte [xavier.rocquefe...@univ-rennes1.fr]
Gesendet: Sonntag, 27. November 2016 12:46
An: wien@zeus.theochem.tuwien.ac.at
Betreff: Re: [Wien] Discrepancy in the simulation of the paramagnetic 
state


Just to add one more point to this funny discussion, the term
"paramagnetic" is sometimes used in the DFT litterature in an improper 
way.


It could clearly lead to misunderstanding for researchers who do not
know so much on how magnetic properties could evolve with temperature
and applied magnetic field. When you see in a paper "paramagnetic 
state"

simulated using DFT ... it is NOT paramagnetic at all, it is simply a
trick which must be considered with care as previously mentionned by
Peter, Eliane and Martin.

If you want to simulate a paramagnetic state you need to include the
temperature effects, i.e. you should consider the spin dynamics and the
competition between magnetic exchange interactions and thermal
fluctuations. This could be done, at least, using Monte-Carlo
calculations based on an effective hamiltonian constructed on top of 
DFT

parameters (including magnetic exchange and anisotropy at least).

Best Regards

Xavier




Le 27/11/2016 à 10:01, Fecher, Gerhard a écrit :
How do you distinguish a diamagnetic, a paramagnetic, a ferromagnetic, 
and an antiferromagnetic state.


Think !

This will answer your question, hopefully.

Ciao
Gerhard

DEEP THOUGHT in D. Adams; Hitchhikers Guide to the Galaxy:
"I think the problem, to be quite honest with you,
is that you have never actually known what the question is."


Dr. Gerhard H. Feche

Re: [Wien] How to include the localized d orbitals in the atomic spheres?

2016-11-29 Thread pieper
Look into section 7.3 of the user guide: ORB (Calculate orbital 
potentials)


The very first sentence reads:

orb calculates the orbital dependent potentials, i.e. >>>potentials 
which are nonzero in the atomic

spheres only <<So, in the region between the atomic spheres the potentials arising from 
U or hybrids are set to zero. Not strictly what happens in the material, 
but a way to improve the model significantly and keep the calculation 
viable.



---
Dr. Martin Pieper
Karl-Franzens University
Institute of Physics
Universitätsplatz 5
A-8010 Graz
Austria
Tel.: +43-(0)316-380-8564


Am 28.11.2016 23:21, schrieb Abderrahmane Reggad:

Thank you Prof Cottenier for your answer

My question is made according to the following statement:

" The DFT+U and EECE are applied only inside atomic spheres "

What does it mean that and how to realize it ?

Best regards

--

Mr:
A.Reggad  

Laboratoire de Génie Physique
Université Ibn Khaldoun - Tiaret

Algerie


___
Wien mailing list
Wien@zeus.theochem.tuwien.ac.at
http://zeus.theochem.tuwien.ac.at/mailman/listinfo/wien
SEARCH the MAILING-LIST at:
http://www.mail-archive.com/wien@zeus.theochem.tuwien.ac.at/index.html

___
Wien mailing list
Wien@zeus.theochem.tuwien.ac.at
http://zeus.theochem.tuwien.ac.at/mailman/listinfo/wien
SEARCH the MAILING-LIST at:  
http://www.mail-archive.com/wien@zeus.theochem.tuwien.ac.at/index.html


Re: [Wien] Discrepancy in the simulation of the paramagnetic state

2016-11-26 Thread pieper

My two cents concerning this problem:

What moments do you consider to be zero, Reggad? With Ni being a METAL I 
would claim that the electron spin moment is the one to inspect. 
Certainely you (or the authors you read) do not propose to set the 
electron spin to zero?


Remember that DFT calculates the situation at T=0. Simulating 
paramagnetism at high temperature by forcing the spin-polarization to 
zero may work for some purposes, but will be bad if you are interested 
in magnetic properties: With zero moments nothing will happen if you 
switch on a magnetic field!


With 4f-electrons there is a good chance that one can consider the whole 
4f-shell as a quantum system of its own, instead of the single 
electrons. Then each lattice site carrys some moment. The paramgnetic 
phase of such an ensemble is by definition the one where the expectation 
value of the total magnetization vanishes due to incoherent random 
fluctuations of the 4f-moments. imho, the view of this phase as all the 
moments pointing in random directions has its merits, but one should not 
over-interprete the picture. Such a state is most probably not even an 
eigenstate of the Hamiltonian, let alone the ground state that DFT is 
concerned with.




---
Dr. Martin Pieper
Karl-Franzens University
Institute of Physics
Universitätsplatz 5
A-8010 Graz
Austria
Tel.: +43-(0)316-380-8564


Am 26.11.2016 22:30, schrieb Abderrahmane Reggad:

Thank you Prof Blaha for your quick answer.

The Ni atom is 3d transition metal . But my question is about the
simulation of the paramagnetic state. There are many people that
considere that the paramagnetic state is the non-spin polarierd one
and the magnetic moment is zero, but you say no and the magnetic
moments exist in arbitrary directions and my quoting is about that.

I have given 2 examples for that discrepancy with your statement.

Best regards--

Mr:
A.Reggad  

Laboratoire de Génie Physique
Université Ibn Khaldoun - Tiaret

Algerie


___
Wien mailing list
Wien@zeus.theochem.tuwien.ac.at
http://zeus.theochem.tuwien.ac.at/mailman/listinfo/wien
SEARCH the MAILING-LIST at:
http://www.mail-archive.com/wien@zeus.theochem.tuwien.ac.at/index.html

___
Wien mailing list
Wien@zeus.theochem.tuwien.ac.at
http://zeus.theochem.tuwien.ac.at/mailman/listinfo/wien
SEARCH the MAILING-LIST at:  
http://www.mail-archive.com/wien@zeus.theochem.tuwien.ac.at/index.html


Re: [Wien] question about tetra's choice of Emax

2016-11-21 Thread pieper
Sure, Cu has a moment in any number of cases with more or less complex 
band structures.


This was just recalling the basic textbook model of itinerant 
ferromagnetism of elemental metals from the 1960's. To me the simple 
band structure underlying that (not too bad) model looks much like the 
cases where the tetra's and lapw1's automatic setting of Emax might have 
difficulties.



---
Dr. Martin Pieper
Karl-Franzens University
Institute of Physics
Universitätsplatz 5
A-8010 Graz
Austria
Tel.: +43-(0)316-380-8564


Am 21.11.2016 09:48, schrieb Lyudmila Dobysheva:

17.11.2016 16:47, pieper wrote:

One might (correctly) expect Cu with 11 electrons for
these bands to be a happy paramagnet: the spins of 10 e in the d-band
compensate, and the exchange for the one in the 4s is too small to
redistribute anything from there, 4s is spin balanced. Furthermore it
costs too much energy to increase the 4s population above half full, 
the

exchange gain in an unbalanced 4d is not sufficient.


I don't follow your discussion, but for the sake of accuracy, I want
to comment that Cu atoms have magnetic moment in the Cu oxides.

Best regards
  Lyudmila Dobysheva
--
Phys.-Techn. Institute of Ural Br. of Russian Ac. of Sci.
426001 Izhevsk, ul.Kirova 132
RUSSIA
--
Tel.:7(3412) 432045(office), 722529(Fax)
E-mail: l...@ftiudm.ru, lyuk...@mail.ru (office)
lyuk...@gmail.com (home)
Skype:  lyuka17 (home), lyuka18 (office)
http://ftiudm.ru/content/view/25/103/lang,english/
--
___
Wien mailing list
Wien@zeus.theochem.tuwien.ac.at
http://zeus.theochem.tuwien.ac.at/mailman/listinfo/wien
SEARCH the MAILING-LIST at:
http://www.mail-archive.com/wien@zeus.theochem.tuwien.ac.at/index.html

___
Wien mailing list
Wien@zeus.theochem.tuwien.ac.at
http://zeus.theochem.tuwien.ac.at/mailman/listinfo/wien
SEARCH the MAILING-LIST at:  
http://www.mail-archive.com/wien@zeus.theochem.tuwien.ac.at/index.html


Re: [Wien] orbital moment-mBJ

2016-11-17 Thread pieper

High Komal,

please consult the LAPWDM sections in the user guide. There calculation 
of the orbital moment is explained.


Best regards,

Martin Pieper


---
Dr. Martin Pieper
Karl-Franzens University
Institute of Physics
Universitätsplatz 5
A-8010 Graz
Austria
Tel.: +43-(0)316-380-8564


Am 17.11.2016 13:35, schrieb Komal Bapna:

Dear Wein users

I am using mBJ potential to perform spin-polarised calculations on a
perovskite structure. The scf file shows spin magnetic moment. Can I
have the info about its orbital moment also using "runsp" command as I
have used or I need to give some more command .

Please reply
Regards

--

KOMAL
___
Wien mailing list
Wien@zeus.theochem.tuwien.ac.at
http://zeus.theochem.tuwien.ac.at/mailman/listinfo/wien
SEARCH the MAILING-LIST at:
http://www.mail-archive.com/wien@zeus.theochem.tuwien.ac.at/index.html

___
Wien mailing list
Wien@zeus.theochem.tuwien.ac.at
http://zeus.theochem.tuwien.ac.at/mailman/listinfo/wien
SEARCH the MAILING-LIST at:  
http://www.mail-archive.com/wien@zeus.theochem.tuwien.ac.at/index.html


Re: [Wien] question about tetra's choice of Emax

2016-11-16 Thread pieper

Thanks for the quick response!

Sorry, I did not fully catch the problem of consistency between what is 
calculated in lapw1 and integrated in tetra.


However, your PS and PPS leave me worried about where things go sideways 
with something as simple as fcc Co should be.


As for the version, my VERSION file says
WIEN2k_14.2 (Release 15/10/2014)
which to my knowledge is not too bad? It does not include the dynamical 
Emax in lapw1?


With that version, kgen 1 vectors, spin-resolved, and everything 
else default in w2web, I get from tetra something like the file below. 
Emax is apparently changed to a value below E_F.


I understand that this might lead to problems already in the scf because 
the wrong states are occupied, but in that case again I cannot rely on 
the DOS, even when I increase Emax in .in1 and run lapw1 ... I have to 
re-run the scf, correct?



-  Co-fcc.outputtup --
 Co-fcc #

 IAV :  0
 NPRINT  :  1
  2 CASES FOR DOS:  ATOM   L

 cc-Co
 LATTICE CONST.= 6.79300 6.79300 6.79300   FERMI ENERGY=   0.56702
   48 <; NMAT <;   63   SPIN=2   NATO=   2
 JATOM  1  MULT= 1  ISPLIT= 2  tot,0,1,2,D-eg,D-t2g,3
 CASE 1 :   ATOM NUMBER  0   COLUMN READ  0   DOSTYPE=total-DOS
 CASE 2 :   ATOM NUMBER  1   COLUMN READ  1   DOSTYPE=  1:total
 We will add0  DOS-cases together:
 BAND LIMITS OF BAND   1 ARE  -3.81359  -3.79544
 BAND LIMITS OF BAND   2 ARE  -3.80710  -3.79544
 BAND LIMITS OF BAND   3 ARE  -3.80235  -3.79544
 BAND LIMITS OF BAND   4 ARE  -0.07833   0.28402
 BAND LIMITS OF BAND   5 ARE   0.24033   0.37076
 BAND LIMITS OF BAND   6 ARE   0.32045   0.48759
 BAND LIMITS OF BAND   7 ARE   0.37076   0.51438
 BAND LIMITS OF BAND   8 ARE   0.40511   0.51502
 BAND LIMITS OF BAND   9 ARE   0.43442   1.04422
  EMAX reduced due to lower HIGHEST BAND-minimum
 EMIN, DE, EMAX  :  -0.5   0.00200   0.43442

  EMIN=  -0.5 EMAX=   0.43442 EFACTR=499.6948 ESTEP =   
0.00200

 ENERGY BAND1 THROUGH9 ENERGY CHANNEL:1   TO  468
 NUMBER OF K-POINTS: 165
 NUMBER OF TETRAHEDRONS: 693
#  BAND9
#EF=   0.56702 NDOS= 2 NENRG=  468Gaussian bradening: 
0.00300

 NUMBER OF ELECTRONS UP TO EF :0.

 DOS in states/Ry/spin
 smearing   1  0.3989422748506432.00
 smearing   1  0.3989422748506432.00
# ENERGY0 total-DOS1   1:total
 -0.5 0.00   0. 0.00   0.
 -0.49800 0.00   0. 0.00   0.
 -0.49600 0.00   0. 0.00   0.
.
.
.
--



---
Dr. Martin Pieper
Karl-Franzens University
Institute of Physics
Universitätsplatz 5
A-8010 Graz
Austria
Tel.: +43-(0)316-380-8564


Am 16.11.2016 07:41, schrieb Peter Blaha:

The DOS is calculated up to a value for which we can guarantee that
the DOS is correct and complete.

Of course, the DOS up to the highest band-maximum would be non-zero,
but there is a chance that some (maybe a lot) of DOS is missing and a
user would not notice this and "misintewrprete" this wrong DOS.

Now he can trust that the calculated DOS is ok, and if he needs higher
DOS, he has to increase the emax in case.in1 and/or in case.int.

PS: You are probably using an older WIEN2k version, because now we use
a "dynamical" Emax in case.in1, which takes the actual EF into
account. But of course in cases of very steep bands above EF, the
default in1 file may still be insufficient.

PPS: In your situation it could even be, that the scf calculation is
"wrong", since you occupied the wrong bands ....

Am 15.11.2016 um 19:12 schrieb pieper:

Hello, mailing list,

yesterday I had for the first time some dispute with the way tetra
automatically chooses its input energy range. I would like to 
understand

why the particular automatic choice of Emax was introduced.

Until then it worked so well that I didn't even notice that Emax is
automatically adjusted, but then I wanted to illustrate Wien2k by
calculating the example fcc Ni. I took Co instead of Ni, lattice
constant adjusted to 6.637, RMT to touching spheres. I used w2web for
initialiazation and initiating scf, kgen with 1 vectors,
spin-polarized in an FM starting configuration, no spin-orbit 
coupling.


Then I chose DOS from the Tasks menu, skipped the optional steps lapw1
and qtl, used lapw2 -qtl, configured .int to calculate just the total
DOS (the default). When I proudly presented the result of dosplot to a
visitor the plot ended below the Fermi energy ...

Increasing Emax in .int did nothing, as the experts probably could 
have

told me beforehand. The (in this case in my view annoying) automatic
choice of Emax that kicks in is indicated in the header of .outputtup
(as well as in the user guide - and yes, I know one should read it):

.
.
.
BAND LIMITS OF BAND   8 ARE   0.40511   0.51502
B

[Wien] question about tetra's choice of Emax

2016-11-15 Thread pieper

Hello, mailing list,

yesterday I had for the first time some dispute with the way tetra 
automatically chooses its input energy range. I would like to understand 
why the particular automatic choice of Emax was introduced.


Until then it worked so well that I didn't even notice that Emax is 
automatically adjusted, but then I wanted to illustrate Wien2k by 
calculating the example fcc Ni. I took Co instead of Ni, lattice 
constant adjusted to 6.637, RMT to touching spheres. I used w2web for 
initialiazation and initiating scf, kgen with 1 vectors, 
spin-polarized in an FM starting configuration, no spin-orbit coupling.


Then I chose DOS from the Tasks menu, skipped the optional steps lapw1 
and qtl, used lapw2 -qtl, configured .int to calculate just the total 
DOS (the default). When I proudly presented the result of dosplot to a 
visitor the plot ended below the Fermi energy ...


Increasing Emax in .int did nothing, as the experts probably could have 
told me beforehand. The (in this case in my view annoying) automatic 
choice of Emax that kicks in is indicated in the header of .outputtup 
(as well as in the user guide - and yes, I know one should read it):


.
.
.
BAND LIMITS OF BAND   8 ARE   0.40511   0.51502
BAND LIMITS OF BAND   9 ARE   0.43442   1.04422
  EMAX reduced due to lower HIGHEST BAND-minimum
 EMIN, DE, EMAX  :  -0.5   0.00200   0.43442

The problem in this case is that the 'lower highest Band-minimum' is 
BELOW the Fermi-energy:


  EF and DOS at fermi level ***
  0.56702 0.00  0.00

So one MUST go back to the (in principle optional) lapw1 step with some 
larger Emax in .in1 (as indicated in the DOS menu and in the UG). 
Increasing Emax there appears to me a little clumsy since one has to 
guess an Emax that will generate a band with a band minimum above E_F.


Why not have tetra choose Emax as the minimum of the Emax input in .int 
and the highest Band-MAXIMUM?



---
Dr. Martin Pieper
Karl-Franzens University
Institute of Physics
Universitätsplatz 5
A-8010 Graz
Austria
Tel.: +43-(0)316-380-8564


___
Wien mailing list
Wien@zeus.theochem.tuwien.ac.at
http://zeus.theochem.tuwien.ac.at/mailman/listinfo/wien
SEARCH the MAILING-LIST at:  
http://www.mail-archive.com/wien@zeus.theochem.tuwien.ac.at/index.html


Re: [Wien] No convergence during Volume Optimization

2016-11-14 Thread pieper
The 'difference between energy and force approaches' is just that: a 
different approach to find the equilibrium positions in space for the 
atoms and their electrons within the structural model given by the 
.struct file.


The equilibrium is the energy minimum in the parameter space under 
consideration. An energy minimum means that the derivatives with respect 
to (atom) positions - that is the forces - vanish. Knowing the 
derivatives (forces) helps, of course, a lot in finding the minimum - 
they point the way to the next (local) minimum.


In the user guide you see:

--User guide --
5.3.1
 Lattice parameters (Volume, c/a, lattice parameters)

Package optimize

The auxilliary program optimize (x optimize) generates from an existing 
case.struct (or
case initial.struct, which is generated at the first call of optimize) a 
series of struct files
with various volumes (or c/a ratios, or other modified parameters) 
(depending on your input):


.
.
.

After execution of this script you should have a series of scf-files 
with energies corresponding to the
modified parameters, which should allow you to find the corresponding 
equillibrium parameters.
For the volume optimization an analysis tool is available, other tools 
are under development).


--

The minimum total energy defines the equilibrium. The derivatives with 
respect to lattice parameters are not easy to obtain during the scf 
cycle so for lattice parameters an 'energy approach' is used.


For the internal parameters this is different. Persons as ingenious as 
Prof. Marks can calculate the derivatives with respect to internal 
parameters from the charge distribution at affordable computational cost 
during the scf.



--User guide --

5.3.2
 Minimization of internal parameters (min lapw)

Most of the more complicated structures have free internal structural 
parameters, which can either
be taken from experiment or optimized using the calculated forces on the 
nuclei.
Starting with WIEN2k 11.1 there are two possibilities to determine the 
equilibrium position of all
individual atoms automatically (obeying the symmetry constraints of a 
certain space group). One

can use either

the shell script min lapw, together with the program mini, which will 
run a scf-cycle, update
the positions using the calculated forces and restarts a new scf cycle. 
This continues until

forces drop below a certain value;

or use the normal scf-scripts run lapw -min where in case.inm the 
switch MSR1 will be
modified to MSR1a such that the charge density and the positions are 
simultaneously opti-

mized during the scf-cycle.

--

The first option uses what you call the 'force approach': in 
equilibrium, no forces should push the atoms around.


The second option indicates a mixed approach: The positions of the atoms 
(according to forces acting on them) and the ones of the electrons (to 
minimize the total energy) are BOTH adjusted in each step of the scf.


As I said, there can be many reasons why your calculation did not reach 
convergence for some structural parameters. Did the scf stop without 
errors because the maximum number of iterations was reached? If yes, 
what was this number of iterations? Maybe your convergence criteria are 
too strong for the numerical precision you set by parameters like RKMAX 
or the k-mesh? Maybe the scf oscillates between several good solutions 
(is it a magnetic case?). Maybe your starting configuration and/or the 
model Hamiltonian is completely off or missing some ingredient and the 
poor scf wanders helpless around, lost in a multidimensional world ...


Best regards,

Martin Pieper


---
Dr. Martin Pieper
Karl-Franzens University
Institute of Physics
Universitätsplatz 5
A-8010 Graz
Austria
Tel.: +43-(0)316-380-8564


Am 13.11.2016 19:14, schrieb Abderrahmane Reggad:

Thank you Dr Pieper for your interesting to my questions.

I have optimized the atomic positions before doing calculation.

Tha thing that I didn't understand is that the convergence is reached
for some points but not for others.

For the "optimization notes " , there is no mention on the difference
betwwen the energy and force approaches.

Best regards

 

--

Mr:
A.Reggad  

Laboratoire de Génie Physique
Université Ibn Khaldoun - Tiaret

Algerie


___
Wien mailing list
Wien@zeus.theochem.tuwien.ac.at
http://zeus.theochem.tuwien.ac.at/mailman/listinfo/wien
SEARCH the MAILING-LIST at:
http://www.mail-archive.com/wien@zeus.theochem.tuwien.ac.at/index.html

___
Wien mailing list
Wien@zeus.theochem.tuwien.ac.at
http://zeus.theochem.tuwien.ac.at/mailman/listinfo/wien
SEARCH the MAILING-LIST at:  
http://www.mail-archive.com/wien@zeus.theochem.tuwien.ac.at/index.html


Re: [Wien] No convergence during Volume Optimization

2016-11-12 Thread pieper


Look at the curve energy vs. volume and decide yourself wether its worth 
the while to increase the number of iterations (assuming that's the 
reason for the 'jump to the next point').


It also is a good idea to look for reasons why the convergence is slow 
(problem in the struct file? Bad position for some atom? Lattice 
constants very far from equilibrium? ... there are a lot of less trivial 
possibilities)


Concerning your other question wether or not optimization with respect 
to energy or with respect to forces is the same I recommend reading the 
section on structure optimization in the user guide, and perhaps the 
'Optimization Notes' of L. D. Marks that you will find on the Wien2k 
website. I am completely unable to improve on those.


Best luck

Martin Pieper


---
Dr. Martin Pieper
Karl-Franzens University
Institute of Physics
Universitätsplatz 5
A-8010 Graz
Austria
Tel.: +43-(0)316-380-8564


Am 12.11.2016 12:31, schrieb Abderrahmane Reggad:

Dear Wien Users

Sometimes , It happens that during volume optimization , the
calculation doesn't converge to the chosen energy criteria and jumps
to the next point . In this case, should we delete this point and
choose another point or we could accept this calculation.

For example , with energy criteria value of 0.0001 and with some
points (we take 3 points as example)/

-  6 percent point:

ETEST: .00083

- 0 percent point:

ETEST: .00034

- 6 percent point

ETEST: .00013

Best regards

--

Mr:
A.Reggad  

Laboratoire de Génie Physique
Université Ibn Khaldoun - Tiaret

Algerie


___
Wien mailing list
Wien@zeus.theochem.tuwien.ac.at
http://zeus.theochem.tuwien.ac.at/mailman/listinfo/wien
SEARCH the MAILING-LIST at:
http://www.mail-archive.com/wien@zeus.theochem.tuwien.ac.at/index.html

___
Wien mailing list
Wien@zeus.theochem.tuwien.ac.at
http://zeus.theochem.tuwien.ac.at/mailman/listinfo/wien
SEARCH the MAILING-LIST at:  
http://www.mail-archive.com/wien@zeus.theochem.tuwien.ac.at/index.html


Re: [Wien] Minimization before Volume Optimization ?

2016-11-11 Thread pieper
sgroup (or other scripts in the initialization) does'nt change the 
positions of the atoms, it changes the basis used to describe the 
positions. 'Optimization' is done with respect to internal forces, or 
energy, both unknown at that step.


Good luck

Martin Pieper

---
Dr. Martin Pieper
Karl-Franzens University
Institute of Physics
Universitätsplatz 5
A-8010 Graz
Austria
Tel.: +43-(0)316-380-8564


Am 11.11.2016 14:43, schrieb Abderrahmane Reggad:

Now I want to know if a low symmetry structure (orthorhombic)
derived  from its high symmetry structure (hexagonal NiAs )has its
atomic positions optimized (because they are created with sgroup
program) or need to be optimized .

Best regards

--

Mr:
A.Reggad  

Laboratoire de Génie Physique
Université Ibn Khaldoun - Tiaret

Algerie


___
Wien mailing list
Wien@zeus.theochem.tuwien.ac.at
http://zeus.theochem.tuwien.ac.at/mailman/listinfo/wien
SEARCH the MAILING-LIST at:
http://www.mail-archive.com/wien@zeus.theochem.tuwien.ac.at/index.html

___
Wien mailing list
Wien@zeus.theochem.tuwien.ac.at
http://zeus.theochem.tuwien.ac.at/mailman/listinfo/wien
SEARCH the MAILING-LIST at:  
http://www.mail-archive.com/wien@zeus.theochem.tuwien.ac.at/index.html


Re: [Wien] Is there an effect of SO on the ground state

2016-10-31 Thread pieper
Of course inclusion of SO interactions may lead to another magnetic 
ground state. For example, at least in principle it certainely will do 
so if the symmetry of your Hamiltonian including SO breaks some symmetry 
of the ground state you found without SO. So you might want to look 
wether or not initso induces changes of your symmetry. However, the 
effect in whatever you are interested in might be too small to make a 
real difference.


Therefore, you also might compare typical SO interaction energies for 
elements in your structure with the energy differences between the 
ground states you worked out without SO. If you have light elements 
(small SO) and large Heisenberg exchange, then probably nothing will 
happen.


In addition you should be aware of the fact that Wien2k (or any other 
DFT program) will find (if you are lucky and everything works out) the 
ground state within the symmetry you set up during initialization. If 
you set up a hexagonal structure with some AF magnetic structure on top 
of it (maybe in a larger unit cell) your result from the scf cycle will 
have that symmetry. The actual ground state of the material you are 
interested in might be completely different. A drastic example: SO 
interaction may lead to non-collinear helical spin structures - which 
are outside the scope of standard Wien2k.



---
Dr. Martin Pieper
Karl-Franzens University
Institute of Physics
Universitätsplatz 5
A-8010 Graz
Austria
Tel.: +43-(0)316-380-8564


Am 30.10.2016 14:55, schrieb Abderrahmane Reggad:

Thanks Dr pieper for the rich information within your answer.

Perhaps I didn't formulated my question well . I am not interesting
exactly to the ground state energy , but to the magnetic ground state.
As I mentionned before , I want to determine the magnetic ground state
from 5 configurations: nm,fm, afm1,afm2 and afm3 for hexagonal crystal
structure (NiAs structure exatly); and I want to know if the inclusion
or not of the SO coupling affect this magnetic ground state.i.e.
without SO , I got a magnetic ground state , and if I include the SO,
may be this leads to another magnetic ground state.

Best regards

--

Mr:
A.Reggad  

Laboratoire de Génie Physique
Université Ibn Khaldoun - Tiaret

Algerie


___
Wien mailing list
Wien@zeus.theochem.tuwien.ac.at
http://zeus.theochem.tuwien.ac.at/mailman/listinfo/wien
SEARCH the MAILING-LIST at:
http://www.mail-archive.com/wien@zeus.theochem.tuwien.ac.at/index.html

___
Wien mailing list
Wien@zeus.theochem.tuwien.ac.at
http://zeus.theochem.tuwien.ac.at/mailman/listinfo/wien
SEARCH the MAILING-LIST at:  
http://www.mail-archive.com/wien@zeus.theochem.tuwien.ac.at/index.html


Re: [Wien] Is there an effect of SO on the ground state

2016-10-30 Thread pieper

My 2 cents to this: Not a particularly meaningful question.

The only meaningful thing about energies, as in eigenvalues of the 
Hamiltonian, is their DIFFERENCE. The evolution with time of the state 
of an isokated quantum system (Schrödinger picture) is completely 
governed by the differences of the eigenvalues of the Hamiltonian. The 
choice of where you put zero is up to you.


A common choice for zero is the ground state energy of the Hamiltonian. 
In this case clearly zero is zero, there is no effect of SO.


Other choices are more practical in various circumstances: The mean of 
the spektrum of the Hamiltonian, or the Fermi energy (definition 
discussed recently here), or some fictitious reference limit like the 
state with all charges at infinite distance ... The ground state energy 
then probably will depend on whether or not you put SO into your 
Hamiltonian. After all you consider SO because it changes energy 
differences and might split degeneracies.



---
Dr. Martin Pieper
Karl-Franzens University
Institute of Physics
Universitätsplatz 5
A-8010 Graz
Austria
Tel.: +43-(0)316-380-8564


Am 29.10.2016 23:20, schrieb Abderrahmane Reggad:

Hello again

I am waiting for an answer to my question .

My question is about the effect of the inclusion of the Spin-Orbit
Coupling on the ground state energy. 

I want to know if the SO affect the ground state energy also or It
only causes the splitting of the degenerate state energies.

Any information will be fruitful for me.

I hope find an answer to my question

--

Mr:
A.Reggad  

Laboratoire de Génie Physique
Université Ibn Khaldoun - Tiaret

Algerie


___
Wien mailing list
Wien@zeus.theochem.tuwien.ac.at
http://zeus.theochem.tuwien.ac.at/mailman/listinfo/wien
SEARCH the MAILING-LIST at:
http://www.mail-archive.com/wien@zeus.theochem.tuwien.ac.at/index.html

___
Wien mailing list
Wien@zeus.theochem.tuwien.ac.at
http://zeus.theochem.tuwien.ac.at/mailman/listinfo/wien
SEARCH the MAILING-LIST at:  
http://www.mail-archive.com/wien@zeus.theochem.tuwien.ac.at/index.html


Re: [Wien] 答复: HELP! cd $PWD; $t $exe ${def}_${loop}.def $loop; rm -f .lock_$lockfile[$p] ) >> .time2_$loop LAPW2 END

2016-07-21 Thread pieper
Since I am unable to think of a reason to HIDE not even the error 
messages or warnings, but only the flag indicating them, I stick to my 
first guess: echo messages from your job(s) informing you what is 
currently done. To stop this you probably will have to look into your 
shell commands, but there I am no epert and I dont know anything about 
your server anyway.



---
Dr. Martin Pieper
Karl-Franzens University
Institute of Physics
Universitätsplatz 5
A-8010 Graz
Austria
Tel.: +43-(0)316-380-8564


Am 21.07.2016 17:40, schrieb Jing Qun:

Thanks.

These are neither ERROR messages nor WARNINGS.

During the calculations, everything is OK.

But these messages were not found during the calculation using other
HPC server, I wonder if these messages are hidden ERROR or WARNINGS.

-

发件人: Wien <wien-boun...@zeus.theochem.tuwien.ac.at> 代表
pieper <pie...@ifp.tuwien.ac.at>
 发送时间: 2016年7月21日 8:44
 收件人: A Mailing list for WIEN2k users
 主题: Re: [Wien] HELP! cd $PWD; $t $exe ${def}_${loop}.def $loop;
rm -f .lock_$lockfile[$p] ) >> .time2_$loop LAPW2 END

Question is, is there anything wrong? Are these parts of ERROR
messages
 or at least WARNINGS, or are they just echoes, your job(s) telling
you
 what they are doing?

 ---
 Dr. Martin Pieper
 Karl-Franzens University
 Institute of Physics
 Universitätsplatz 5
 A-8010 Graz
 Austria
 Tel.: +43-(0)316-380-8564

 Am 21.07.2016 07:07, schrieb Jing Qun:
 > Dear all users,
 > When I submit a job, some messages like "
 > cd $PWD; $t $exe ${def}_${loop}.def $loop; rm -f
.lock_$lockfile[$p] )
 >>> .time2_$loop
 > LAPW2 END " will be obtained. Can anyone tell what's wrong with it?
 >
 >
 > ___
 > Wien mailing list
 > Wien@zeus.theochem.tuwien.ac.at
 > http://zeus.theochem.tuwien.ac.at/mailman/listinfo/wien [1]

 Wien -- A Mailing list for WIEN2k users [1]
 zeus.theochem.tuwien.ac.at
 A Mailing list for WIEN2k users. Please post questions, suggestions
or comments about WIEN2k ONLY in this list. Please follow the
following "Nettiquette" (depending ...

 > SEARCH the MAILING-LIST at:
 >
http://www.mail-archive.com/wien@zeus.theochem.tuwien.ac.at/index.html
[2]
 ___
 Wien mailing list
 Wien@zeus.theochem.tuwien.ac.at
 http://zeus.theochem.tuwien.ac.at/mailman/listinfo/wien [1]
 SEARCH the MAILING-LIST at:
http://www.mail-archive.com/wien@zeus.theochem.tuwien.ac.at/index.html
[2]


Links:
--
[1] http://zeus.theochem.tuwien.ac.at/mailman/listinfo/wien
[2] 
http://www.mail-archive.com/wien@zeus.theochem.tuwien.ac.at/index.html


___
Wien mailing list
Wien@zeus.theochem.tuwien.ac.at
http://zeus.theochem.tuwien.ac.at/mailman/listinfo/wien
SEARCH the MAILING-LIST at:
http://www.mail-archive.com/wien@zeus.theochem.tuwien.ac.at/index.html

___
Wien mailing list
Wien@zeus.theochem.tuwien.ac.at
http://zeus.theochem.tuwien.ac.at/mailman/listinfo/wien
SEARCH the MAILING-LIST at:  
http://www.mail-archive.com/wien@zeus.theochem.tuwien.ac.at/index.html


Re: [Wien] HELP! cd $PWD; $t $exe ${def}_${loop}.def $loop; rm -f .lock_$lockfile[$p] ) >> .time2_$loop LAPW2 END

2016-07-21 Thread pieper
Question is, is there anything wrong? Are these parts of ERROR messages 
or at least WARNINGS, or are they just echoes, your job(s) telling you 
what they are doing?



---
Dr. Martin Pieper
Karl-Franzens University
Institute of Physics
Universitätsplatz 5
A-8010 Graz
Austria
Tel.: +43-(0)316-380-8564


Am 21.07.2016 07:07, schrieb Jing Qun:

Dear all users,
When I submit a job, some messages like "
cd $PWD; $t $exe ${def}_${loop}.def $loop; rm -f .lock_$lockfile[$p] )

.time2_$loop

 LAPW2 END " will be obtained. Can anyone tell what's wrong with it?


___
Wien mailing list
Wien@zeus.theochem.tuwien.ac.at
http://zeus.theochem.tuwien.ac.at/mailman/listinfo/wien
SEARCH the MAILING-LIST at:
http://www.mail-archive.com/wien@zeus.theochem.tuwien.ac.at/index.html

___
Wien mailing list
Wien@zeus.theochem.tuwien.ac.at
http://zeus.theochem.tuwien.ac.at/mailman/listinfo/wien
SEARCH the MAILING-LIST at:  
http://www.mail-archive.com/wien@zeus.theochem.tuwien.ac.at/index.html


Re: [Wien] x nn failed

2016-05-19 Thread pieper

There is far too little information to really help, but:

- is your .struct file ok? Do you have one in the case directory? Does 
the structure look ok when viewed with xcrysden?
- did you succeed with the TiC example as described extensively in the 
UG?

- Were there any WARNINGS or even ERRORS during installation?
- What does the error message file of nn say?

A recommendation: As a beginner use the w2web interface! It goes a long 
way to help keeping the correct sequence of tasks, setting defaults and 
so on.


Good luck

---
Dr. Martin Pieper
Karl-Franzens University
Institute of Physics
Universitätsplatz 5
A-8010 Graz
Austria
Tel.: +43-(0)316-380-8564


Am 18.05.2016 16:52, schrieb ben abdallah houda:

Dear users
I have just finish installation of wien2k code, but when I start with
x nn it writes x nn failed. Please can someone help me.
Thanks for your help

Envoyé depuis Yahoo Mail pour Android [1]

Links:
--
[1] https://overview.mail.yahoo.com/mobile/?.src=Android

___
Wien mailing list
Wien@zeus.theochem.tuwien.ac.at
http://zeus.theochem.tuwien.ac.at/mailman/listinfo/wien
SEARCH the MAILING-LIST at:
http://www.mail-archive.com/wien@zeus.theochem.tuwien.ac.at/index.html

___
Wien mailing list
Wien@zeus.theochem.tuwien.ac.at
http://zeus.theochem.tuwien.ac.at/mailman/listinfo/wien
SEARCH the MAILING-LIST at:  
http://www.mail-archive.com/wien@zeus.theochem.tuwien.ac.at/index.html


Re: [Wien] Fwd: Strain

2015-12-07 Thread pieper

Hello Muhammad!

A DFT code will not simply say *crack* at some point of strain. Wien2k 
will always distribute the force you apply according to the perfect 
translational symmetry of your crystal structure - in contrast to nature 
where things like cracking or breaking occure at weak links - points 
where this translational symmetry is broken and the bonding is weaker.


DFT will do its very best to distribute the electrons in any structure 
you throw at it and tell you (an upper bound of) the minimum energy of 
the configuration.  I am no expert but I understand that from a 
calculated variation of total energy with lattice parameter you can 
estimate the intrinsic strength of a structure within some model for the 
process.


In addition Wien2k gives information where the total energy is stored, 
for example the forces on atoms in your structure. Therefore, if you 
look at the total energy of your strained structure you will see that it 
is larger - at 5% distortion perhaps considerably so. Depending on site 
symmetry some calculated internal forces may indicate where in the 
structure the 'springs' you pulled are located. Furthermore the phonon 
code might inform you of instabilities in your structure.


Good luck,

Martin  Pieper


---
Dr. Martin Pieper
Karl-Franzens University
Institute of Physics
Universitätsplatz 5
A-8010 Graz
Austria
Tel.: +43-(0)316-380-8564


Am 04.12.2015 15:25, schrieb Muhammad Sajjad:

Dear Tomas and Fecher
Thank you very much. It is working perfect with lattice type F and
symmetry operations equal to 16. But one thing is confusing me that I
applied 5 % strain to bulk Si and still it is semiconductor with gap
0.2eV. Is it correct? as 5 % is too much strain and bulk materials
tolerate very low strain. In my thinking it should break.

On Fri, Dec 4, 2015 at 5:14 PM, Fecher, Gerhard <fec...@uni-mainz.de>
wrote:


but your structure becomes tetragonal
you should reset the symmetry operations to (generate) to be
recalculated during initialisation

Ciao
Gerhard

DEEP THOUGHT in D. Adams; Hitchhikers Guide to the Galaxy:
"I think the problem, to be quite honest with you,
is that you have never actually known what the question is."


Dr. Gerhard H. Fecher
Institut of Inorganic and Analytical Chemistry
Johannes Gutenberg - University
55099 Mainz
and
Max Planck Institute for Chemical Physics of Solids
01187 Dresden

Von: wien-boun...@zeus.theochem.tuwien.ac.at
[wien-boun...@zeus.theochem.tuwien.ac.at] im Auftrag von Muhammad
Sajjad [sajja...@gmail.com]
Gesendet: Freitag, 4. Dezember 2015 15:08
An: A Mailing list for WIEN2k users
Betreff: Re: [Wien] Fwd: Strain

I am sorry having no idea about it. Simply I used optimized
structure and increased a by a0*1.001 and decreased b & c by keeping
lattice type F. Then run init_lapw -b.

On Fri, Dec 4, 2015 at 4:51 PM, Fecher, Gerhard
<fec...@uni-mainz.de<mailto:fec...@uni-mainz.de>> wrote:
you applied a tetragonal distortion along x
but how did you manage that you still have 48 symmetry operations ?

Ciao
Gerhard

DEEP THOUGHT in D. Adams; Hitchhikers Guide to the Galaxy:
"I think the problem, to be quite honest with you,
is that you have never actually known what the question is."


Dr. Gerhard H. Fecher
Institut of Inorganic and Analytical Chemistry
Johannes Gutenberg - University
55099 Mainz
and
Max Planck Institute for Chemical Physics of Solids
01187 Dresden

Von:


wien-boun...@zeus.theochem.tuwien.ac.at<mailto:wien-boun...@zeus.theochem.tuwien.ac.at>



[wien-boun...@zeus.theochem.tuwien.ac.at<mailto:wien-boun...@zeus.theochem.tuwien.ac.at>]

im Auftrag von Muhammad Sajjad
[sajja...@gmail.com<mailto:sajja...@gmail.com>]

Gesendet: Freitag, 4. Dezember 2015 14:04
An: wien
Betreff: [Wien] Fwd: Strain

Dear All
I got Si structure from wien2k examples and after optimization it
is
SILICON
F   LATTICE,NONEQUIV.ATOMS:  1
MODE OF CALC=RELA unit=ang
 10.305626 10.305626 10.305626 90.00 90.00 90.00
ATOM   1: X=0.1250 Y=0.1250 Z=0.1250
          MULT= 2          ISPLIT= 2
       1: X=0.8750 Y=0.8750 Z=0.8750
Si         NPT=  781  R0=0.0001 RMT= 2.22        Z:
14.0
LOCAL ROT MATRIX:    1.000 0.000 0.000
                     0.000 1.000 0.000
                     0.000 0.000 1.000
  48      NUMBER OF SYMMETRY OPERATIONS

mBJLDA gives band gap of 1.19 eV (perfect).

 Now I apply strain (0.1 %) and structure is

SILICON
F   LATTICE,NONEQUIV.ATOMS:  1
MODE OF CALC=RELA unit=ang
 10.315932 10.300477 10.300477 90.00 90.00 90.00
ATOM  -1: X=0.1250 Y=0.1250 Z=0.1250
          MULT= 2          ISPLIT=-2
      -1: X=0.8750 Y=0.8750 Z=0.8750
Si         NPT=  781  R0=0.0001 RMT= 2.22        Z:
14.0
LOCAL ROT MATRIX:    0.000 0.000 

Re: [Wien] Calculate spin orbit coupling with external magnetic field (ORB package)

2015-11-11 Thread pieper
Re-reading the thread my guess is that (as usual) Peter was right from 
the start: Nobody tried before ... if symmetso is compatible with nmod=3 
in inorb. Your difficulties indicate that it's not. One additional thing 
I can think of to check there is wether the direction of M in your 
case.inso and the field direction in your case.inorb are compatible.


However, such an incompatibility would not imply that SO and external 
field in ORB are incompatibel. At least I myself initialized SO (using 
w2web) always BEFORE turning on any orbital potential and did at least 
one SCF + save. Actually, the reason was some warning sentence in the UG 
sec. 7.3 about possible instabilities of the SCF for LDA+U without SO. 
Maybe I circumvented your initialization problems that way.


Good luck

Martin


---
Dr. Martin Pieper
Karl-Franzens University
Institute of Physics
Universitätsplatz 5
A-8010 Graz
Austria
Tel.: +43-(0)316-380-8564


Am 10.11.2015 22:06, schrieb Jing-Han Chen:

Thank Gerhard and Martin for checking and trying this question with me.

I am using wien2k 14.2 and case.indmc.
Beside the issues pointed out by Gerhard and Martin, another thing I
noticed is that symmetso has reading error of case.inorb only if the
structure has more than one site.
It might be the reason why the error does not appear in Pt.


2015-11-10 5:10 GMT-06:00 Fecher, Gerhard <fec...@uni-mainz.de>:

indeed, the same with me
as I just noted: the problem is that w2web creates .indm and likes to 
create a new one if its missing, even if you have a correct .indmc.



Ciao
Gerhard

DEEP THOUGHT in D. Adams; Hitchhikers Guide to the Galaxy:
"I think the problem, to be quite honest with you,
is that you have never actually known what the question is."


Dr. Gerhard H. Fecher
Institut of Inorganic and Analytical Chemistry
Johannes Gutenberg - University
55099 Mainz
and
Max Planck Institute for Chemical Physics of Solids
01187 Dresden

Von: wien-boun...@zeus.theochem.tuwien.ac.at 
[wien-boun...@zeus.theochem.tuwien.ac.at] im Auftrag von pieper 
[pie...@ifp.tuwien.ac.at]

Gesendet: Dienstag, 10. November 2015 11:47
An: A Mailing list for WIEN2k users
Betreff: Re: [Wien] Calculate spin orbit coupling with external 
magnetic field (ORB package)


Being a stupid and lazy person I always use the w2web interface, and 
the

*_so versions stay hidden with that.

I would assume that they are temporary versions, probably built by the
script to be renamed at the end to case.indm(c) and case.inorb.

I would like to point out again what Gerhard said: this is a magnetic
case so you have to use complex versions, especially case.indmc!

Good luck

Martin


---
Dr. Martin Pieper
Karl-Franzens University
Institute of Physics
Universitätsplatz 5
A-8010 Graz
Austria
Tel.: +43-(0)316-380-8564


Am 09.11.2015 21:59, schrieb Jing-Han Chen:

Thanks for the comments from Martin, Gerhard and Peter.
I assumes all inputs accurate since ORB and SOC can be run
individually.

It works successfully if I change case.inorb to some other name 
before

x symmetso and create it manually.

I have one further question.
The script symmetso generates case.inorb_so and case.indm_so.
Should they be the same as case.inorb case.indm respectively for SOC
with external magnetic field, or should they be kept as empty after
symmetso?




2015-11-09 3:40 GMT-06:00 pieper <pie...@ifp.tuwien.ac.at>:


The same with me: I would have to dig through old archives to find 
out

what
I actually did, but I am fairly sure that I used SO + external field 
a

few
years ago (probably Wien2k 10 or 12) - and don't recall any
incompatibilities between SO and external field at the time.

Keep fingers crossed that it checks out with an input file error

Martin


---
Dr. Martin Pieper
Karl-Franzens University
Institute of Physics
Universitätsplatz 5
A-8010 Graz
Austria
Tel.: +43-(0)316-380-8564



Am 09.11.2015 08:51, schrieb Fecher, Gerhard:


I tried it once for Pt and it worked
most probably there is an error in one of the input files inorb,
indm,
inso, or wherever else
(maybe "c" versions of the input files are needed, or the
m-directions
are not consistent).

I don't remember any conflict between initso and inorb.


Ciao
Gerhard

DEEP THOUGHT in D. Adams; Hitchhikers Guide to the Galaxy:
"I think the problem, to be quite honest with you,
is that you have never actually known what the question is."


Dr. Gerhard H. Fecher
Institut of Inorganic and Analytical Chemistry
Johannes Gutenberg - University
55099 Mainz
and
Max Planck Institute for Chemical Physics of Solids
01187 Dresden

Von: wien-boun...@zeus.theochem.tuwien.ac.at
[wien-boun...@zeus.theochem.tuwien.ac.at] im Auftrag von Peter 
Blaha

[pbl...@theochem.tuwien.ac.at]
Gesendet: Montag, 9. November 2015 07:17
An: A Mailing list for WIEN2k users
Betreff

Re: [Wien] large deviation of atomic volume in BiNi compound

2015-11-10 Thread pieper

Dear Tomas,

at the moment I do not see something being obviously suspicious. Maybe 
the culprit is some structural phase transition invalidating the 
experimental structure you compare with. You might get one or two ideas 
from Stefaan Cottenier's work?


Error Estimates for Solid-State Density-Functional
Theory Predictions: An Overview by Means of the
Ground-State Elemental Crystals
K. Lejaeghere , V. Van Speybroeck , G. Van Oost & S. Cottenier

http://dx.doi.org/10.1080/10408436.2013.772503

Best regards,

Martin


---
Dr. Martin Pieper
Karl-Franzens University
Institute of Physics
Universitätsplatz 5
A-8010 Graz
Austria
Tel.: +43-(0)316-380-8564


Am 10.11.2015 13:05, schrieb Tomas Kana:

Dear Martin and Gerhard,

Thank you for your suggestions. Gerhard, thank you for mentioning this


experimental work. Will you please send me the pdf

of the article? I do not have access to it.

Regarding Martin's questions:

I tried to include magnetism

of the constituents by performing spin polarized calculations, too,

but the equilbrium volume was the same. The forces within the
hexagonal unit cell

were not given in case.scf (I think there was too much symmetry
operations).

However, I recently tried to express the hexagonal unit cell in a
orthorhombic base

and cancel the symmetry operations by using inequivalent atoms

(I send the structure file in attachment). The volume was still wrong
but

I know the values of the forces. For the experimental

atomic volume they were at most 0.84 mRy/a.u.

With best regards

Tomas Kana


Since you ask for ideas and without really looking at the problem:
Assuming that the experimental numbers are correct, is this a room
temperature structure? The calculations are, of course, ground state

zero Kelvin, so things might go south if there is a phase transition

somewhere. Considering the elements you deal with maybe magnetic?
What
are the forces in your calculations?

Good luck,

Martin


___
Wien mailing list
Wien@zeus.theochem.tuwien.ac.at
http://zeus.theochem.tuwien.ac.at/mailman/listinfo/wien
SEARCH the MAILING-LIST at:
http://www.mail-archive.com/wien@zeus.theochem.tuwien.ac.at/index.html

___
Wien mailing list
Wien@zeus.theochem.tuwien.ac.at
http://zeus.theochem.tuwien.ac.at/mailman/listinfo/wien
SEARCH the MAILING-LIST at:  
http://www.mail-archive.com/wien@zeus.theochem.tuwien.ac.at/index.html


Re: [Wien] large deviation of atomic volume in BiNi compound

2015-11-10 Thread pieper
Since you ask for ideas and without really looking at the problem: 
Assuming that the experimental numbers are correct, is this a room 
temperature structure? The calculations are, of course, ground state 
zero Kelvin, so things might go south if there is a phase transition 
somewhere. Considering the elements you deal with maybe magnetic? What 
are the forces in your calculations?


Good luck,

Martin


---
Dr. Martin Pieper
Karl-Franzens University
Institute of Physics
Universitätsplatz 5
A-8010 Graz
Austria
Tel.: +43-(0)316-380-8564


Am 10.11.2015 10:21, schrieb Tomas Kana:

Dear Wien2k users,

I came across a problem with equilibrium atomic volume of

the BiNi compound. The experimental lattice is hexagonal

with a = 4.079 Angstroem, c = 5.359 Angstroem

(P. Villars, Pearson's Handbook: Crystallographic Data for
Intermetallic Phases)

However, the equilibrium volume turns out to be more

than 16 % higher than the experimental one.

I wonder since the equilibrium volume of

pure Bi and Bi3Ni comes out with much better agreement with

experiment (about 4 to 5 % deviation).
I used GGA (no spin orbit coupling),

Rmt*Kmax = 8.8, lmax = 10, Gmax = 16, 5000 k-points in the

whole Brillouin zone. I enclosethe structure file in attachment.

I tried LDA that gives better agreement with experiment

(about 10 % deviation) but I think this is still too much. I have
tried

to use gaussian smearing instead of the tetrahedron method,
increase Rmt*Kmax to 11, increase k-points to 20 000 in the whole
Brillouin zone but nothing helped.
In the mailing list I found someone had similar problem with a more
complicated structure containing bismuth, but that was not solved:
http://www.mail-archive.com/wien%40zeus.theochem.tuwien.ac.at/msg10479.html
Do you have any idea?
Thank you in advance
With best regards
Tomas Kana
Institute of Physics of Materials,
Academy of Sciences of the Czech Republic
Brno, Czech Republic

___
Wien mailing list
Wien@zeus.theochem.tuwien.ac.at
http://zeus.theochem.tuwien.ac.at/mailman/listinfo/wien
SEARCH the MAILING-LIST at:
http://www.mail-archive.com/wien@zeus.theochem.tuwien.ac.at/index.html

___
Wien mailing list
Wien@zeus.theochem.tuwien.ac.at
http://zeus.theochem.tuwien.ac.at/mailman/listinfo/wien
SEARCH the MAILING-LIST at:  
http://www.mail-archive.com/wien@zeus.theochem.tuwien.ac.at/index.html


Re: [Wien] Calculate spin orbit coupling with external magnetic field (ORB package)

2015-11-10 Thread pieper
Being a stupid and lazy person I always use the w2web interface, and the 
*_so versions stay hidden with that.


I would assume that they are temporary versions, probably built by the 
script to be renamed at the end to case.indm(c) and case.inorb.


I would like to point out again what Gerhard said: this is a magnetic 
case so you have to use complex versions, especially case.indmc!


Good luck

Martin


---
Dr. Martin Pieper
Karl-Franzens University
Institute of Physics
Universitätsplatz 5
A-8010 Graz
Austria
Tel.: +43-(0)316-380-8564


Am 09.11.2015 21:59, schrieb Jing-Han Chen:

Thanks for the comments from Martin, Gerhard and Peter.
I assumes all inputs accurate since ORB and SOC can be run 
individually.


It works successfully if I change case.inorb to some other name before
x symmetso and create it manually.

I have one further question.
The script symmetso generates case.inorb_so and case.indm_so.
Should they be the same as case.inorb case.indm respectively for SOC
with external magnetic field, or should they be kept as empty after
symmetso?




2015-11-09 3:40 GMT-06:00 pieper <pie...@ifp.tuwien.ac.at>:


The same with me: I would have to dig through old archives to find out 
what
I actually did, but I am fairly sure that I used SO + external field a 
few

years ago (probably Wien2k 10 or 12) - and don't recall any
incompatibilities between SO and external field at the time.

Keep fingers crossed that it checks out with an input file error

Martin


---
Dr. Martin Pieper
Karl-Franzens University
Institute of Physics
Universitätsplatz 5
A-8010 Graz
Austria
Tel.: +43-(0)316-380-8564



Am 09.11.2015 08:51, schrieb Fecher, Gerhard:


I tried it once for Pt and it worked
most probably there is an error in one of the input files inorb, 
indm,

inso, or wherever else
(maybe "c" versions of the input files are needed, or the 
m-directions

are not consistent).

I don't remember any conflict between initso and inorb.


Ciao
Gerhard

DEEP THOUGHT in D. Adams; Hitchhikers Guide to the Galaxy:
"I think the problem, to be quite honest with you,
is that you have never actually known what the question is."


Dr. Gerhard H. Fecher
Institut of Inorganic and Analytical Chemistry
Johannes Gutenberg - University
55099 Mainz
and
Max Planck Institute for Chemical Physics of Solids
01187 Dresden

Von: wien-boun...@zeus.theochem.tuwien.ac.at
[wien-boun...@zeus.theochem.tuwien.ac.at] im Auftrag von Peter Blaha
[pbl...@theochem.tuwien.ac.at]
Gesendet: Montag, 9. November 2015 07:17
An: A Mailing list for WIEN2k users
Betreff: Re: [Wien] Calculate spin orbit coupling with external
magnetic field (ORB package)

Probably nobody has ever tried it with a magnetic field in 
case.inorb.


Move cse.inorb to some other name before   x symmetso and create it
manually (Just check, if the number of non-equivalent atoms has 
changed

or not).

Am 08.11.2015 um 20:38 schrieb Jing-Han Chen:


Dear All

I am trying to calculate the effect of spin-orbit coupling while the
external magnetic field is specified by ORB package. However, it
continues to give an error of reading the case.inorb during "x
symmetso", one of initso_lapw step. I wonder whether SOC is actually
not compatible with the nmod=3 ORB. Did anyone have the experience
about this?



--
--
Peter BLAHA, Inst.f. Materials Chemistry, TU Vienna, A-1060 Vienna
Phone: +43-1-58801-165300 FAX: +43-1-58801-165982
Email: bl...@theochem.tuwien.ac.atWIEN2k: http://www.wien2k.at
WWW:   http://www.imc.tuwien.ac.at/staff/tc_group_e.php
--
___
Wien mailing list
Wien@zeus.theochem.tuwien.ac.at
http://zeus.theochem.tuwien.ac.at/mailman/listinfo/wien
SEARCH the MAILING-LIST at:
http://www.mail-archive.com/wien@zeus.theochem.tuwien.ac.at/index.html
___
Wien mailing list
Wien@zeus.theochem.tuwien.ac.at
http://zeus.theochem.tuwien.ac.at/mailman/listinfo/wien
SEARCH the MAILING-LIST at:
http://www.mail-archive.com/wien@zeus.theochem.tuwien.ac.at/index.html


___
Wien mailing list
Wien@zeus.theochem.tuwien.ac.at
http://zeus.theochem.tuwien.ac.at/mailman/listinfo/wien
SEARCH the MAILING-LIST at:
http://www.mail-archive.com/wien@zeus.theochem.tuwien.ac.at/index.html

___
Wien mailing list
Wien@zeus.theochem.tuwien.ac.at
http://zeus.theochem.tuwien.ac.at/mailman/listinfo/wien
SEARCH the MAILING-LIST at:  
http://www.mail-archive.com/wien@zeus.theochem.tuwien.ac.at/index.html


Re: [Wien] Calculate spin orbit coupling with external magnetic field (ORB package)

2015-11-09 Thread pieper


The same with me: I would have to dig through old archives to find out 
what I actually did, but I am fairly sure that I used SO + external 
field a few years ago (probably Wien2k 10 or 12) - and don't recall any 
incompatibilities between SO and external field at the time.


Keep fingers crossed that it checks out with an input file error

Martin


---
Dr. Martin Pieper
Karl-Franzens University
Institute of Physics
Universitätsplatz 5
A-8010 Graz
Austria
Tel.: +43-(0)316-380-8564


Am 09.11.2015 08:51, schrieb Fecher, Gerhard:

I tried it once for Pt and it worked
most probably there is an error in one of the input files inorb, indm,
inso, or wherever else
(maybe "c" versions of the input files are needed, or the m-directions
are not consistent).

I don't remember any conflict between initso and inorb.


Ciao
Gerhard

DEEP THOUGHT in D. Adams; Hitchhikers Guide to the Galaxy:
"I think the problem, to be quite honest with you,
is that you have never actually known what the question is."


Dr. Gerhard H. Fecher
Institut of Inorganic and Analytical Chemistry
Johannes Gutenberg - University
55099 Mainz
and
Max Planck Institute for Chemical Physics of Solids
01187 Dresden

Von: wien-boun...@zeus.theochem.tuwien.ac.at
[wien-boun...@zeus.theochem.tuwien.ac.at] im Auftrag von Peter Blaha
[pbl...@theochem.tuwien.ac.at]
Gesendet: Montag, 9. November 2015 07:17
An: A Mailing list for WIEN2k users
Betreff: Re: [Wien] Calculate spin orbit coupling with external
magnetic field (ORB package)

Probably nobody has ever tried it with a magnetic field in case.inorb.

Move cse.inorb to some other name before   x symmetso and create it
manually (Just check, if the number of non-equivalent atoms has changed
or not).

Am 08.11.2015 um 20:38 schrieb Jing-Han Chen:

Dear All

I am trying to calculate the effect of spin-orbit coupling while the
external magnetic field is specified by ORB package. However, it
continues to give an error of reading the case.inorb during "x
symmetso", one of initso_lapw step. I wonder whether SOC is actually
not compatible with the nmod=3 ORB. Did anyone have the experience
about this?



--
--
Peter BLAHA, Inst.f. Materials Chemistry, TU Vienna, A-1060 Vienna
Phone: +43-1-58801-165300 FAX: +43-1-58801-165982
Email: bl...@theochem.tuwien.ac.atWIEN2k: http://www.wien2k.at
WWW:   http://www.imc.tuwien.ac.at/staff/tc_group_e.php
--
___
Wien mailing list
Wien@zeus.theochem.tuwien.ac.at
http://zeus.theochem.tuwien.ac.at/mailman/listinfo/wien
SEARCH the MAILING-LIST at:
http://www.mail-archive.com/wien@zeus.theochem.tuwien.ac.at/index.html
___
Wien mailing list
Wien@zeus.theochem.tuwien.ac.at
http://zeus.theochem.tuwien.ac.at/mailman/listinfo/wien
SEARCH the MAILING-LIST at:
http://www.mail-archive.com/wien@zeus.theochem.tuwien.ac.at/index.html

___
Wien mailing list
Wien@zeus.theochem.tuwien.ac.at
http://zeus.theochem.tuwien.ac.at/mailman/listinfo/wien
SEARCH the MAILING-LIST at:  
http://www.mail-archive.com/wien@zeus.theochem.tuwien.ac.at/index.html


Re: [Wien] Can we perform layered AFM calculations with Wien2k?

2015-09-29 Thread pieper

Hello Krishnaveni Parthasarathy

Yes you can - though admittedly I did not look up your reference, and I 
might misinterprete your idea of 'layered AFM'. And I suppose you 
consider collinear magnetic moments. If that is not the case you might 
look into the (unsupported) WienNCM.


In general, study the UG, especially the chapters on AFM calculations.

You can think of any collinear AFM as ferromagnetic layers stacked along 
one particular crystalographic direction, the direction to the nearest 
antiparallel neighbor. For calculations with Wien2k you need a .strcut 
with a unit cell that has at least two magnetic atoms along that 
direction (perhaps from supercell if the crystalografic primitve cell is 
not large enough).


Since you want them to be magnetically inequivalent the two have to be 
on inequievalent crystalografic sites. If they are on crystalografic 
equivalent positions you have to split the position and number the atoms 
to tell Wien2k that they are not same (perhaps Mn1 and Mn2). Note that 
this results in a different symmetry of your structure.


If you want to compare total energies for different magnetic structures 
be careful to calculate all of them on the same crystalographic unit 
cell, changing only in the initialization which magnetic atom belongs to 
the group pointing 'up' and which one to the group pointing 'down'. If 
you have, for example, constructed a supercell with four inequivalent 
Mn-sites Mn1 .. Mn4, you can calculate AFM structures corresponding to 
Mn1, Mn2 up and Mn3, Mn4 down, or Mn1,3 up and Mn2,4 down, or ... This 
will correspond to ferromagnetic layers of Mn1, Mn2 stacked with 
antiparallel ferromagnetic layers Mn3,4, or ferromagnetic layers of 
Mn1,3 ... Not all of these arrangements are necessarily different. If 
the direction Mn1 - Mn2 is symmetry related to the one Mn1-Mn3 in the 
underlying crystalografic lattice, then there is no need to calculate 
with the stacking in these two equivalent directions. And obviously, for 
a complicated AF wave vector (the direction of the stacking), you will 
need large supercells so think, before you start.


Good luck,

Martin Pieper


---
Dr. Martin Pieper
Karl-Franzens University
Institute of Physics
Universitätsplatz 5
A-8010 Graz
Austria
Tel.: +43-(0)316-380-8564


Am 28.09.2015 18:43, schrieb Krishnaveni. S:

Ref: Electronic Structure, Chemical Bonding, and Finite-Temperature
Magnetic Properties of Full Heusler Alloys

Authors: YASEMIN KURTULUS, MICHAEL GILLEßEN, RICHARD DRONSKOWSKI*

DOI 10.1002/jcc.20308

online in Wiley InterScience (www.interscience.wiley.com [1]).

Ref to the article (Page 93) mentioned above, I understand that one
can perform layered calculation in AFM calculations.

In page 93, the authors have mentioned as below: “Here, the symmetry
has been slightly reduced to obtain a model in which the Mn planes can
be treated separately. This particular alignment is characterized by
alternating planes along (001) of spin-up and spin-down manganese
moments, and the identically oriented manganese moments within each
plane are coupled by the p orbitals of the Z elements. An alternative
anti ferromagnetic model with alternating magnetic planes along (111)
is also thinkable, but earlier total-energycalculations11 have already
clarified that the (001) model is characterized by lower energies in
almost all cases.”

 The author has specified that these calculations have been done in
VASP. Can some one help me understand if it is possible to perform
similar type of calculations in Wien2k?

--

Thanks and regards

Krishnaveni Parthasarathy
8939675012

Links:
--
[1] http://www.interscience.wiley.com

___
Wien mailing list
Wien@zeus.theochem.tuwien.ac.at
http://zeus.theochem.tuwien.ac.at/mailman/listinfo/wien
SEARCH the MAILING-LIST at:
http://www.mail-archive.com/wien@zeus.theochem.tuwien.ac.at/index.html

___
Wien mailing list
Wien@zeus.theochem.tuwien.ac.at
http://zeus.theochem.tuwien.ac.at/mailman/listinfo/wien
SEARCH the MAILING-LIST at:  
http://www.mail-archive.com/wien@zeus.theochem.tuwien.ac.at/index.html


Re: [Wien] Error in init_lapw

2015-09-22 Thread pieper

Dear Paresh,

your initial .struct is almost certainely wrong. The WARNINGs from nn 
indicate that you put two different atoms at the same locations, 
probably by entering them into equivalent crystalografic positions. Take 
nn's WARNINGs seriously: your calculations will almost certainely crash 
if already nn has problems. At least your calculations will be 
meaningless if you don't understand its complaints since in that case 
you don't understand what the structure the program works with looks 
like.


Start a new case in a fresh directory. Correct your structure, and 
inspect it with your favorite viewer (perhaps with xcrysden as 
recommended in the user guide). When it looks ok try nn again.


Best luck,

Martin


---
Dr. Martin Pieper
Karl-Franzens University
Institute of Physics
Universitätsplatz 5
A-8010 Graz
Austria
Tel.: +43-(0)316-380-8564


Am 22.09.2015 09:12, schrieb Paresh Chandra Rout:

Dear all,
I am facing a warning message while setting   nn-bondlength  as
follows
nn (12:18:17)  specify nn-bondlength factor: (usually=2) [and
optionally dlimit, dstmax (about
  1.d-5, 20)]
2
WARNING: Mult not equal. PLEASE CHECK outputnn-file 

WARNING: ityp not equal. PLEASE CHECK outputnn-file

NN created a new Bi2FeReO6_G-type.struct_nn file .

But when I use Old case.struct file I am able to enter into the next
step . But when I accept to use the new case.struct case it is just
get stuck there giving the error message as follows
 forrtl: severe (174): SIGSEGV, segmentation fault occurred
Image              PC                Routine      
     Line        Source             
nn                 0040779C  MAIN__            
       396  nn.f
nn                 0040312C  Unknown          
    Unknown  Unknown
libc.so.6          003F57A1ED5D  Unknown            
  Unknown  Unknown
nn                 00403029  Unknown          
    Unknown  Unknown
0.022u 0.002s 0:06.38 0.3% 0+0k 0+480io 0pf+0w
error: command   /home/paresh/Downloads/WIEN2k_14.2/nn nn.def  
failed
 n stop error n 

 I am proceeding with the old case.struct file . Is there any problem
if I use old case.struct file for the calculation ? If yes kindly give
me some direction  to resolve the above problem .
Any help would be highly appreciated .

Kind Regards
Paresh Chandra Rout
Research Scholar
Indian Institute of Science Education and Research Bhopal


___
Wien mailing list
Wien@zeus.theochem.tuwien.ac.at
http://zeus.theochem.tuwien.ac.at/mailman/listinfo/wien
SEARCH the MAILING-LIST at:
http://www.mail-archive.com/wien@zeus.theochem.tuwien.ac.at/index.html

___
Wien mailing list
Wien@zeus.theochem.tuwien.ac.at
http://zeus.theochem.tuwien.ac.at/mailman/listinfo/wien
SEARCH the MAILING-LIST at:  
http://www.mail-archive.com/wien@zeus.theochem.tuwien.ac.at/index.html


Re: [Wien] Diference in EFG values

2015-09-16 Thread pieper

Hello Stefaan and Muhammad

From the Wien2k UG, chapter 7.1 on lapw0, I take it that Wien2k 
calculates the potential, and from that the EFG, from the TOTAL electron 
density. For lapw0 explicitely including interstitials, for the 
decomposition in lapw2 (chapter 7.7) explicitely only the electron 
density in the atomic sphere. So as a reminder to Muhammad there may 
appear (minor) differences within Wien2k between the two methods.


But in my understanding both calculations in Wien2k always include core, 
semi-core, and valence electrons (within the atomic sphere)? The 
difference to VASP can then occure because that code only considers core 
states for the EFG?


Best regards,

Martin


---
Dr. Martin Pieper
Karl-Franzens University
Institute of Physics
Universitätsplatz 5
A-8010 Graz
Austria
Tel.: +43-(0)316-380-8564


Am 15.09.2015 13:55, schrieb Stefaan Cottenier:
According to my understanding, core-state with tag F are valence 
states.

AM I right? If yes then why it is taking 3P states as valence states?
The valence electrons for Co are 3d7, 4s2.


You use the words 'valence electrons' in the traditional 'chemical'
way as states outside the noble gas core. In the context of the DFT,
the term 'valence electrons' refers to all states that are not forced
to behave as in a free atom (i.e. they allow to feel the presence of
the surrounding crystal).

You cannot avoid to use Co-3p as valence states with wien2k, the
numerics would break down if you didn't.

Your output shows that wien2k has Co-3s and Co-3p as valence states,
whereas vasp considers these as core states (i.e. they will not
contribute to the EFG in vasp). Co-3s will never contribute to the EFG
(spherical), whereas Co-3p could (albeit not very likely).

First consider the suggestion by Peter Blaha: make sure you have
identical XC-functionals in both codes, and inspect whether all
magnetic moments and the DOS and band structure pictures agree for
both codes.

If that is all right, then use the procedure to which I refered in my
previous post to isolate the EFG contribution of Co-3p. If that is
nearly zero, I don't know what is going on. If it would be equal to
the vasp-wien2k difference, then you have found the origin of your
problem.

Stefaan

___
Wien mailing list
Wien@zeus.theochem.tuwien.ac.at
http://zeus.theochem.tuwien.ac.at/mailman/listinfo/wien
SEARCH the MAILING-LIST at:
http://www.mail-archive.com/wien@zeus.theochem.tuwien.ac.at/index.html

___
Wien mailing list
Wien@zeus.theochem.tuwien.ac.at
http://zeus.theochem.tuwien.ac.at/mailman/listinfo/wien
SEARCH the MAILING-LIST at:  
http://www.mail-archive.com/wien@zeus.theochem.tuwien.ac.at/index.html


Re: [Wien] need your help please

2015-09-16 Thread pieper

Hello Sikander,

you might want to read some introductionary chapter of your favorite 
book on statistical physics or on thermodynamics ... My first try at a 
short answer would be: Calculate microscopic properties from a proper 
Hamiltonian and density matrix for the system. Calculate macroscopic 
properties from ensemble averages of these.


I hope this does not add to your confusion,

Martin


---
Dr. Martin Pieper
Karl-Franzens University
Institute of Physics
Universitätsplatz 5
A-8010 Graz
Austria
Tel.: +43-(0)316-380-8564


Am 15.09.2015 10:20, schrieb sikander Azam:

Resp. all
I am confused a bit that the Properties like, Electronic structure,
optical and thermoelectric properties are microscopic or macroscopic
properties.
Regards
SIkander
___
Wien mailing list
Wien@zeus.theochem.tuwien.ac.at
http://zeus.theochem.tuwien.ac.at/mailman/listinfo/wien
SEARCH the MAILING-LIST at:
http://www.mail-archive.com/wien@zeus.theochem.tuwien.ac.at/index.html

___
Wien mailing list
Wien@zeus.theochem.tuwien.ac.at
http://zeus.theochem.tuwien.ac.at/mailman/listinfo/wien
SEARCH the MAILING-LIST at:  
http://www.mail-archive.com/wien@zeus.theochem.tuwien.ac.at/index.html


Re: [Wien] Energy vs Volume is linear!

2015-09-09 Thread pieper

Hello Marzieh,

I took the freedomn to use your data to test a little fitting procedure 
I just wrote. I tried to fit parameters a, b, c of the function

y=a(x-b)**2+c
to your data.

The fit crashed - probably like the one you may have used - when 
presented the data and asked to fit the parabola from scratch. The 
reason for such a behaviour of a fit procedure frequently is bad user 
behaviour: The program is quite understandably unable to guess 
reasonable starting values for the fit parameters from such data.


Presented with starting values
a=1.00e-06,   b=9.50e+03,  c=-8.857000e+04

the fit finds without complaining a very good fit with
a=5.687893e-07,  b=9.612186e+03,  c=-8.856770e+04

So you should do your DFT volume optimization around a volume of 9612 - 
and probably give a little thought to the problem of why you started so 
far off the optimum at larger volumes.


Good luck

Martin Pieper


---
Dr. Martin Pieper
Karl-Franzens University
Institute of Physics
Universitätsplatz 5
A-8010 Graz
Austria
Tel.: +43-(0)316-380-8564


Am 09.09.2015 03:43, schrieb Marzieh Gh:

Dear Prof.Blaha & Tran

I have calculated optimization of supercell (1*1*2). But Energy vs
Volume is linear!

case.outputeos:

   10575.3364   -88567.151612   
    11162.8553   -88566.366642
    12925.4112   -88561.471064   
    11750.3745   -88565.095045   
    12337.8929   -88563.437182

Is this normal?  If no what is reason? What do I do?

Please help me

Best Regards

--

Marzieh Ghoohestani
PhD Student of Computational Nano Physics
Nano Research Center, Department of Physics
University of Technology, Isfahan, Iran


___
Wien mailing list
Wien@zeus.theochem.tuwien.ac.at
http://zeus.theochem.tuwien.ac.at/mailman/listinfo/wien
SEARCH the MAILING-LIST at:
http://www.mail-archive.com/wien@zeus.theochem.tuwien.ac.at/index.html

___
Wien mailing list
Wien@zeus.theochem.tuwien.ac.at
http://zeus.theochem.tuwien.ac.at/mailman/listinfo/wien
SEARCH the MAILING-LIST at:  
http://www.mail-archive.com/wien@zeus.theochem.tuwien.ac.at/index.html


Re: [Wien] f orbital under an external magnetic field

2015-08-06 Thread pieper

Dear Bin Shao,

unfortunately I am travelling and won't be able to contribute during the 
next days. I am looking forward to comments from people with experience 
in calculations with rare earths.


May I just ask why you go for the energy and not for the magnetization 
or the susceptibility? If there is some change of the crystal field 
ground state this should show. From your calculation you get the size of 
the magnetic moments for a given field, from that you get a 
susceptibility. From what you say something happens around 4 T. I cannot 
guess from the information I have what, but I would expect it to show in 
the susceptibility as well.


Good luck with this interesting problem

Martin Pieper


---
Dr. Martin Pieper
Karl-Franzens University
Institute of Physics
Universitätsplatz 5
A-8010 Graz
Austria
Tel.: +43-(0)316-380-8564


Am 06.08.2015 15:47, schrieb Bin Shao:

Dear Martin Pieper,

Thank you for your comments!

Actually, I intend to demonstrate that the energy difference between
the ground state of Er^3+ (S=3/2; L=6; J=15/2) and the excited state
(S=3/2; L=0; J=3/2) can be tuned by the external magnetic field, With
the magnetic filed and the crystal field, the excited state splits
into four states, |+3/2, |+1/2, |-1/2, and |-3/2. For the 45 Tesla
magnetic field, the delta energy between the |+3/2 and |-3/2 is over
10 meV. Since we can not directly get the excited state in wien2k,
even by forcing the occupation number, the calculation will still be
trick. 

However, because the spin quantum number of the two states is the same
(S=3/2), there is no spin flip from the ground state to the excited
state. In this case, we can estimate the energy difference between the
ground state and the excited state by calculating the energy
difference between the occupied states of f electron in minority spin
of the ground state and the unoccupied counterparts in minority spin
of the ground state. The energy difference should become smaller with
increasing the magnetic field, which can be attributed to the lower in
energy of the |-3/2 state relative to the |+/-3/2 state with no
magnetic field.

Since the energy shift is in the magnitude of meV, we can not seen
this shift from the dos calculation due to the smear of the dos. Since
the f band is usually very local and the band is very flat, so I
checked the eigenvalues of the 7 f-electron at the Gamma point and try
to show the energy shift from the variations of the eigenvalues.
However, the results show that there is only an energy shift from the
0 T to 4 T. When the magnetic filed is increasing, the eigenvalues are
almost the same as that of 4 T.


This most probably is the old problem of the energy zero in
disguise.


This may be the problem. But I have calculated all the energy
differences between the 3 unoccupied and 4 occupied states of f
electron in minority spin, the 12 (3*4) values are keep the same trend
while the magnetic filed is varied and they are all flat. For the
different f states, they get different J and the energy shifts
(g_J*mu_B*J*B) induced by the magnetic filed should be also different.
So I am confused. It should be noted that the energy difference is
independent to the energy zero. 

Best,

Bin

On Thu, Aug 6, 2015 at 7:23 PM, pieper pie...@ifp.tuwien.ac.at
wrote:


As an afterthought:

This most probably is the old problem of the energy zero in
disguise. The Zeeman interaction you estimated and as accounted for
in Wien2k is basically g*mu_B*S*B. It gives you the energy
difference between a moment pointing up and one pointing down.
However, it has a vanishing trace, the zero is at B=0 and the center
stays there.

Best regards,

Martin Pieper

---
Dr. Martin Pieper
Karl-Franzens University
Institute of Physics
Universitätsplatz 5
A-8010 Graz
Austria
Tel.: +43-(0)316-380-8564 [3]

Am 06.08.2015 04:55, schrieb Bin Shao:


Dear all,

I made calculations of a compound with Er^3+(4f^11 5d^0 6s^0,
ground
state S=3/2, L=6, J=15/2) doping under an external magnetic
field. I
got the corresponding occupation of Er^3+ with 7 electrons in
majority
spin and 4 electrons in minority spin. With soc including, I got
eigenvalues at Gamma point of the Er^3+ under the magnetic field
from
4 Tesla to 45 Tesla. However, the picture indicates that the
eigenvalues with the different magnetic fields almost keep the
same as
that of 4 T. Why? According to a simple estimation, the magnetic
field
of 45 T will introduce an energy shift about 10 meV, that would
definitely be seen from the figure.

Any comments will be appreciated. Thank you in advance!

Best regards,

Bin

___
Wien mailing list
Wien@zeus.theochem.tuwien.ac.at
http://zeus.theochem.tuwien.ac.at/mailman/listinfo/wien [1]
SEARCH the MAILING-LIST at:




http://www.mail-archive.com/wien@zeus.theochem.tuwien.ac.at/index.html

[2]


___
Wien mailing list
Wien@zeus.theochem.tuwien.ac.at
http://zeus.theochem.tuwien.ac.at

Re: [Wien] (no subject)

2015-06-10 Thread pieper


Dear Sikander Azam,

there seems to be agreement that DFT calculates the ground state, and 
that this state is occupied at T=0 K. So there are two cases:


1) If comparison with your experiment works at some finite temperature 
it stands to reason that the probability of observing the ground state 
is large. Thermodynamics tells you that this should be the case when 
excitation energies (at least for operators related to your experiment) 
are large compared to thermal energy.


2) If comparison with your experiment does not work ... you might look 
in this mailing list if you did something wrong in the calculations. Or 
your computational model of the situation is wrong. Or your system is 
not in the ground state. Or ...


Best luck with your coparisons

Martin Pieper

---
Dr. Martin Pieper
Karl-Franzens University
Institute of Physics
Universitätsplatz 5
A-8010 Graz
Austria
Tel.: +43-(0)316-380-8564


Am 09.06.2015 12:02, schrieb sikandar azam:

Dear All
Please answer me this question
explain why zero kelvin DFT based calculations are compared with
experimentally calculated values at 0 K temp

with regards
sikander
___
Wien mailing list
Wien@zeus.theochem.tuwien.ac.at
http://zeus.theochem.tuwien.ac.at/mailman/listinfo/wien
SEARCH the MAILING-LIST at:
http://www.mail-archive.com/wien@zeus.theochem.tuwien.ac.at/index.html

___
Wien mailing list
Wien@zeus.theochem.tuwien.ac.at
http://zeus.theochem.tuwien.ac.at/mailman/listinfo/wien
SEARCH the MAILING-LIST at:  
http://www.mail-archive.com/wien@zeus.theochem.tuwien.ac.at/index.html


Re: [Wien] correct parameters for geometry optimization in magnetic states

2015-05-26 Thread pieper

Dear Prof. Kervan,

the easiest thing to do is use the case.struct file you have for the AFM 
calculation and initialize the calculation in a fresh directory with the 
same parameters as for AFM calculation, only point all magnetic moments 
in the same direction (for FM) or set them zero (for PM). Usually the 
scf-cycle does not change the initial magnetic state from FM (or PM) to 
AFM but stays in the local minimum of that spin arrangement.


Good luck,

Martin Pieper


---
Dr. Martin Pieper
Karl-Franzens University
Institute of Physics
Universitätsplatz 5
A-8010 Graz
Austria
Tel.: +43-(0)316-380-8564


Am 26.05.2015 09:39, schrieb Selcuk KERVAN:

Dear wien2k users,

During  the optimization for PM, FM and AFM states, ground state
energy  diffrences  are  too  small. For PM and FM calculations, space
group and number of k points are the same. But for AFM states, space
group  and  number  of  k  points  are  different.  So, how can I find
correctly the most stable magnetic states.

Thanks in advance,

**
Prof. Dr. Selçuk KERVANProf. Dr. Selçuk KERVAN
Gazi Üniversitesi  Gazi University
Polatlı Fen-Edebiyat FakültesiPolatlı Science and Arts Faculty
Fizik Bölümü Department of Physics
06900 Polatlı06900 Polatlı
ANKARA TÜRKİYE   ANKARA TURKEY

___
Wien mailing list
Wien@zeus.theochem.tuwien.ac.at
http://zeus.theochem.tuwien.ac.at/mailman/listinfo/wien
SEARCH the MAILING-LIST at:
http://www.mail-archive.com/wien@zeus.theochem.tuwien.ac.at/index.html

___
Wien mailing list
Wien@zeus.theochem.tuwien.ac.at
http://zeus.theochem.tuwien.ac.at/mailman/listinfo/wien
SEARCH the MAILING-LIST at:  
http://www.mail-archive.com/wien@zeus.theochem.tuwien.ac.at/index.html


Re: [Wien] error on SCF calculation

2015-05-20 Thread pieper

Dear Rachida,

Pavel asked you to check if lapw1c exists - the c at the end is 
important!


Does ZnS need the complex version?

Did you have any error messages during initialization?

Are you able to do SCF's on example structures (the TiC example)?

Best regards,

Martin Pieper

---
Dr. Martin Pieper
Karl-Franzens University
Institute of Physics
Universitätsplatz 5
A-8010 Graz
Austria
Tel.: +43-(0)316-380-8564


Am 20.05.2015 15:54, schrieb rachida lamouri:

Dear Pavel,
thank you for your contribution.
I checked if lapw1 exists and there is, and for version 14.2 I have
tried and I had the same problem.
I thought it may be due to an error in the installation, if there is a
trick or something to change during installation.

Best regards.
Rachida.


From: pavel.ondra...@email.cz
To: wien@zeus.theochem.tuwien.ac.at
Date: Wed, 20 May 2015 14:58:52 +0200
Subject: Re: [Wien] error on SCF calculation

Dear Rachida,

the hup: Command not found. line is harmless, however the lapw1c:
Command not found is not.
It looks like the lapw1c doesn't exist. It is possible it wasn't
compiled properly (especially since there are multiple known

problems

when compiling Wien2k 13 with gfortran). Check if the lapw1c binary
exists and if not then check your compile log for errors (file
compile.msg in SRC_lapw1 subfolder).
Also Wien2k 14.2 is the recommended version with many compilation

fixes,

so please upgrade to the latest version.

Best regards
Pavel


On Wed, 2015-05-20 at 13:18 +0100, rachida lamouri wrote:
 hello,
 I am running wien2k_13 with gfortran compiler.
 when i try to run SCF calculation for ZnS, i get this error msg :

 hup: Command not found.
 STOP LAPW0 END
 /home/rachida/wien2k/lapw1c: Command not found.

  stop error

 If you could help me I would be very honorable.
 Pending a favorable response, please accept my respectful

greetings.

 best regards.
 ___
 Wien mailing list
 Wien@zeus.theochem.tuwien.ac.at
 http://zeus.theochem.tuwien.ac.at/mailman/listinfo/wien
 SEARCH the MAILING-LIST at:

http://www.mail-archive.com/wien@zeus.theochem.tuwien.ac.at/index.html


___
Wien mailing list
Wien@zeus.theochem.tuwien.ac.at
http://zeus.theochem.tuwien.ac.at/mailman/listinfo/wien
SEARCH the MAILING-LIST at:

http://www.mail-archive.com/wien@zeus.theochem.tuwien.ac.at/index.html

___
Wien mailing list
Wien@zeus.theochem.tuwien.ac.at
http://zeus.theochem.tuwien.ac.at/mailman/listinfo/wien
SEARCH the MAILING-LIST at:
http://www.mail-archive.com/wien@zeus.theochem.tuwien.ac.at/index.html

___
Wien mailing list
Wien@zeus.theochem.tuwien.ac.at
http://zeus.theochem.tuwien.ac.at/mailman/listinfo/wien
SEARCH the MAILING-LIST at:  
http://www.mail-archive.com/wien@zeus.theochem.tuwien.ac.at/index.html


Re: [Wien] AFM calculations for YBCO6

2015-05-18 Thread pieper

Good evening,

Sorry, I commented first without remembering the structure correctly.

The antiferromagnetic and superconducting phase diagram of Y123O(6+x) 
has been studied experimentally very extensively in the 90's with - in 
my opinion - very clear results on the exact AF phases from neutron 
diffraction and NMR. I don't want to drown anyone in literature, so 
being an NMR guy I strongly suggest to look into the NMR work:


E. Brecht and W. W. Schmahl and H. Fuess and S. Schmenn and H. 
L{\u}tgemeier and N. H. Andersen and B. Lebech and Th. Wolf,
Neutron diffraction and NQR study of the intermediate turn angle phase 
formed during AFI to AFII reordering in 
{YBa$_2$Cu$_{3-x}$Al$_x$O$_{6+\delta}$},

Physical Review B, 1997, 56, no.2, p 940

Allow me to suggest in addition my own work

K. Nehrke and M. W. Pieper and T. Wolf,
Local magnetic properties of PrBa2Cu3O6+x: {NMR} and {NQR} in crystals,
Phys. Rev. B, 1996, 53, 1

Perhaps you can work from there through the references for details and 
additional studies.


There are two crystalografic different Cu sites in the perovskite, the 
so-called chain- and twice as many plane-sites. The variable oxygen 
content is between the Cu chain sites. In accord with what Prof. 
Delamora noted, the Cu on chains never carrys a magnetic moment. Cu in 
the planes forms a (well studied) spin-1/2 2D Heisenberg square lattice 
AF (nearest neighbors antiparallel) at low oxygen concentration (the 
Y123O6 end of the phase diagram), and the famous high-temperature 
superconductor when the oxygen sites between chain-Cu are filled, doping 
the electron holes into the planes.


If the system orders AF the planes ALWAYS form a square lattice spin-1/2 
AF with nearest neighbors antiparallel. So you need to assign numbers 
like Cu1 (for up) and Cu2 (for down) for Cu in the planes. The chain Cu 
is on a different crystalografic site and doesn't need numbering.


What I did not remember correctly is that since you have two plane site 
Cu even in the primitive cell you can, in fact, get away with a 1*1*1 
cell. BUT look in XCRYSDEN or something like it to make sure that your 
numbering makes nearest neighbors in the planes antiparallel! A 
configuration with parallel spins along the a- or b-axis should have 
significantly higher E_tot.


If you consider not only Y123O6 but introduce oxygen between Cu 
chain-sits you loose the 4-fold symmetry of the c-axis even in the 
non-magnetic case. This is why, as far as I recall, at least a 2*2*1 
supercell is necessary. To describe the different stacking sequences 
along the c-axis you will have to at least double the cell along the 
c-axis as well. See the above mentioned Phys Revs and the literature 
cited therein for the variety of AF phases that come about by different 
stacking of these planes along the c-axis depending on the oxygen 
content, or on the rare earth element.


As an additional comment: To my knowledge all evidence indicates that 
there is NO coexistence of AF-order and superconductivity. At 
intermediate oxygen content a phase separation occures into 
superconducting and AF phases.


Good luck with your calculations,

Martin Pieper

---
Dr. Martin Pieper
Karl-Franzens University
Institute of Physics
Universitätsplatz 5
A-8010 Graz
Austria
Tel.: +43-(0)316-380-8564


Am 16.05.2015 21:15, schrieb delamora:

This is a problem I had studied before, but non magnetic, so I was
curious to try the magnetic case; w
With YBa2Cu3O7 you have magnetism and superconductivty! braking one of
Matthias rules

The result I got for, YBa2Cu3O6, was that Cu1 MM=0.0007 or non
magnetic as far as the calculations show.
Cu2=0.5873 with U=4eV
So, you have to study only Cu2 for magnetism.
From my point of view you the most and I would guess only ordering of
the inplane spins is like a chess board, alternating un and dn.
the interplanar Cu2-Cu2 ordering can be up-up or up-dn, but since
there are no oxygen atoms between the planes the difference should be
quite small.

I did the calculation with RxK=7 and 100 k-points with 4 parallel
cores, it took half an hour.

In the supercell output you have to delete all the numbers
Ba 1 = Ba, O 1 = O and only number 1 and 2 the Cu as you want to 
order them

I did;
Cu 00x and 1/2,1/2,x as Cu 1 and later as 'up'
and for 0,1/2,x and 1/2,0,x as Cu 2 and later as 'dn'
Now you can have 00,-x as 'dn' and 0,1/2,-x as up and this gives
another ordering

De: wien-boun...@zeus.theochem.tuwien.ac.at
wien-boun...@zeus.theochem.tuwien.ac.at en nombre de Madesis
Ioannis(John) imade...@physics.uoc.gr
Enviado: sábado, 16 de mayo de 2015 01:09 p. m.
Para: Wien
Asunto: Re: [Wien] AFM calculations for YBCO6

Mr. Delamora, first of all thank you for your dedication, and thorough
examination of my problem.

I haven't fully tested your solution, however, there are plenty of AFM
orderings that I wish to test such as A, G and C type, all of which are
different combinations of in-plane

Re: [Wien] AFM calculations for YBCO6

2015-05-15 Thread pieper


Good eveneing, Ioannis Madesis

As you observed yourself: The (primitve) unit cell of your structure 
does not have enough Cu1 to support AFM on that sublattice. For that you 
need at least two Cu1 atoms. You achieve that by doubling (at least) the 
unit cell (perhaps using supercell) and assigning two different numbers 
to the Cu-sites you want to assign opposite spins to. As far as I recall 
you should double the unit cell along the a- AND the b-axis to be able 
to assign the correct spin directions. If you want to describe AF 
stacking sequences along the c-axis you will have to double the unit 
cell along that axis. Once you assigned numbers to Cu according to their 
two spin directions in your supercell (say, leave Cu1 for up, rename the 
Cu1 you want to point down into Cu3) you can let nn and symmetry do the 
job of finding the corresponding space group - which will necessarily be 
different from 123 (consult the UG for this procedure).


Good luck

Martin Pieper


---
Dr. Martin Pieper
Karl-Franzens University
Institute of Physics
Universitätsplatz 5
A-8010 Graz
Austria
Tel.: +43-(0)316-380-8564


Am 15.05.2015 14:32, schrieb Madesis Ioannis(John):

Good evening everyone

I am having trouble with the struct file for these calculations. To be
more specific: This material has 2 nonequivalent Cu atoms, Cu1 and
Cu2. Each one of these sits at the four corners of the unit cell, Cu1
at z=0 and Cu2 at approximately z=1/3, and the space group is 123.
In order to achieve AFM ordering, I need to have 2 types of Cu1, and
the spacegroup doesn't let me do that. Once I place Cu1(up), all 4
corners have Cu1(up). What can I do to solve this?

___
Wien mailing list
Wien@zeus.theochem.tuwien.ac.at
http://zeus.theochem.tuwien.ac.at/mailman/listinfo/wien
SEARCH the MAILING-LIST at:  
http://www.mail-archive.com/wien@zeus.theochem.tuwien.ac.at/index.html


Re: [Wien] A few (more) elementary -so questions (with onsite -eece)

2015-05-06 Thread pieper

Sorry, the comment was confusing.

The spin moments specified in the initial configuration of course point 
along the axis specified by the direction.


What I wanted to point out is that for symmetry reasons this is the only 
meaningfull component you can specify. In this sense symmetry comes 
first. The direction is more than just the spin direction. Its the only 
direction along which any vector operator that Wien2k can represent in 
the given case can point.


Someone should correct me if I am wrong, but in my understanding this is 
(one of) the reason(s) that Wien2k can only cope with collinear magnetic 
moments: The axial symmetry is introduced as a global symmetry. The 
direction of the axis is the same in all muffin-tin spheres. I expect 
the Wienncm code handles this on basis of the local symmetry.



---
Dr. Martin Pieper
Karl-Franzens University
Institute of Physics
Universitätsplatz 5
A-8010 Graz
Austria
Tel.: +43-(0)316-380-8564


Am 05.05.2015 19:31, schrieb Laurence Marks:

I would be interested in clarification from others, but from what I
can see in the code it appears that this is the spin direction that is
used, not just the direction of breaking the symmetry. I may be wrong.

On Tue, May 5, 2015 at 11:47 AM, pieper pie...@ifp.tuwien.ac.at
wrote:


I definitely am not an expert for -so, therefore I will not shoot
down
anything, only a comment:

 From my point of view from magnetism I would ask for some caution
with
identifying the direction given in .inorb and .inso with 'the spin
direction'. As Gerhard pointed out earlier in this thread, it's all
about symmetry: The specified direction only sets up the symmetry
of the
case to be compatibel with whatever has a rotational invariance
with
that (quantization) axis - be that a spin, or orbital moment,
magnetization, a magnetic field, ... The symmetry of the basis has
to
allow for a magnetization otherwise it won't appear when you
calculate
expectation values. Personally I find Pavel's 'lecture on
spin-orbit.ps [1]'
here in the Wien documentation files (I hope it's still there) very
illuminating.

---
Dr. Martin Pieper
Karl-Franzens University
Institute of Physics
Universitätsplatz 5
A-8010 Graz
Austria
Tel.: +43-(0)316-380-8564

Am 05.05.2015 14:51, schrieb Laurence Marks:

Me a culpa, I should have checked the mailing list first for the
answers.

That said, this issue has come up enough times in the past that I
think the UG should be tweaked so it is clearer. Let me try my
interpretation, so I can be shot down if needed.

Within Wien2k magnetic effects can be approximately included in a
number of ways. Some such as the spin-orbit coupling assume a
direction for the spin vector (for all electrons actively

considered),

others such as Bext in orb specify a direction for an applied

magnetic

field (in Tesla) and use the same direction for the spin vector.

(The

two spin states are then either parallel or anti parallel to the
specified direction.) When a direction is specified in case.inso

or

case.inorb this fixes the spin vector and (if used) the external
magnetic field direction. Via the output files from lapwdm
(case.scfdmXX) one can monitor how the angular momentum changes

[1].

By using different directions for the spin vector (and field) one

can

probe how the energy changes and/orbital occupancies with assumed
directions for the spin/external magnetic field.

To escape the assumption that the spin vectors all have one

direction

the Wienncm code has to be used.

[1] My addendum. Changes in the occupancies can be a soft

electronic

mode, i.e. very small changes in the energy for quite large

changes in

the density. The older mixing algorithms (MSEC1 or MSEC3) are not

so

good for soft modes and can stagnate. MSR1 is better and the next
release (7.0) is much better. With onsite -eece /or -orb it may

help

to push the mixer by either forcing a larger step (echo .2 

.msec

or echo .1  .pratt) or stopping, doing a force on the orbital
potential (x orb -up; x orb -dn) then restarting with -NI. It is
probably wise to check how the orbital momentum is converging

(grep

:ORB0 *.scf, perhaps other) and make sure that the mixing is not
starving (grep GREED: *.scf and check the values are not small,

e.g.

0.035).

___
  Professor Laurence Marks
  Department of Materials Science and Engineering
  Northwestern University
  www.numis.northwestern.edu [2] [1]
  MURI4D.numis.northwestern.edu [3] [2]



  Co-Editor, Acta Cryst A
  Research is to see what everybody else has seen, and to think

what

nobody else has thought
  Albert Szent-Gyorgi
On May 4, 2015 6:22 PM, Laurence Marks

l-ma...@northwestern.edu

wrote:


Typo:

although I remember don't symmetry operations being split into
these two classes everywhere in the code

On Mon, May 4, 2015 at 6:04 PM, Laurence Marks
l-ma...@northwestern.edu wrote:


I am a newbie at -so, so a few simple questions.

a) What is the meaning of the orbital moment in case.scfdm

Re: [Wien] A few (more) elementary -so questions (with onsite -eece)

2015-05-05 Thread pieper
I definitely am not an expert for -so, therefore I will not shoot down 
anything, only a comment:


From my point of view from magnetism I would ask for some caution with 
identifying the direction given in .inorb and .inso with 'the spin 
direction'. As Gerhard pointed out earlier in this thread, it's all 
about symmetry: The specified direction only sets up the symmetry of the 
case to be compatibel with whatever has a rotational invariance with 
that (quantization) axis - be that a spin, or orbital moment, 
magnetization, a magnetic field, ... The symmetry of the basis has to 
allow for a magnetization otherwise it won't appear when you calculate 
expectation values. Personally I find Pavel's 'lecture on spin-orbit.ps' 
here in the Wien documentation files (I hope it's still there) very 
illuminating.




---
Dr. Martin Pieper
Karl-Franzens University
Institute of Physics
Universitätsplatz 5
A-8010 Graz
Austria
Tel.: +43-(0)316-380-8564


Am 05.05.2015 14:51, schrieb Laurence Marks:

Me a culpa, I should have checked the mailing list first for the
answers.

That said, this issue has come up enough times in the past that I
think the UG should be tweaked so it is clearer. Let me try my
interpretation, so I can be shot down if needed.

Within Wien2k magnetic effects can be approximately included in a
number of ways. Some such as the spin-orbit coupling assume a
direction for the spin vector (for all electrons actively considered),
others such as Bext in orb specify a direction for an applied magnetic
field (in Tesla) and use the same direction for the spin vector. (The
two spin states are then either parallel or anti parallel to the
specified direction.) When a direction is specified in case.inso or
case.inorb this fixes the spin vector and (if used) the external
magnetic field direction. Via the output files from lapwdm
(case.scfdmXX) one can monitor how the angular momentum changes [1].
By using different directions for the spin vector (and field) one can
probe how the energy changes and/orbital occupancies with assumed
directions for the spin/external magnetic field.

To escape the assumption that the spin vectors all have one direction
the Wienncm code has to be used.

[1] My addendum. Changes in the occupancies can be a soft electronic
mode, i.e. very small changes in the energy for quite large changes in
the density. The older mixing algorithms (MSEC1 or MSEC3) are not so
good for soft modes and can stagnate. MSR1 is better and the next
release (7.0) is much better. With onsite -eece /or -orb it may help
to push the mixer by either forcing a larger step (echo .2  .msec
or echo .1  .pratt) or stopping, doing a force on the orbital
potential (x orb -up; x orb -dn) then restarting with -NI. It is
probably wise to check how the orbital momentum is converging (grep
:ORB0 *.scf, perhaps other) and make sure that the mixing is not
starving (grep GREED: *.scf and check the values are not small, e.g.
0.035).

___
 Professor Laurence Marks
 Department of Materials Science and Engineering
 Northwestern University
 www.numis.northwestern.edu [1]
 MURI4D.numis.northwestern.edu [2]
 Co-Editor, Acta Cryst A
 Research is to see what everybody else has seen, and to think what
nobody else has thought
 Albert Szent-Gyorgi
On May 4, 2015 6:22 PM, Laurence Marks l-ma...@northwestern.edu
wrote:


Typo:

although I remember don't symmetry operations being split into
these two classes everywhere in the code

On Mon, May 4, 2015 at 6:04 PM, Laurence Marks
l-ma...@northwestern.edu wrote:


I am a newbie at -so, so a few simple questions.

a) What is the meaning of the orbital moment in case.scfdm* ? Is
that the average direction projected to the global axis system?

b) What is the physical significance of the orbital moment being
parallel (or not quite parallel) to the direction used in
case.inso?

c) I understand that the results for different directions of B in
case.inso reflect the magnetic anisotropy, but what are the units
of field (if any)?

d) What else is worth looking at? The partial orbital moment
(:POM) seems relevant, but what exactly is it?

e) I am blindly trusting that initso knows what it is doing, and
have left the B symmetry operations in case.struct (although I
remember symmetry operations being split into these two classes
everywhere in the code). This seems to conflict with Pavel's
notes, although those may be too old.

Thanks.

--

Professor Laurence Marks
Department of Materials Science and Engineering
Northwestern University
www.numis.northwestern.edu [1]
Corrosion in 4D: MURI4D.numis.northwestern.edu [2]
Co-Editor, Acta Cryst A
Research is to see what everybody else has seen, and to think
what nobody else has thought
Albert Szent-Gyorgi


--

Professor Laurence Marks
Department of Materials Science and Engineering
Northwestern University
www.numis.northwestern.edu [1]
Corrosion in 4D: MURI4D.numis.northwestern.edu [2]
Co-Editor, Acta Cryst A
Research is to see what everybody

Re: [Wien] Physical significance of magnetization direction with -so

2015-05-04 Thread pieper

I hope my five cent might be usefull:

If you do have magnetic moments, be they ferro-, ferri-, or 
antiferromagnetic, or induced by an external field, the results can 
depend on the orientation of the moments. In addition, keep in mind that 
the various magnetic moments one likes to think of may not be constants 
of motion, or good quantum numbers, so they cannot be used to specify 
the eigenstates.


Inasmuch as S and L are good quantum numbers Hund's first and second 
rules for single ions state that the ground state of an electron shell 
in the Coulomb potential of the nuclear charge will have maximum total 
spin moment S=\sum s_i and maximum total angular momentum L=\sum l_i 
compatibel with the Pauli principle. L and S are constants of motion, 
they commute with the Hamiltonian of the nuclear Coulomb potential. L 
and S can, therefore, be used to enumerate the eigenstates. The energy 
is given by the size of L and S. The direction of L and S is unimportant 
(for a single ion without magnetic field applied!) the nuclear Coulomb 
potential is radial symmetric. Each state of the shell is 
(2L+1)(2S+1)-fold degenerate. This degeneracy is partially split by 
spin-orbit coupling and by the electrostatic crystal field.


You find the Hamiltionian for spin-orbit coupling H_so=\lambda (S*L) 
with S and L the vectors of the total spin and angular momentum of the 
electrons in one shell (say, d-, or f-shell) from considering the energy 
of an electron spin s in the magnetic field due to the relative motion 
of the nuclear charge with respect to the electron on its path at 
angular momentum l around the nucleus. With this H_so only J=L+S stays a 
constant of motion, neither S nor L. You can find the magnitude of J by 
Hund's third rule if H_so stays the most important player down to that 
energy. However, this is still just the radial symmetric potential of a 
single nuclear charge, the energy of the eigenstates is given by 
specifying J and you can point J in any direction.


In a crystal the charge distribution of the surrounding ions also acts 
on the electrons in the shell. This crystal field Hamiltonian obviously 
is not spherical symmertric, it will reduce the rotational symmetry to 
the point symmetry of the site. If there is a magnetic moment on an ion 
the energy will depend on its direction in the lattice. This is the 
source of the magnetic single-ion anisotropy.


What the eigenstates of the combined Hamiltonian look like depends on 
the relative size and symmetry of the contributions. For outer d-shells 
crystal fields usually dominate over H_so leading e.g. to what is called 
the quenching of the orbital angular momentum (L=0) which is sensitive 
to magnetic fields. For 4f-shells which are shielded by outer d- and 
s-shells H_so is frequently dominant and Hund's third rule often 
survives, allowing to calculate J and consider the dependance of the 
energy on its direction in the lattice.


Martin

---
Dr. Martin Pieper
Karl-Franzens University
Institute of Physics
Universitätsplatz 5
A-8010 Graz
Austria
Tel.: +43-(0)316-380-8564


Am 03.05.2015 19:37, schrieb Laurence Marks:

An elementary question: do the results of -so depend upon the
magnetization direction used in initso, or should they in principle be
independent of it?

--

Professor Laurence Marks
Department of Materials Science and Engineering
Northwestern University
www.numis.northwestern.edu [1]
Corrosion in 4D: MURI4D.numis.northwestern.edu [2]
Co-Editor, Acta Cryst A
Research is to see what everybody else has seen, and to think what
nobody else has thought
Albert Szent-Gyorgi

Links:
--
[1] http://www.numis.northwestern.edu
[2] http://MURI4D.numis.northwestern.edu

___
Wien mailing list
Wien@zeus.theochem.tuwien.ac.at
http://zeus.theochem.tuwien.ac.at/mailman/listinfo/wien
SEARCH the MAILING-LIST at:
http://www.mail-archive.com/wien@zeus.theochem.tuwien.ac.at/index.html

___
Wien mailing list
Wien@zeus.theochem.tuwien.ac.at
http://zeus.theochem.tuwien.ac.at/mailman/listinfo/wien
SEARCH the MAILING-LIST at:  
http://www.mail-archive.com/wien@zeus.theochem.tuwien.ac.at/index.html


Re: [Wien] Crystal field splitting

2014-11-04 Thread pieper

Dear Salman,

unfortunately I have neither means (no Wien2k on my notebook) nor time 
right now to quickly provide an answer to your problems. Anyhow, 
directing you to literature you can cite is probably better than an 
elaborate email lecture on crystal field splittings (not being really an 
expert I am not the ideal person to give such a lecture anyway).


To me the best way to proceed seems to be be:

- get your hands on some textbook on magnetism with a chapter on crystal 
field effects. Kei Yosida: Theory of Magnetism, Springer Series in Solid 
State Science vol 122 might be one choice. Personally, when I ran into 
crystal field effects a rather long time ago, I liked the article of M. 
T. Hutchings: Point charge ... in F. Seitz, D. Turnbull, Solid State 
Physics, vol. 16, p. 227, Academic Press 1964.


- read (understand) that chapter. You should know at this stage which 
parameters appear in the crystal field Hamiltonian if you know L and the 
point symmetry. I did not use Pavel Novaks new package for Wien2k up to 
now but I expect that you can understand its I/O on that basis. And you 
should be aware of the limitations of the concept, especially in a 
3d-metal!


- Find the configuration of B nearest neighbors for the two 
non-equivalent Ni sites in your structure. The idea is that charges 
farther out are pretty much shielded. Simply look at the structure using 
Xcrysden or something similar to identify this configuration. In Wien2k, 
outputnn gives you the number and position of these B atoms. My guess (a 
guess, not more!) is, that one Ni site has a (distorted) octahedral 
configuration, the other a (also distorted) tetrahedral one. CF 
splitting usually is considered in steps of decreasing importance of the 
contribution in the Hamiltonian: first approximate the situation by a 
cubic crystal field, then take into account, say, tetragonal distortions 
of the ideal octahedron, and so on, lowering symmetry in each step.


I hope this helps,

Martin


---
Dr. Martin Pieper
Karl-Franzens University
Institute of Physics
Universitätsplatz 5
A-8010 Graz
Austria
Tel.: +43-(0)316-380-8564


Am 03.11.2014 13:41, schrieb Salman Zarrini:

+++
Dear Martin and Delamora,

Many thanks for your answer, actually, controversial state for me here
 is 3d orbitals of Ni elements crystalline together with boron in
a  orthorhombic structure, Pnma space group. I can see different kind
of  point group in case.outputsgroup(1,1,C1 for sort one, m,m,Cs for
atom  sort 2 and m,m,Cs for atom sort 3 and a mmm,2/m,2/m 2/m,D2h in
the  end for the whole structure)but I can not make a link between
them and  potential crystal filed splitting for this structure, for
convenience  the struct file has been enclosed, I would be thankful if
you guided  me to find the proper crystal filed.

Cheers,
Salman
+++



___
Wien mailing list
Wien@zeus.theochem.tuwien.ac.at
http://zeus.theochem.tuwien.ac.at/mailman/listinfo/wien
SEARCH the MAILING-LIST at:
http://www.mail-archive.com/wien@zeus.theochem.tuwien.ac.at/index.html

___
Wien mailing list
Wien@zeus.theochem.tuwien.ac.at
http://zeus.theochem.tuwien.ac.at/mailman/listinfo/wien
SEARCH the MAILING-LIST at:  
http://www.mail-archive.com/wien@zeus.theochem.tuwien.ac.at/index.html


Re: [Wien] Crystal field splitting

2014-11-03 Thread pieper

Dear Salman,

to be a little bit more precise, crystal field splitting is a local 
concept: The Coulomb interaction of all electrons in a given shell 
characterized by its angular momentum with the surrounding charge 
distribution is represented by a minimal set of angular momentum 
operators. To make sense this should to be a shell of reasonably well 
localized electrons (usually a 4f-shell). If you have a metal the 
bandwidth of that particular band should be small, the conduction 
electrons should be contributed by other shells.


It also means that it is NOT the overall crystal symmetry you have to 
consider, but the point symmetry group of the crystalografic site 
occupied by the (Rare Earth?) element you are interested in. When your 
structure has sites with, say, octahedral and tetrahedral symmetry the 
crystal field Hamiltonian (and its eigenvalues, the splitting) will 
reflect these symmetries. The point group symmetries are tabulated, e.g. 
for the Wyckoff positions at the Bilbao server 
(http://www.cryst.ehu.es/)


Best regards,

Martin


---
Dr. Martin Pieper
Karl-Franzens University
Institute of Physics
Universitätsplatz 5
A-8010 Graz
Austria
Tel.: +43-(0)316-380-8564


Am 03.11.2014 03:56, schrieb delamora:

The crystal field splitting depends on the crystal symmetry!!, but for
a compound with metallic characteristics the bandwidth will be larger
that the field splitting.

De: wien-boun...@zeus.theochem.tuwien.ac.at
wien-boun...@zeus.theochem.tuwien.ac.at en nombre de Salman Zarrini
salman.zarr...@tu-darmstadt.de
Enviado: domingo, 02 de noviembre de 2014 04:57 p.m.
Para: wien@zeus.theochem.tuwien.ac.at
Asunto: [Wien] Crystal field splitting

Dear Wien2k users,

I was wondering that how can I find out which kinds of crystal field
splitting (Octahedral, Tetrahedral, Pentagonal bipyramidal,...) have
been applied on my bulk metallic structure?

Best regards,

Salman

___
Wien mailing list
Wien@zeus.theochem.tuwien.ac.at
http://zeus.theochem.tuwien.ac.at/mailman/listinfo/wien
SEARCH the MAILING-LIST at:
http://www.mail-archive.com/wien@zeus.theochem.tuwien.ac.at/index.html
___
Wien mailing list
Wien@zeus.theochem.tuwien.ac.at
http://zeus.theochem.tuwien.ac.at/mailman/listinfo/wien
SEARCH the MAILING-LIST at:
http://www.mail-archive.com/wien@zeus.theochem.tuwien.ac.at/index.html

___
Wien mailing list
Wien@zeus.theochem.tuwien.ac.at
http://zeus.theochem.tuwien.ac.at/mailman/listinfo/wien
SEARCH the MAILING-LIST at:  
http://www.mail-archive.com/wien@zeus.theochem.tuwien.ac.at/index.html


  1   2   >